Você está na página 1de 190

QUMICA

2013
PROGRAMA
NACIONAL
OLIMPADAS
DE QUMICA
Olimpada
Brasileira
de Qumica
VI Olimpiada
Brasileira de
Qumica Jnior
XIX Olimpada
Norte/Nordeste
de Qumica
18 Olimpada
Ibero-americana
de Qumica
45rd International
Chemistry
Olympiad
www.obquimica.org
Prof. Jesualdo Pereira Farias
Reitor da Universidade Federal do Cear
Prof. Jos Arimatia Dantas Lopes
Reitor da Universidade Federal do Piau
Prof
a
. Mrcia Maria Tavares Machado
Pr-Reitora de Extenso da UFC
Profa. Maria da Glria Carvalho Moura
Pr-Reitora de Extenso da UFPI
Prof. Alvaro Chrispino
Presidente da Associao Brasileira de Qumica

Prof. Srgio Maia Melo
Coordenador do Programa Nacional Olimpadas de Qumica
Prof. Jos Arimatia Dantas Lopes
Vice-coordenador do Programa Nacional Olimpadas de Qumica e
Coordenador da Equipe Pedaggica (docentes do Estado do Piau)
Cristiano de A.C. Marcelino Jr.
Ricardo Oliveira da Silva
Srgio Maia Melo
Jisa Bezerra Vila Nova
Marlene Barbosa Vieira
Luiz Henrique Soares
Preparao dos exames experimentais e edio de ftas de vdeo

ISSN: 1809-2012
Imprensa Universitria
Universidade Federal do Cear
Organizao de originais:
Prof. Srgio Melo

Capa:
Maherle
Editorao e Projeto Grfco:
Maherle/Srgio Melo
2012 Programa Nacional Olimpadas de Qumica
Lanamento em 29.11.2013 por ocasio da solenidade de encerramento dos eventos: XIX Olimpada Norte/
Nordeste de Qumica, Olimpada Brasileira de Qumica Jnior e Olimpada Brasileira de Qumica - 2013
Tiragem: 15.000 exemplares.
Distribuio gratuita.
QUMICA

N
D
I
C
E
2
0
1
3
Parte 1 - Olimpadas Nacionais de Qumica
Calendrio . . . . . . . . . . . . . . . . . . . . . . . . . . . . . . . . 04
Mensagem . . . . . . . . . . . . . . . . . . . . . . . . . . . . . . . . 05
Opinio . . . . . . . . . . . . . . . . . . . . . . . . . . . . . . . . . . 07
XIX Olimpada Norte/Nordeste
Exames . . . . . . . . . . . . . . . . . . . . . . . . . . . . . 09
Solues escolhidas . . . . . . . . . . . . . . . . . . . . . . . 14
Resultados . . . . . . . . . . . . . . . . . . . . . . . . . . . 21
VI OBQ Junior
Exames Fase I . . . . . . . . . . . . . . . . . . . . . . . . . . 23
Exames Fase II . . . . . . . . . . . . . . . . . . . . . . . . . . 30
Solues escolhidas . . . . . . . . . . . . . . . . . . . . . . . 37
Resultados . . . . . . . . . . . . . . . . . . . . . . . . . . . 40
OBQ 2013
Fase III Modalidade A. . . . . . . . . . . . . . . . . . . . . . . 43
Fase III Modalidade B . . . . . . . . . . . . . . . . . . . . . . 52
Resultados . . . . . . . . . . . . . . . . . . . . . . . . . . . 63
Parte 2 - Processo seletivo para compor a equipe nas olimpadas internacionais de 2013
PROCESSO SELETIVO . . . . . . . . . . . . . . . . . . . . . . . . . . . . 68
OBQ Fase IV . . . . . . . . . . . . . . . . . . . . . . . . . . . . . . . . 68
OBQ FASE V . . . . . . . . . . . . . . . . . . . . . . . . . . . . . . . . 70
OBQ FASE VI . . . . . . . . . . . . . . . . . . . . . . . . . . . . . . . . 72
Resultado Final . . . . . . . . . . . . . . . . . . . . . . . . . 78
OBQ Treinamento laboratorial . . . . . . . . . . . . . . . . . . . . . . . . 79
45
th
International Chemistry Olympiad . . . . . . . . . . . . . . . . . . 80
Exame Terico . . . . . . . . . . . . . . . . . . . . . . . . . . 80
Exame Prtico . . . . . . . . . . . . . . . . . . . . . . . . . 114
18 Olimpada Iberoamericana . . . . . . . . . . . . . . . . . . . . . 141
Exame Prtico . . . . . . . . . . . . . . . . . . . . . . . . . 141
Exame Terico . . . . . . . . . . . . . . . . . . . . . . . . . 153
Destaques Olmpicos . . . . . . . . . . . . . . . . . . . . . . . . . . . 175
Depoimentos . . . . . . . . . . . . . . . . . . . . . . . . 178
Consideraes Finais . . . . . . . . . . . . . . . . . . . . . . . . . . . 182
Endereos dos Cooordenadores . . . . . . . . . . . . . . . . . . . . . . 184
Programa Nacional Olimpadas de Qumica <<
4
Calendrio 2013
Data Hora Atividade
26/01/2013 14:00h Fase IV da OBQ-2012 (Exame sobre tcnicas laboratoriais com o objetivo
de selecionar a equipe que representar o Brasil na Rssia e na Bolvia).
25/02/2013 a
08/03/2013
8h-12h e
14h-18h
Curso de Aprofundamento e Excelncia (Fase V) para os 15 estudantes
selecionados no exame de conhecimentos de laboratrio. Ser ministrado
pela UFPI. Estudantes se deslocaro at a cidade-sede.
06/04/2013 9:00h Exames da Olimpada Brasileira de Qumica - 2012 Fase VI - Questes de
baseadas na lista sugerida pelos organizadores da 45th IChO.
30/04/2013 23:00h Divulgao dos nomes dos quatro estudantes que representaro o Brasil
nas competies internacionais
25/05/2013 14:00h XIX Olimpada Norte/Nordeste de Qumica - XIX ONNeQ. Cinquenta
estudantes por estado. Inscries restritas aos coordenadores estaduais.
03/07/2013 23:00h Divulgao de resultados da Olimpada Norte/Nordeste de Qumica - XVIX
ONNeQ.
06/05 a
08/06/2013
On line Inscries para a VI Olimpada Brasileira de Qumica Jnior. Escolas
inscrevem seus alunos de 8 e 9 anos do ensino fundamental. On line.
Entre 08/06 e
08/06/2013
14:00h Exames da VI Olimpada Brasileira de Qumica Jnior - OBQjr,
para estudantes de 8 e 9 anos. (Fase I). Lanar notas at 20/06/2013
15 a
24/07/2013
45 Olimpada Internacional de Qumica, Moscou - Rssia
01 a
15/05/2013
Inscries para a Olimpada Brasileira de Qumica - 2013. Vinte e cinco
estudantes por estado na modalidade A (estudantes da penltima srie do
ensino mdio ou srie anterior, 25 estudantes por estado na modalidade B
(3 srie). Inscries reservadas ao coordenador estadual.
31/08/2013 14:00h Exames da Olimpada Brasileira de Qumica - 2013. Fase III - Modalidades A
e B. Questes analtico-expositivas.
12 a
19/10/2013
18 Olimpada Ibero-americana de Qumica, La Paz - Bolvia
05/10/2013 14:00h Exames da VI OBQjr (FaseII). Obs.: s sero recebidos os exames da 2 fase
se postados nos Correios at 11/outubro
30/10/2013 23h Divulgao dos resultados da OBQ-2013 a partir de 30/10/2013
30/10/2013 23h Divulgao de resultados da VI Olimpada Brasileira de Qumica Jnior,
VI OBQjr, a partir de 30/10/2013.
28 a
29/11/2013
8:30h a
18:00h
Reunio do Conselho de coordenadores, em Fortaleza.
29/11/2013 19:00h Solenidade Nacional de encerramento e premiao das Olimpadas de
Qumica, em Fortaleza.
>> Olimpada Brasileira de Qumica - 2013
5
Mensagem
O ensino de qumica na Educao Bsica de muitas
escolas brasileiras ganhou uma nova motivao com
a realizao das Olimpadas Brasileira de Qumica.
Anualmente, coordenadores de ensino em qumica de
escolas participantes, professores e alunos, trabalham
para que seus desempenhos nas Olimpadas sejam os
melhores possveis. A constatao anual da movimen-
tao de todos os entes escolares, com vistas ao bom
resultado das Escolas, representadas por seus respec-
tivos alunos, tem, sem sombra de dvidas, contribu-
do para a melhoria do ensino de qumica na Educao
Bsica Brasileira. Os resultados colhidos so amplamente divulgados e usados
como peas publicitrias para mostrar a qualidades das escolas. Apropriam-se
dessa divulgao Escolas privadas e pblicas; Secretarias Municipais e Estaduais
de Educao e Governos.
Outro aspecto relevante do que se tem observado, a partir do surgimento
das Olimpadas Brasileira de Qumica, o forte intercambio entre professores e
pesquisadores da rea, principalmente, da educao bsica e universitria. Este
intercmbio, que se fortalece a cada evento, constitui um dos alcances que pro-
puseram os idealizadores dessas Olimpadas: levar conhecimento e despertar
o interesse pela cincia qumica logo nas primeiras fases de formao dos es-
tudantes. Este intento tem sido motivador para insero de muitos jovens na
formao em qumica, seja na graduao e/ou nas ps-graduaes.
Ainda, devemos reconhecer o papel central, na realizao desses eventos,
desempenhado pelos parceiros CNPq, CAPES, Universidades, Escolas e Secreta-
ria de Educao Bsica do MEC. Essa articulao interinstitucional tem permiti-
do aproximar os rgos responsveis pelo fomento das aes de pesquisa e ino-
vao tecnolgica, pela formao de professores e/ou de profssionais docentes,
com aqueles que so responsveis por dirigir a poltica de formao bsica dos
estudantes brasileiros. Fruto disso, os resultados no poderiam ser outros, jo-
vens incentivados para o desenvolvimento de pesquisas acadmicas, vocacio-
nados com o esprito inventivo de novos conhecimentos, produtos e tecnolo-
gias. Tudo isso nos permite ter a certeza que essas aes trazem perspectivas
Mensagem do reitor da Universidade Estadual do
Piau aos participantes das Olimpadas de Qumica
Programa Nacional Olimpadas de Qumica <<
6
Mensagem
de contribuio signifcativa para o desenvolvimento do nosso pas, bem como
melhorar nossa visibilidade no cenrio da educao na rea de qumica tanto
nacional quanto internacionalmente.
Finalmente, quero parabenizar a todos os professores, gestores e dirigentes
institucionais e os discentes; que ano a ano contribuem para a realizao das
Olimpadas Brasileira de Qumica, tornando-a um grande evento. E que ela pos-
sa se fortalecer e continuar revelando talentos; jovens futuros pesquisadores,
como muito bem os resultados anteriores tem demonstrado ser possvel.
Nouga Cardoso Batista (*)
Vice Reitor (2010-2013), Reitor eleito (2014-2017)
da Universidade Estadual do Piau UESPI
*Graduado em Licenciatura Plena em Cincias
habilitao em Qumica UFPI (1993).
Mestre em Qumica Analtica IQSC/USP (1995).
Doutor em Fsico Qumica IQSC/USP (1999).
>> Olimpada Brasileira de Qumica - 2013
7
BEIJO DO DESPREZO
O abandono de nossas escolas no mata diretamente, mas difculta o futuro de
cada criana que no estuda No difcil perceber como as manchetes das revis-
tas do ltimo fm de semana se referem tragdia humana da boate Kiss de Santa
Maria: Quando o Brasil vai aprender?, A asfxia no acabou, To jovens, to
rpido e to absurdo e Futuro roubado. tambm uma tragdia que pode ser
associada s escolas de todo o Brasil. como se a boate de Santa Maria fosse uma
metfora da escola brasileira. Na primeira delas, os jovens perderam a vida por
inalar um gs venenoso; na outra as crianas perdem o futuro por no inalarem
o oxignio do conhecimento. A imprevidncia de proprietrios, msicos e fscais
levou morte por falta de ar; a de polticos, pais e eleitores leva a uma vida in-
completa por falta de educao. A tragdia despertou para os riscos que correm
nossos jovens em seus fns de semana em boates, mas ainda no despertou para o
que perdem nossas crianas e jovens no dia a dia de suas escolas. Estamos fechan-
do boates sem sistemas de segurana, mas ainda deixamos abertas escolas sem
qualidade. Os pais comearam a no deixar seus flhos irem a boates inseguras,
mas levam confantemente suas crianas a escolas que no asseguram o futuro
delas. Exigimos que as boates tenham portas de emergncia, mas no exigimos
que as escolas sejam a porta para o futuro das crianas. A tragdia de Santa Maria
provoca a percepo imediata da fragilidade vergonhosa na segurana de boates,
mas a tragdia de nossa educao, apesar de suas vtimas, no percebida. Isto
porque ela uma tragdia qual nos acostumamos e nos embrutece, ou porque
so crianas invisveis pela pobreza, ou ainda porque somos um povo sem gosto
pela antecipao, s ouvimos o grito de fogo e vemos a fumaa depois que matam.
Por isso fechamos os olhos tragdia da educao que hoje devasta a economia,
a poltica e o tecido social do Brasil. O abandono de nossas escolas no mata di-
retamente, mas difculta o futuro de cada criana que no estuda. Se as escolas
fossem de qualidade para todos, teramos menos violncia urbana, maior produti-
vidade, mais avanos no mundo das invenes de novas tecnologias e um pas me-
lhor. Por isso, ao mesmo tempo em que choramos as trgicas mortes dos jovens
de Santa Maria, choremos tambm pelo futuro das crianas que no vo receber
a educao necessria para enfrentar o mundo. Choremos pelos que perderam a
vida na boate ao respirar o ar venenoso, e pelos que no vo receber nas escolas
Opinio
Programa Nacional Olimpadas de Qumica <<
8
o ar puro do conhecimento. No vamos recuperar as vidas eliminadas na boate
Kiss, podemos apenas chorar e nos envergonhar. Mas podemos evitar o desperd-
cio das vidas que esto hoje nas Escolas Kiss: metfora que une boate e escola,
sobretudo, quando lembramos que a boate se chamava Kiss, nome que tambm
deveramos dar s nossas escolas de hoje: beijo do desprezo. Desprezo pelas vidas
de jovens ou pelo futuro de nossas crianas.
CristovamBuarque
PublicadoemOGLOBO-09/02/2013
Opinio
A educao faz com que as pessoas sejam fceis de
guiar, mas difceis de arrastar; fceis de governar,
mas impossveis de escravizar.
Herry Peter
>> Olimpada Brasileira de Qumica - 2013
9
XIX ONNeQ
XIX Olimpada Norte/Nordeste de Qumica
25/05/2013 - Tempo de durao: 4 horas
AT E N O:
Cada questo deve ser respondida em folha nica.
Use o verso da folha se necessrio.
No responda 2 ou mais questes em uma folha.
Colocar apenas seu nmero de identifcao na folha de resposta (Inscrio).
Observe que a QUESTO 2 ser respondida em uma folha diferente das demais, nela voc ir traar um grfco
no espao quadriculado.
Devolver apenas as folhas de respostas, leve o caderno de perguntas para casa..
QUESTO 1
O arsnio muitas vezes obtido por aquecimento da arsenopirita minrio, FeAsS.
A decomposio do FeAsS representada pela equao equilibrada abaixo.
FeAsS(s) FeS (s )+As(g)
Na fase slida, o arsnio ocorre em duas formas. Numa forma, o arsnio amare-
lo, tem uma densidade de 1,97 g cm
-3
a temperatura e presso padro. A outra
forma, o arsnio cinzento, tem uma densidade de 5,78 g cm
-3
a temperatura e
presso padro. Quando o arsnio aquecido rapidamente em ar, xido de ars-
nio (III) formado. Apesar de o arsnico ser txico, o corpo humano necessita de
quantidades muito pequenas. O corpo de um adulto saudvel humano contm
cerca de 5 mg de arsnio.
Converter a massa de arsnio encontrado no corpo de uma pessoa adulta saud-
vel para gramas.
Quando aquecido, uma amostra de 125,0 kg de arsenopirita produz 67,5 kg de
FeS. Determinar a massa total de arsnio produzida nesta reao.
Escrever a reao balanceada quando o arsnio aquecido rapidamente ao ar.
Em termos de arranjo de tomos, por que as duas formas de arsnio tm densi-
dades diferentes em temperatura e presso padro.
Programa Nacional Olimpadas de Qumica <<
10
XIX ONNeQ
Calcule a composio percentual em massa de arsnio na arsenopirita.
DADOS:Massasmolares,considere:As=74,9g/mol;Fe=55,8g/mol;S=32,1g/mol
QUESTO 2
Em um ensaio experimental, 50,0 mililitros HCl (aq) de concentrao desconhe-
cida titulada com NaOH 0,10 mol L
-1
(aq). Durante a titulao, o volume total
de NaOH (aq) adicionado e o valor de pH correspondente da mistura reacional
so medidos e registados conforme mostrado na tabela abaixo.
Volume Total de
NaOH(aq)Adicionado(mL)
daMisturaReacional
ValordepH
10,0 1,6
20,0 2,2
24,0 2,9
24,9 3,9
25,1 10,1
26,0 11,1
30,0 11,8
(i) Plote os dados da tabela, circule e ligue os pontos.
(ii) Determine o volume total de NaOH (aq) adicionado quando a mistura rea-
cional tem um valor de pH prximo de 7.
(iii) Escreva a equao balanceada que representa essa reao de neutraliza-
o.
(iv) Em outro estudo, 40,0 mL de HCl (aq) completamente neutralizada por
20,0 mL de 0,10 mol L
-1
de NaOH (aq). Calcule a concentrao molar do
cido titulado no ensaio.
>> Olimpada Brasileira de Qumica - 2013
11
XIX ONNeQ
QUESTO 3
O radioistopo urnio-238 ocorre naturalmente na crosta terrestre. A desinte-
grao do radioistopo a primeira de uma srie de decaimentos espontneos.
O sexto decaimento desta srie produz o radioistopo de radnio-222. O decai-
mento do radnio-222 produz o radioistopo polnio-218, que tem uma meia-
-vida de 3,04 minutos. Eventualmente, o istopo estvel de chumbo-206 pro-
duzido pelo decaimento alfa de um nucldeo instvel.
Explique, em termos de confgurao eletrnica, por que tomos de um radioi-
stopo produzido pelo sexto decaimento da srie desintegrao do U-238 no
reagem facilmente para formar compostos.
Mostre a equao nuclear para o decaimento do nucldeo instvel que produz
Pb-206.
Determinar a massa original de uma amostra de Po-218, se 0,50 miligrama da
amostra permanece inalterada depois de 12,16 minutos.
DADOS: No. atmico Pb = 82
QUESTO 4
Numa investigao de laboratrio, uma amostra
de 50,0 gramas de cobre aquecida 100,0
o
C num
banho de gua fervente.

Um copo de isopor com uma tampa usado como
um calormetro. Nele contm 100,0 g de gua des-
tilada temperatura de 23,2
o
C.
Programa Nacional Olimpadas de Qumica <<
12

O cobre quente despejado no copo de gua, e o
copo rapidamente coberto com a tampa.

Um termmetro inserido atravs em um orifcio
na tampa. O cobre e gua so suavemente agita-
dos no copo. A temperatura verifcada periodi-
camente. A maior temperatura registrada foi de
26,3
o
C.
(i) Em termos de fuxo de energia, explique por que a temperatura da gua
aumenta no calormetro.
(ii) Em uma tabela estruture ou organize os dados das grandezas fsicas obser-
vados no experimente.
(iii) Estime a quantidade de calor, em Joule, ganho pela gua.
(iv) Estime a capacidade calorfca deste aparente calormetro rudimentar.
(v) Na presente investigao, a alterao no calor de cobre maior do que a al-
terao no calor da gua. O erro pode explicar essa aparente violao da Lei
de Conservao de Energia? No use o erro humano como parte da resposta.
Dados: calor especfco da gua: c(H
2
O) = 4,18 J g
-1
K
-1
; calor especfco do
cobre:c(Cu)=0,387Jg
-1
K
-1
.
XIX ONNeQ
>> Olimpada Brasileira de Qumica - 2013
13
XIX ONNeQ
QUESTO 5
A gasolina uma mistura composta principalmente por hidrocarbonetos, tais
como iso-octano, o qual tambm conhecido como 2,2,4-trimetilpentano. A ga-
solina atribuda um nmero chamado de ndice de octano. A gasolina com um
ndice de octanas de 87 tem a mesma forma de uma mistura que consiste em 87
% de iso-octano e 13 % de heptano.
Um combustvel alternativo, E-15, pode ser utilizado em alguns automveis.
Este combustvel uma mistura de 85 % de gasolina e 15 % de etanol.
(i) Calcule as massas molares do iso-octano, heptano e etanol?
(ii) Desenhe a frmula estrutural para a molcula de 2,2,4-trimetilpentano.
(iii) Identifcar o grupo funcional da molcula de etanol no combustvel alter-
nativo E-15.
Se a gasolina uma substncia apolar e o etanol uma substncia polar, porque
elas se misturam dando o combustvel E-15, ou melhor, porque elas se solubili-
zam entre si?
Se as coisas, o dinheiro, o mundanismo, se
tornam o centro da vida, perdemos nossa
prpria identidade como homens
Papa Francisco
Programa Nacional Olimpadas de Qumica <<
14
XIX ONNeQ
XIX Olimpada Norte/Nordeste de Qumica
Resolues Escolhidas ONNEQ
QUESTO 1
Beatriz Luberiaga Bezerra, Col. Boa Viagem, Recife - PE
i) 5mg de arsnio: 5 x 10
-3
de arsnio
ii) FeAsS (s) FeS(s) + As (g)
1 mol 1 mol 1mol
Massa de arsnio produzida (m) = 125Kg 67,5Kg = 57,5Kg de As.
iii) 2 As(g) + 3/2 O
2
As
2
O
3
(g)
iv) A densidade depende do volume arranjado dos tomos. O arsnio cinzento
tem densidade maior, por que possui ligaes mais compactas, seu volume
menor. J o arsnio amarelo tem densidade menor devido as ligaes es-
tarem afastadas. Portanto os dois compostos possuem densidades diferentes
porque o arranjo dos tomos com suas ligaes so distintas, com volumes
distintos, e consequentemente densidades diferentes.
v) % As = (74,9 g/mol)/(162,8 g/mol) x 100% = 46.007% As
>> Olimpada Brasileira de Qumica - 2013
15
Solues Escolhidas
QUESTO 2
Geisa Costa Oliveira de M Santana, Farias Brito, Fortaleza - CE
Item i
Item ii
Quando 100 mL de NaOH so adicionados, .
Logo, na primeira adio de NaOH, 110
-3
mol do HCl so neutralizados (propor-
o 1:1). O valor do pH, porm, de 1,6, indicando que ainda h H
+
na soluo.
[H
+
]=10
pH
= 10
-1,6
= 0,025 M
Volume da soluo + 10 mL de NaOH
Considerando que todo H
+
calculado provm da desprotonao do HCl, poss-
vel descobrir o nmero de mols total de HCl.
H
+
neutralizado
Programa Nacional Olimpadas de Qumica <<
16
Sabendo que em uma titulao de cido forte com base forte, a concentrao
atinge o pH=7 no ponto de equivalncia, o volume de NaOH 0,1 mol/L, necess-
rio para neutralizar todo HCl :
Item iii
HCl
(aq)
+ NaOH
(aq)
NaCl
(aq)
+ H
2
O
(l)
Item iv
Nmero de mols de NaOH adicionados:
Se todo cido foi neutralizado, o nmero de mols de cido inicialmente presente
:
1 mol HCl -------------------------------- 1 mol de NaOH (de acordo com o item ii)
2 X 10
-3
mol --------------------------2 X 10
-3
mol
[HCl] =
[HCl] = 0,05 mol/L
QUESTO 3
Nathercia Castro Mota - Master, Fortaleza
(i) Faamos a distribuio eletrnica para o Radnio:
86
Rn = 1s
2
2s
2
2p
6
3s
2
3p
6
4s
2
3d
10
4p
6
5s
2
4d
10
5p
6
6s
2
4f
14
5d
10
6p
6
(o Radnio o radioi-
stopo produzido pelo sexto decaimento da serie de decaimento do U-238 refe-
rido no enunciado).
Para um tomo reagir e formar compostos, ele deve ser capaz de efetuar liga-
es qumicas. Para isso, ele utiliza seus eltrons de valncia, compartilhando-
-os (ligao covalente) ou doando-os (ligao inica) para outros tomos.
Analisemos agora o eltron de Valencia do Radnio: 6p
6
=
XIX ONNeQ
>> Olimpada Brasileira de Qumica - 2013
17
Solues Escolhidas
Note que este orbital p j est completamente ocupado, no estando disponvel
para receber um eltron de um outro tomo, necessitando a ocupao de um
novo orbital (7s), o que no vantajoso em termos de estabilidade. Demais, reti-
rar um eltron desse orbital 6p tambm no vantajoso, essa confgurao j
a mais estvel possvel, visto que todos os eltrons esto emparelhados. Logo, o
Radnio tem uma confgurao estvel, sem precisar efetuar ligaes qumicas
para alcan-la. De fato, o Rn um gs nobre e ostenta 8 eltrons em sua camada
de valncia, conforme sugere a regra do octeto.
Devido ao seu tamanho e ao grande nmero de camadas, o radnio consegue
formar compostos sob condies de presso e temperaturas mais elevadas, ex-
pandindo seu octeto. Outros gases nobres, como o Hlio e o Nenio, contudo,
nem sob condies extremas so capazes de reagir, no sendo conhecidos com-
postos de He nem de Ne.
(ii)

(i) Faamos a distribuio eletrnica para o Radnio:
86
Rn = 1s
2
2s
2
2p
6
3s
2
3p
6
4s
2
3d
10
4p
6
5s
2
4d
10
5p
6
6s
2
4f
14
5d
10
6p
6
(o Radnio o
radioistopo produzido pelo sexto decaimento da serie de decaimento do U-238
referido no enunciado).
Para um tomo reagir e formar compostos, ele deve ser capaz de efetuar
ligaes qumicas. Para isso, ele utiliza seus eltrons de valncia, compartilhando-os
(ligao covalente) ou doando-os (ligao inica) para outros tomos.
Analisemos agora o eltron de Valencia do Radnio: 6p
6
=
Note que este orbital p j est completamente ocupado, no estando
disponvel para receber um eltron de um outro tomo, necessitando a ocupao de
um novo orbital (7s), o que no vantajoso em termos de estabilidade. Demais,
retirar um eltron desse orbital 6p tambm no vantajoso, essa configurao j a
mais estvel possvel, visto que todos os eltrons esto emparelhados. Logo, o
Radnio tem uma configurao estvel, sem precisar efetuar ligaes qumicas para
alcan-la. De fato, o Rn um gs nobre e ostenta 8 eltrons em sua camada de
valncia, conforme sugere a regra do octeto.
Devido ao seu tamanho e ao grande nmero de camadas, o radnio
consegue formar compostos sob condies de presso e temperaturas mais
elevadas, expandindo seu octeto. Outros gases nobres, como o Hlio e o Nenio,
contudo, nem sob condies extremas so capazes de reagir, no sendo
conhecidos compostos de He nem de Ne.
(ii)



(iii) t
1/2
= ln2/K (todo decaimento radioativo segue uma cintica de 1 ordem)
Para o Po
218
:



(i) Faamos a distribuio eletrnica para o Radnio:
86
Rn = 1s
2
2s
2
2p
6
3s
2
3p
6
4s
2
3d
10
4p
6
5s
2
4d
10
5p
6
6s
2
4f
14
5d
10
6p
6
(o Radnio o
radioistopo produzido pelo sexto decaimento da serie de decaimento do U-238
referido no enunciado).
Para um tomo reagir e formar compostos, ele deve ser capaz de efetuar
ligaes qumicas. Para isso, ele utiliza seus eltrons de valncia, compartilhando-os
(ligao covalente) ou doando-os (ligao inica) para outros tomos.
Analisemos agora o eltron de Valencia do Radnio: 6p
6
=
Note que este orbital p j est completamente ocupado, no estando
disponvel para receber um eltron de um outro tomo, necessitando a ocupao de
um novo orbital (7s), o que no vantajoso em termos de estabilidade. Demais,
retirar um eltron desse orbital 6p tambm no vantajoso, essa configurao j a
mais estvel possvel, visto que todos os eltrons esto emparelhados. Logo, o
Radnio tem uma configurao estvel, sem precisar efetuar ligaes qumicas para
alcan-la. De fato, o Rn um gs nobre e ostenta 8 eltrons em sua camada de
valncia, conforme sugere a regra do octeto.
Devido ao seu tamanho e ao grande nmero de camadas, o radnio
consegue formar compostos sob condies de presso e temperaturas mais
elevadas, expandindo seu octeto. Outros gases nobres, como o Hlio e o Nenio,
contudo, nem sob condies extremas so capazes de reagir, no sendo
conhecidos compostos de He nem de Ne.
(ii)



(iii) t
1/2
= ln2/K (todo decaimento radioativo segue uma cintica de 1 ordem)
Para o Po
218
:


(iii) t1/2 = ln2/K (todo decaimento radioativo segue uma cintica de 1 ordem)
Para o Po218 : 3,04 =

(i) Faamos a distribuio eletrnica para o Radnio:
86
Rn = 1s
2
2s
2
2p
6
3s
2
3p
6
4s
2
3d
10
4p
6
5s
2
4d
10
5p
6
6s
2
4f
14
5d
10
6p
6
(o Radnio o
radioistopo produzido pelo sexto decaimento da serie de decaimento do U-238
referido no enunciado).
Para um tomo reagir e formar compostos, ele deve ser capaz de efetuar
ligaes qumicas. Para isso, ele utiliza seus eltrons de valncia, compartilhando-os
(ligao covalente) ou doando-os (ligao inica) para outros tomos.
Analisemos agora o eltron de Valencia do Radnio: 6p
6
=
Note que este orbital p j est completamente ocupado, no estando
disponvel para receber um eltron de um outro tomo, necessitando a ocupao de
um novo orbital (7s), o que no vantajoso em termos de estabilidade. Demais,
retirar um eltron desse orbital 6p tambm no vantajoso, essa configurao j a
mais estvel possvel, visto que todos os eltrons esto emparelhados. Logo, o
Radnio tem uma configurao estvel, sem precisar efetuar ligaes qumicas para
alcan-la. De fato, o Rn um gs nobre e ostenta 8 eltrons em sua camada de
valncia, conforme sugere a regra do octeto.
Devido ao seu tamanho e ao grande nmero de camadas, o radnio
consegue formar compostos sob condies de presso e temperaturas mais
elevadas, expandindo seu octeto. Outros gases nobres, como o Hlio e o Nenio,
contudo, nem sob condies extremas so capazes de reagir, no sendo
conhecidos compostos de He nem de Ne.
(ii)



(iii) t
1/2
= ln2/K (todo decaimento radioativo segue uma cintica de 1 ordem)
Para o Po
218
:




Questo 04
Nmero de sigilo 135
David Silva Almeida Teresina Dom Barreto PI

(i) Como a gua est a uma temperatura menor que a da amostra de cobre, h
passagem de calor (energia) para a gua o que faz a temperatura dela subir.
(ii)
Massa (g)
Temperatura
Inicial (C)
Temperatura Final
(C)
Cobre 50 100 26,3
gua
(calormetro)
100 23,2 26,3

(iii) m x c x
100 x 4,18 x (26,3 23,2)
1295,8 J
(iv) Q
cedido
= Q
recebido

Programa Nacional Olimpadas de Qumica <<
18
XIX ONNeQ
QUESTO 4
David Silva Almeida - Dom Barreto, Teresina - PI
(i) Como a gua est a uma temperatura menor que a da amostra de cobre, h
passagem de calor (energia) para a gua o que faz a temperatura dela subir.
(ii)
Massa (g) Temperatura
Inicial (C)
Temperatura
Final (C)
Cobre 50 100 26,3
gua (calormetro) 100 23,2 26,3
(iii) m x c x
100 x 4,18 x (26,3 23,2)
1295,8 J
(iv) Q
cedido
= Q
recebido
m x c x )
cobre
= (m x c x )
gua
+ (c x )
calormetro
50 x 0,387 x 73,7 = 1295,8 + c x (26,3 23,2)
130,3 = c x 3,1
c = 42 J K
-1
(v) Sim. A variao diferente porque nem todo o calor liberado pelo cobre
direcionado para a gua, ele tambm foi distribudo para o calormetro, para
o ar que entrou em contato com o cobre e at mesmo para o termmetro,
para que fosse feita a leitura da temperatura mdia. Logo, os erros expli-
cam porque houve diferena e, portanto, comprovam a lei de conservao
da energia.
>> Olimpada Brasileira de Qumica - 2013
19
Solues Escolhidas
QUESTO 5
Rodrigo Reis Silva - Col. Sto Antnio de Jesus, Santo Antnio de Jesus - BA
Item 1
Para efetuar o clculo das massas molares necessrio primeiramente descobrir
as frmulas moleculares dos trs compostos. O iso-octano e o heptano so alca-
nos, portanto de frmula molecular geral C
n
H
2n + 2
. Assim:
Iso-octano: C
8
H
2.8 + 2
MM = 8.12 + 18.1
C
8
H
18
MM = 114 g/mol
Heptano: C
7
H
2.7 + 2
MM = 7.12 + 16.1
C
7
H
16
MM = 100 g/mol
O etanol representado por:
CH
3
CH
2
OH C
2
H
6
O MM = 2.12 + 6.1 + 1.16
MM = 46 g/mol
Ento as massas molares do: Iso-octano = 114 g/mol
Heptano = 100 g/mol
Etanol = 46 g/mol
Item 2
2, 2, 4-trimetil-pentano

Ento as massas molares do: Iso-octano = 114 g/mol
Heptano = 100 g/mol
Etanol = 46 g/mol
Item 2
2, 2, 4-trimetil-pentano
C
CH
3
H
3
C
CH
3
CH
2
CH
CH
3
CH
3

Item 3
Grupo funcional do etanol:

H
3
C CH
2
OH

Solubilizao entre etanol e gasolina:
A gasolina constituda de uma mistura de hidrocarbonetos, e, portanto,
totalmente apolar. Como na funo dos hidrocarbonetos s h carbono e hidrognio,
no se nota a presena de polos. Porm, o etanol enquadra-se nos lcoois, que se
caracterizam por apresentar o grupo funcional OH (hidroxila). Em razo deste
grupamento, os lcoois apresentam duas fraes. Uma polar, marcada pela
hidroxila, uma vez que o oxignio altamente eletronegativo; e uma apolar, que
constitui-se apenas de carbono e hidrognio:
CH
3
CH
2
OH

Como possui uma parte polar e uma apolar ele capaz de solubilizar-se em
compostos polares, como a gua (H
2
O), e apolares, como a gasolina.

Hidroxila ligada a
carbono saturado =
funo lcool.
APOLAR POLAR
Programa Nacional Olimpadas de Qumica <<
20
XIX ONNeQ
Item 3
Grupo funcional do etanol:
Ento as massas molares do: Iso-octano = 114 g/mol
Heptano = 100 g/mol
Etanol = 46 g/mol
Item 2
2, 2, 4-trimetil-pentano
C
CH
3
H
3
C
CH
3
CH
2
CH
CH
3
CH
3

Item 3
Grupo funcional do etanol:

H
3
C CH
2
OH

Solubilizao entre etanol e gasolina:
A gasolina constituda de uma mistura de hidrocarbonetos, e, portanto,
totalmente apolar. Como na funo dos hidrocarbonetos s h carbono e hidrognio,
no se nota a presena de polos. Porm, o etanol enquadra-se nos lcoois, que se
caracterizam por apresentar o grupo funcional OH (hidroxila). Em razo deste
grupamento, os lcoois apresentam duas fraes. Uma polar, marcada pela
hidroxila, uma vez que o oxignio altamente eletronegativo; e uma apolar, que
constitui-se apenas de carbono e hidrognio:
CH
3
CH
2
OH

Como possui uma parte polar e uma apolar ele capaz de solubilizar-se em
compostos polares, como a gua (H
2
O), e apolares, como a gasolina.

Hidroxila ligada a
carbono saturado =
funo lcool.
APOLAR POLAR
Solubilizao entre etanol e gasolina:
A gasolina constituda de uma mistura de hidrocarbonetos, e, portanto, to-
talmente apolar. Como na funo dos hidrocarbonetos s h carbono e hidrog-
nio, no se nota a presena de polos. Porm, o etanol enquadra-se nos lcoois,
que se caracterizam por apresentar o grupo funcional OH (hidroxila). Em razo
deste grupamento, os lcoois apresentam duas fraes. Uma polar, marcada
pela hidroxila, uma vez que o oxignio altamente eletronegativo; e uma apo-
lar, que constitui-se apenas de carbono e hidrognio:
Ento as massas molares do: Iso-octano = 114 g/mol
Heptano = 100 g/mol
Etanol = 46 g/mol
Item 2
2, 2, 4-trimetil-pentano
C
CH
3
H
3
C
CH
3
CH
2
CH
CH
3
CH
3

Item 3
Grupo funcional do etanol:

H
3
C CH
2
OH

Solubilizao entre etanol e gasolina:
A gasolina constituda de uma mistura de hidrocarbonetos, e, portanto,
totalmente apolar. Como na funo dos hidrocarbonetos s h carbono e hidrognio,
no se nota a presena de polos. Porm, o etanol enquadra-se nos lcoois, que se
caracterizam por apresentar o grupo funcional OH (hidroxila). Em razo deste
grupamento, os lcoois apresentam duas fraes. Uma polar, marcada pela
hidroxila, uma vez que o oxignio altamente eletronegativo; e uma apolar, que
constitui-se apenas de carbono e hidrognio:
CH
3
CH
2
OH

Como possui uma parte polar e uma apolar ele capaz de solubilizar-se em
compostos polares, como a gua (H
2
O), e apolares, como a gasolina.

Hidroxila ligada a
carbono saturado =
funo lcool.
APOLAR POLAR
Como possui uma parte polar e uma apolar ele capaz de solubilizar-se em com-
postos polares, como a gua (H
2
O), e apolares, como a gasolina.
Ns somos aquilo que fazemos
repetidamente. Excelncia, ento no um
modo de agir, mas um hbito.
Aristteles
>> Olimpada Brasileira de Qumica - 2013
21
Resultados
XIX OLIMPADA NORTE/NORDESTE
de QUMICA 2013
O U R O
Nome Cidade Escola UF Nota Escore
Joaquim Ivo Vasques Dantas Landim Fortaleza Farias Brito CE 99 100,0
Luckeciano Carvalho Melo Teresina Lettera PI 99 100,0
Artur Souto Martins Fortaleza Ari de S CE 98 99,00
Gabriel Adriano de Melo Palmas Objetivo TO 98 99,00
Joel Gustavo Pinto Oliveira Belm IFPA PA 98 99,00
Lucas de Melo Jurema Guimaraes Joo Pessoa Motiva PB 98 99,00
Lucas Rebelo Vieira da Silva Recife GGE PE 98 99,00
Luiz de Paulo Santana Salvador Inst. Fed. da Bahia BA 98 99,00
Pedro Jorge Luz A. Cronemberger Teresina Sag. Cora. de Jesus PI 98 99,00
Rodrigo Reis Silva Santo Antonio de Jesus S. Antonio de Jesus BA 98 99,00
Nilo Parente Pessoa Dias Fortaleza Farias Brito CE 94,2 95,16
Nilo Parente Pessoa Dias Fortaleza Farias Brito CE 94,2 95,16
P R A T A
Nome Cidade Escola UF Nota Escore
Victor Machado Teresina Integral PI 97 97,99
Leonardo Lacet Cordeiro Recife Marista So Lus PE 95,5 96,47
Gabriel Jos Moreira da Costa Silva Macei Contato AL 95,4 96,37
Iana Soares Negreiros Fortaleza Farias Brito CE 95,4 96,37
Lia de Oliveira Domingues Fortaleza Farias Brito CE 95,4 96,37
Yasmin Thaise Lisba da Veiga Belm IFPA PA 95,4 96,37
Manoel Maurcio Neves Gumes Salvador Integral BA 95 95,97
Victor Hugo Fernandes Breder Teresina Dom Barreto PI 95 95,97
Lucas Moura Andrade Teresina Lavoisier PI 94,4 95,36
Nilo Parente Pessoa Dias Fortaleza Farias Brito CE 94,2 95,16
Lucas Bastos Oliveira Fortaleza Dulia Bringel CE 93,4 94,35
Lucas Levy Alves de Moraes Teresina Sag. Cor. de Jesus PI 93,3 94,25
Christian Do Lago F. Bezerra de Melo Manaus La Salle AM 93 93,95
Joo Lucas Fernandes dos Santos Recife GGE PE 93 93,95
Geisa Costa Oliveira de M Santana Fortaleza Farias Brito CE 92,8 93,75
Daniel Meira Nbrega de Lima Joo Pessoa GEO Tamba PB 92,4 93,34
Livia Rodrigues de Araujo Fortaleza Farias Brito CE 92,4 93,34
Maurocelio Rocha Pontes Filho Fortaleza Farias Brito CE 92,4 93,34
Programa Nacional Olimpadas de Qumica <<
22
XIXONNeQ
Nathercia Castro Mota Fortaleza Master CE 92,4 93,34
Nicholas de Souza Costa Lima Fortaleza Ari de S CE 92,4 93,34
Pedro Paulo Barreto Guimares Salvador Sartre Coc - Nobel BA 92,4 93,34
Sebastio Beethoven Brando Filho Teresina Dom Barreto PI 92,4 93,34
B R O N Z E
Nome Cidade Escola UF Nota Escore
Alexandre Hamrich Macei SEB COC AL 92 92,94
Isabela Martins Costa Teresina Dom Barreto PI 92 92,94
Joana Loureno de carvalho Joo Pessoa Motiva PB 92 92,94
Thiago Matheus Santos Rios Salvador IFBA BA 92 92,94
Alex Alcino de Oliveira Andr Recife Boa Viagem PE 91,8 92,74
Alan Motta Ganem Natal Henrique Castriciano RN 91,5 92,43
Jlia Cristina Soares de Sousa Teresina Dom Barreto PI 91,2 92,13
Marcelo dos Santos Bandeira Filho Teresina Dom Barreto PI 91 91,93
Tayna Milfont S Fortaleza Farias Brito CE 91 91,93
Carlos Henrique Mesquita Peres Manaus Lato Sensu AM 90,5 91,42
Eder Leandro da Silva Dantas Natal Overdose e Curso RN 90,4 91,32
Gabriel Castro Tavares Manaus Militar AM 90,4 91,32
Jos Rodolfo de Farias Neto Macei Contato AL 90,4 91,32
Jorge Luis Ferreira Alencar Dom Pedro AEPRON MA 90 90,92
Joo Luiz Vieira Maciel Borges Belm Ideal Militar PA 89,9 90,82
Arthur Furtado Santos Recife Motivo PE 89,5 90,41
Daniel Imbassahy de S B. C. e Silva Salvador Integral BA 89,4 90,31
Diego Henrique de Oliveira Sales Recife Sagrado Corao PE 89,4 90,31
Jonatas Melo de Santana Valenca CEFET - Bahia BA 89 89,91
Murilo Leo Pereira Belm Ideal Militar PA 89 89,91
Roberto Hangley Mendes Dantas Jnior Sousa Ao e Curso PB 88,9 89,81
Eduardo Serpa Fortaleza Master CE 88,8 89,71
Fbio Gabriel Costa Nunes Teresina Dom Barreto PI 88,8 89,71
Bernado Jos Pinto de Mello e Silva Filho So Lus Santa Teresa MA 88,6 89,50
Camila Mendes Pontes Caruaru Diocesano PE 88,5 89,40
Gustavo Henrique dos Santos Aracaju COESI SE 88,4 89,30
Maria Luiza Costa Nascimento Natal Cei - Romualdo Galvo RN 88,4 89,30
Paula Araujo Gomes Joo Pessoa Motiva PB 88,4 89,30
Yago Rodrigues Incio Recife Santa Maria PE 88,4 89,30
A lista de alunos agraciados com MENO HONROSA e DEMAIS CLASSIFICADOS
na XIX Olimpada Norte/Nordeste de Qumica encontra-se disponvel em www.obquimica.org
>> Olimpada Brasileira de Qumica - 2013
23
VI OBQ Jr Fase I
VI OLIMPADA BRASILEIRA DE QUMICA JNIOR
8 e 9 Sries do Ensino Fundamental
Fase I
I NS T R UE S
1. A prova consta de 20 questes objetivas, cada uma contendo quatro alternativas, das quais apenas uma
deve ser assinalada.
2. A prova tem durao de 3 horas.
3. Voc receber o gabarito aps 1 hora do incio da prova, para registrar as suas opes de respostas.
5. Voc receber a Folha de Respostas aps 1 hora do incio da prova, para registrar as suas opes.
QUESTES DE MLTIPLA ESCOLHA
01 As cartas de um jogo apresentavam um formato
no qual a frente de cada uma delas correspon-
dia imagem de uma clula da tabela peridica,
conforme mostrado ao lado. A sequncia (I, II,
III e IV) que indica corretamente as principais
caractersticas para a identifcao de um ele-
mento qumico desenhado em uma dessas car-
tas dada pela seguinte a ordem:
A) I Massa atmica; II Smbolo; III Distribuio eletrnica; IV Nmero
atmico.
B) I Massa atmica; II Smbolo; III Quantidade de nutrons; IV Nmero de
prtons.
C) I Nmero atmico; II Sigla; III Quantidade de nutrons; IV Nmero de
prtons.
D) I Nmero atmico; II Smbolo; III Distribuio eletrnica; IV Massa
atmica.
Programa Nacional Olimpadas de Qumica <<
24
VI OBQ Jr Fase I
02 Uma palavra cruzada do tipo Diretas
apresentada abaixo. Preencha esse dia-
grama.

Os enigmas contidos nas clulas indicadas
por I e II correspondem respectivamente a
A) I - Ferro, II CaO B) I - Ferro, II - NaO
C) I Trio, II CaO D) I Trio, II - NaO
03 Um instrumento de medio utiliza uma substncia simples, de alta densida-
de e que possui uma propriedade termomtrica (uma propriedade que varia
com a temperatura). Qual entre as substncias apresentadas abaixo atende a
essas caractersticas?
A) Gs carbnico B) Gs hlio C) Hg D) O2
04 As idades de alguns materiais podem ser determinadas pela utilizao de
uma tcnica conhecida como datao por carbono-14. O carbono-14 um
istopo radioativo natural do elemento qumico carbono (Z=6) e possui
A) 8 prtons no seu ncleo.
B) nmero atmico igual a 14.
C) nmero de massa igual a 14.
D) 2 nutrons a menos no seu ncleo que o istopo estvel carbono-12.
05 Qual das alternativas abaixo exemplifca uma atividade associada qumica
verde?
A) Sntese de clorofla utilizando metais pesados como reagentes.
B) Explorao das reservas naturais de ltio para produo de baterias.
>> Olimpada Brasileira de Qumica - 2013
25
VI OBQ Jr Fase I
C) Utilizao de combustveis fsseis para reciclagem de metais.
D) Substituio do petrleo por biomassa, como fonte natural para obteno de
combustveis.
06 Uma etapa utilizada tanto do processo de separao do etanol do mosto fer-
mentado quanto no processo de separao da gasolina do petrleo bruto a
A) destilao simples. B) destilao fracionada.
C) fermentao. D) liquefao.
07 Analise a imagem mostrada abaixo.

Adaptadodehttps://www.google.com.br/search?hl=pt-
-BR&gs_rn=15&gs_ri=psy-
Esse experimento ilustra uma
A) calcinao. B) fuso.
C) liquefao. D) sublimao.
08 Determinada substncia lquida considerada uma fonte da sua forma gasosa
despressurizada. Por causa da sua capacidade de manter a temperatura bem
abaixo do ponto de congelamento da gua, ela tem sido muito utilizada na
refrigerao e preservao de materiais e amostras biolgicas, como de san-
gue e de clulas reprodutivas.
Qual a frmula molecular dessa substncia?
A) CO
2
B) H
2
C) O
2
D) N
2
09 Verifca-se que em um tipo de solda, uma liga metlica formada por 63% de
estanho e 37% de chumbo, mantm-se constante na temperatura no ponto
Programa Nacional Olimpadas de Qumica <<
26
de fuso, desde o incio at a mudana de estado. No entanto, a sua tempera-
tura de ebulio varia.
Essa situao indica um exemplo de uma:
A) mistura euttica. B) mistura azeotrpica.
C) substncia composta. D) substncia simples.
10 Membranas porosas vm sendo usados na remoo de enzimas que causam a
degradao da gua de coco. As seletividades das mesmas so determinadas
a partir da relao entre o tamanho e a forma dos solutos a serem fraciona-
dos e a distribuio de tamanho dos poros existentes nas superfcies dessas
membranas.
AdaptadodeMAGALHAES,MrciaPimenteletal.Conservaodeguadecocoverdeporfltrao
commembrana.Cinc.Tecnol.Aliment.,Campinas,v.25,n.1,Mar.2005
O processo de separao citado no texto se refere
A) centrifugao. B) decantao.
C) fltrao. D) fotao.
11 A confeco de alguns artefatos requer, como matria-prima, determinados
tipos de elementos qumicos que so raros na crosta terrestre. Eles apresen-
tam altas densidades, so maleveis e dcteis. Um desses elementos rara-
mente se combina com outros elementos qumicos, por isso, ele geralmente
encontrado na natureza no estado nativo.
Qual dos objetos listados abaixo exemplifca um artefato produzido com o ele-
mento qumico enfatizado no texto acima?
A) Anel de ouro B) Esfera de ferro
C) Faca de prata D) Grade de alumnio
VI OBQ Jr Fase I
>> Olimpada Brasileira de Qumica - 2013
27
12 A charge abaixo foi produzida em um
momento em que houve denncias en-
volvendo a adulterao do leite.
De acordo com o contexto da charge, o leite
estaria contaminado com uma soluo
de
A) cloreto de sdio, NaCl B) glicose.
C) soda custica, NaOH. D) sacarose
13 A fgura abaixo ilustra a evoluo histrica de propostas de modelos atmicos.

Quais desses modelos propem a existncia de eltrons em camadas externas
ao ncleo?
A) Dalton e Thomson B) Rutherford e Bohr
C) Rutherford e Dalton D) Bohr e Thomson
14 Leia o dilogo abaixo.
- Cad o objeto que eu comprei para fazer uma fltrao a vcuo no laboratrio
da escola? Ele de vidro e possui uma sada lateral prxima abertura.
- Eita! E agora? Foi nele que coloquei as fores...
De acordo com as informaes contidas no texto, a vidraria em questo se trata
de um
A) bquer. B) erlenmeyer.
C) funil de separao. D) kitassato
VI OBQ Jr Fase I
Programa Nacional Olimpadas de Qumica <<
28
15 A amnia (NH
3
) foi citada como possvel responsvel pelo acidente ocorrido
em uma fbrica de fertilizante, no Texas, EUA. Um dos gases mais solveis
em gua, a amnia usada para a produo de nitrato de amnio (NH
4
NO
3
).
NH
3
no explosiva, mas pode formar uma mistura explosiva com o ar a
certas concentraes (16% a 25% em volume no ar). Do mesmo modo, pode
formar uma mistura explosiva em contato com outras substncias, como o
for (F
2
) e o mercrio (Hg).
Adaptado de http://g1.globo.com/ciencia-e-saude/noticia/2013/04/entenda-os-perigos-ligados-
-ao-fertilizante-nitrato-de-amonio.html
A substncia inica citada no texto
A) o fertilizante produzido.
B) a que solubiliza a amnia.
C) o composto responsvel pelo acidente na fbrica.
D) uma das substncias que pode formar uma mistura explosiva com a amnia.
16 A classifcao dos elementos qumicos considerada uma das mais valiosas
generalizaes cientfcas. Em relao a esse aspecto, reunindo dados pr-
prios e de outros cientistas, Dmitri Mendeleiev concluiu que certas proprie-
dades dos elementos qumicos so uma funo peridica de seus
A) pontos de fuso. B) pesos atmicos.
C) nmeros de massa. D)estados de oxidao.
17 Estudantes de uma escola realizaram uma atividade no laboratrio de uma
universidade. Ao analisarem uma determinada amostra, a olho nu, eles ve-
rifcaram que esse sistema se apresentava uniforme e com caractersticas
iguais em todos os seus pontos. Depois, analisando-o com um microscpio,
constataram que ele no se apresentava uniforme e nem mantinha caracte-
rsticas iguais em todos os seus pontos.
Esse tipo de experimento permite verifcar que o critrio para classifcao de
um sistema em homogneo e heterogneo
VI OBQ Jr Fase I
>> Olimpada Brasileira de Qumica - 2013
29
A) independe do uso de aparelhos.
B) deve ser feita sem o uso de instrumentos.
C) deve ser realizada tanto a olho nu quanto com aparelhos.
D) relativo, pois depende da aparelhagem disponvel para as observaes.
18 Uma pessoa resolveu limpar as suas moedas de R$ 0,05, que estavam cheias
de manchas. Para isso, colocou-as em um recipiente de plstico ao qual
acrescentou vinagre e, depois, transferiu um pouco de sal de cozinha. Passa-
dos alguns minutos, as superfcies das moedas fcaram brilhantes.
No incio desse procedimento, quem representa um exemplo de uma mistura
homognea?
A) A moeda
B) O vinagre
C) O sistema moeda-vinagre
D) O sistema moeda-vinagre-sal de cozinha
19 Um dos princpios da qumica verde considera que os produtos qumicos
precisam ser projetados para a biocompatibilidade. Nesse sentido, aps sua
utilizao, uma substncia lanada no meio ambiente
A) no deve se degradar.
B) deve se degradar em substncias orgnicas.
C) deve se degradar em substncias inorgnicas.
D) no deve permanecer no ambiente, degradando-se em produtos incuos.
20 A partir da segunda metade do sculo XIX, a qumica teve um grande desen-
volvimento e permitiu a sntese de muitas substncias constitudas por ca-
deias carbnicas. Conhecida como a qumica do carbono, a rea da qumica
relacionada tradicionalmente sntese desses tipos de compostos denomi-
nada de:
A) eletroqumica B) qumica orgnica
C) qumica inorgnica D) fsico-qumica.

VI OBQ Jr Fase I
Programa Nacional Olimpadas de Qumica <<
30
VI OLIMPADA BRASILEIRA DE QUMICA JNIOR
8 e 9 anos do Ensino Fundamental
Fase II
I NS T R UE S
1. A prova consta de 10 (dez) questes do tipo mltipla escolha (mximo 40 pontos) e 3 (trs) questes
analtico-expositivas (mximo 60 pontos).
2. Para responder as questes de mltipla escolha, identifque APENAS UMA NICA alternativa correta e
marque a letra correspondente no gabarito existente na Folha de Respostas.
3. Para responder as questes analtico-expositivas, utilize APENAS o espao destinado para cada uma das
trs questes na Folha de Respostas.
4. A prova tem durao de 3 horas.
5. Voc receber a Folha de Respostas aps 1 hora do incio da prova, para registrar as suas opes.
QUESTES DE MLTIPLA ESCOLHA
01 O mineral constituinte dos dentes dos tubares contm um elemento qu-
mico que tambm est presente em cremes dentais e enxaguatrios bucais
destinados a proteger os dentes dos humanos. Esse elemento contribui para
que o esmalte do dente fque mais mineralizado e ajuda a explicar por que
os tubares so to efcientes para despedaar e cortar suas presas, alm de
nunca apresentarem cries.
Adaptado de http://animalplanet.discoverybrasil.uol.com.br/dentes-de-tubarao-contem-fuor/
O elemento qumico em questo o
A) Au. B) F. C) He. D) Hg.
02 Uma verso eletrnica para a tabela peridica traz diferentes informaes
sobre um determinado elemento qumico. Conforme mostrado abaixo, ela
possibilita a obteno de dados sobre: o nmero atmico, a massa atmica, a
confgurao eletrnica, uma fonte natural e aplicaes.
VI OBQ Jr Fase II
>> Olimpada Brasileira de Qumica - 2013
31
VI OBQ Jr Fase II
Fonte: http://abiquim.org.br/tabelaperiodica/tabela_est.asp
(Baseado em material do Prof. Dr. Srgio Melo para as Olimpadas de Qumica)
A partir das informaes apresentadas, CORRETO afrmar que:
A) apenas o O e o Br ocorrem na natureza.
B) todos so de um mesmo perodo da tabela peridica.
C) o K e o Br formam um composto inico.
D) dois deles so de um mesmo grupo da tabela peridica
03 As medies realizadas na atmosfera de Marte pelo rob Curiosity no en-
contraram vestgios de um determinado gs, que considerado um poten-
cial indcio de vida na Terra. Essa substncia composta gasosa a 25
o
C e
possui molculas formadas apenas por ligaes covalentes. Em nosso plane-
ta, ela produzida por uma variedade de seres, de micrbios a animais de
grande porte.
Adaptado de http://noticias.uol.com.br/ciencia/album/2013/01/10/imagens-e-noticias-sobre-o-espaco-2013.htm?abrefoto=240
A substncia investigada pelo rob na atmosfera de Marte era o
A) cloreto de sdio (NaCl). B) etanol (CH
3
CH
2
OH).
C) metano (CH
4
). D) oxignio (O
2
).
O texto mostrado abaixo ser utilizado nas questes 04, 05 e 06.

O polnio (Z=84; [Xe]4f
14
5d
10
6s
2
6p
4
; T
fus.
= 254 C e T
ebul.
= 962 C) apresenta sete
Programa Nacional Olimpadas de Qumica <<
32
VI OBQ Jr Fase II
istopos naturais e foi o terceiro elemento radio-
ativo identifcado. Nos vegetais, como no fumo,
ou tabaco (nome dado a espcies do gnero Ni-
cotiana), o polnio pode provir do ar ou ser ab-
sorvido pelas razes. Na dcada de 1960, alguns
pesquisadores desenvolveram uma tcnica para
medir nveis muito baixos de rdio e polnio. Ao
testarem as cinzas de cigarro com essa tcnica,
eles tiveram uma surpresa: no encontraram si-
nais de polnio. Nenhum outro material orgni-
co, incluindo as plantas, havia dado um resultado negativo para o polnio na
presena do rdio. Mas na temperatura do tabaco em brasa, o polnio se vapo-
riza rapidamente. Ento, percebeu-se que o polnio procurado deveria estar na
fumaa do cigarro! E isso signifcava que os fumantes o inalariam diretamente
para os pulmes. Apesar de o polnio no ser o principal carcingeno (agente
causador de cncer) do cigarro, ele pode causar milhares de mortes ao ano,
pois, os istopos de polnio-210 (
210
Po) da fumaa do cigarro se concentram nos
pontos de bifurcao dos pulmes, formando focos de radioatividade.
Adaptado de http://www2.uol.com.br/sciam/reportagens/fumaca_radioativa.html
Crdito da imagem: http://comofazeronline.blogspot.com.br/2011/03/substancias-toxicas-do-cigarro.html
04 A nfase central do texto est na
A) ao de um istopo radioativo presente na fumaa do cigarro.
B) atividade carcingena de uma substncia orgnica do cigarro.
C) constatao de que o polnio presente no fumo no agente causador de
cncer.
D) emisso radioativa nos pulmes, causada por um gs nobre da fumaa do
cigarro.
05 Os tomos desse carcingeno
A) possuem 210 prtons.
B) so de um elemento qumico do 6 perodo da tabela peridica.
C) apresentam 84 nutrons.
D) constituem uma substncia simples, que lquida, temperatura ambiente.
>> Olimpada Brasileira de Qumica - 2013
33
VI OBQ Jr Fase II
06 A partir das informaes contidas no texto, CORRETO afrmar que a pre-
sena desse carcingeno nos pulmes dos fumantes potencializada por um
processo associado
A) produo de polnio-210 no estado gasoso.
B) destilao de polnio-210 no tabaco em brasa.
C) decantao dos tomos de polnio nas cinzas do cigarro.
D) formao de vapor de uma substncia orgnica radioativa.
07 Um vdeo postado na Internet mostra um jeito bem legal, baseado em uma
tcnica usada para investigar crimes, para se identifcar impresses digitais.
Nesse vdeo, inicialmente, uma pessoa aperta a ponta do
dedo polegar contra um pedao de papel. Depois, o papel
preso em um pregador de roupas e transferido para den-
tro de um erlenmeyer. O interior da vidraria contm um
vapor de colorao prpura, formado com o aquecimento
de alguns cristais de I
2
. Ao ter contato com esse vapor, a
digital, a marca deixada pela ponta do dedo aparece, ou
seja, revelada. A explicao: as molculas presentes no vapor se ligam s
molculas de gordura, liberadas pelas pontas dos dedos, produzindo subs-
tncias coloridas, que podem ser visualizadas facilmente.
Adaptado de http://www.manualdomundo.com.br/2013/07/como-tirar-impressao-digital-em-casa/
A revelao da impresso digital apresentada nesse vdeo envolve um exemplo de:
A) um processo baseado em fenmenos qumicos, apenas.
B) um processo baseado em fenmenos biolgicos, apenas.
C) ebulio de um elemento qumico, existente na forma de
cristais.
D) sublimao de uma substncia, que possui ligao covalen-
te em sua molcula.
Programa Nacional Olimpadas de Qumica <<
34
08 Uma das etapas da fabricao de um perfume consiste na diluio de uma es-
sncia em uma mistura de etanol e gua. Esse processo exige o cuidado com
a aparncia fnal do lquido. Na preparao de alguns tipos de perfumes essa
ateno deve ser ainda maior, pois, aps a diluio da essncia, possvel se
observar a formao acentuada de partculas na mistura.
Com base nas informaes apresentadas, indique dois processos de separao
que devem ser usados na fabricao de perfumes para dar qualidade visual ao
esse tipo de produto.
A) Centrifugao e destilao simples
B) Centrifugao e destilao fracionada
C) Decantao e destilao fracionada
D) Decantao e fltrao
09 A tabela mostrada abaixo contm a informao nutricional impressa no rtulo
de um produto alimentcio, analise-a.

Fonte: Consultoria Equilbrio Nutricional
Entre os produtos apresentados abaixo, qual deles se adqua s informaes for-
necidas nesse rtulo?
A) Iogurte B) Isotnico C) gua mineral D) Requeijo
VI OBQ Jr Fase II
>> Olimpada Brasileira de Qumica - 2013
35
VI OBQ Jr Fase II
10 Em um laboratrio de uma escola, um determinado volume de gua foi trans-
ferido para um bquer. Depois, um mesmo volume de clorofrmio (CHCl
3
) foi
transferido para outro bquer. Em seguida, 10 (dez) pedaos de cenoura foram
adicionados a cada um dos bqueres. Passados alguns minutos, observou-se
que apenas o lquido de um dos recipientes fcou com uma colorao alaran-
jada. Ento, cada um dos lquidos foi fltrado e transferidos para um funil de
separao. Aps a agitao vigorosa do funil, o sistema fcou em repouso e foi
verifcada a formao de duas fases. Em uma delas, um dos lquidos permane-
ceu translcido e colorido; na outra, o lquido permaneceu incolor.
Considerando que, na temperatura da realizao do experimento, a densidade do
clorofrmio igual a 1,48 g/mL e a da gua igual a 1,0 g/mL, CORRETO afrmar
que, nesse sistema bifsico:
A) a fase colorida a mais densa.
B) a gua fca na parte inferior do funil.
C) a fase menos densa uma mistura heterognea, a olho nu.
D) a fase colorida do sistema uma soluo de gua e de corantes da cenoura, apenas.
QUESTES ANALTICO-EXPOSITIVAS

11 Um esquema de roubo de cabos de uma empresa de telefonia foi descober-
to e os envolvidos na ao criminosa acabaram presos. Em um galpo, os
cabos eram descascados e os fos de cobre eram derretidos. De acordo com
a Polcia Militar, a venda rendia mais de R$ 200.000,00 por semana, pois, o
quilograma de cobre era negociado por R$ 14,00 (catorze reais).
Adaptado de http://noticias.r7.com/minas-gerais/esquema-de-roubo-de-fos-de-cobre-da-oi-e-descoberto-e-cinco-presos-em-
-bh-09082013
Em mdia, qual quantidade de matria (em mols) de cobre metlico era comer-
cializada semanalmente pelos ladres?
Dados: a massa atmica do cobre igual a 63,5 u; n = m / M.
Programa Nacional Olimpadas de Qumica <<
36
12 Considerando as propostas contidas nos modelos atmicos de Bohr, de Dal-
ton e de Thomson, verifca-se que apenas um deles traz fundamentos para
explicar as cores observadas na exploso dos fogos de artifcios.
Indique qual esse modelo atmico e, com base no mesmo, proponha uma ex-
plicao para a observao desse tipo de fenmeno.
13 A gua e o etanol formam uma mistura azeotrpica. Tal caracterstica dif-
culta a separao dessas substncias por destilao. A curva de aquecimento
representativa para esse tipo de mistura est em um dos grfcos mostrados
abaixo. Nos demais, h uma curva de aquecimento representativa para uma
substncia simples e outra para uma mistura comum. Analise-as.

Grfco 1 Grfco 2 Grfco 3
Justifcando a sua escolha, indique qual grfco se relaciona a uma curva de
aquecimento representativa de uma mistura azeotrpica.
VI OBQ Jr Fase II
>> Olimpada Brasileira de Qumica - 2013
37
Solues Escolhidas
Questo 11
Aluno: Leonardo Shigueo Uchiyama Fonseca, 9 ano, Col. Santa Isabel, Santa Isabel - SP.
Nesta questo precisaremos saber a quantidade mdia de matria, em mols,
comercializada pelos ladres semanalmente. Para isso, sabemos que os ladres
recebiam 200 mil reais semanalmente com a venda do cobre, vendendo a 14
reais o quilo do cobre e sabemos tambm, atravs do enunciado da questo, que
1 mol de cobre igual a 63,5 g/mol.
Primeiro, devemos descobrir a massa total de cobre comercializada por eles,
semanalmente. Como eles faturavam 200 mil reais por semana vendendo o co-
bre a um valor de 14 reais o quilo, conclumos que eles comercializavam aproxi-
madamente 14.286 kg de cobre, que igual a 14.286.000 gramas de cobre.
Agora vamos calcular esse valor em mols. Para isso usamos a frmula
(dados do enunciado da questo). Onde: n = valor em mols; m = massa de cobre
vendida semanalmente em gramas; e M = massa atmica do cobre.
Substituindo os valores, temos que:
Efetuando a conta, chegamos ao resultado que os ladres comercializavam,
aproximadamente 224.976 mols de cobre semanalmente.
Questo 12
Aluno: Letcia Marin Romero, 9 ano, Col. London, So Jos do Rio Preto - SP
O nico modelo atmico que traz fundamentos para explicar as cores ob-
servadas na exploso dos fogos de artifcios o modelo atmico de Rutherford-
Bohr. O modelo de estado de excitao dos eltrons. De acordo com esse modelo
Solues Escolhidas
Programa Nacional Olimpadas de Qumica <<
38
VI OBQ Jr
quando em um tomo aplicamos determinada quantidade de energia trmica
(calor) o eltron se v obrigado a pular para uma camada mais externa, por-
tanto, mais energtica. Porm, esse estado de excitao do eltron passageiro
e aps um determinado tempo ele retorna sua camada original. Durante esse
processo de retorno ele libera energia luminosa (em forma de luz). Essa luz
varia sua cor e tonalidade de acordo com o elemento em questo. No caso dos
fogos de artifcios, ao se colocar fogo, pela combusto da plvora, nos sais de
elementos qumicos o eltron que recebeu energia trmica vai para uma camada
mais energtica e no seu retorno emite diferentes cores, conforme a diferena
de energia entre as rbitas. Assim, conclumos o porqu das cores observadas
nos fogos de artifcios, como pode ser observado no esquema abaixo:
Questo 13
Aluno: Tiago de Sousa Viana, 8 ano, Col. Master, Fortaleza - CE
Analisemos separadamente cada um dos grfcos:
Observando o grfco 1 visvel a presena de dois patamares, um durante
a fuso do material em questo e outro na ebulio do mesmo. Assim sendo,
conclui-se que o material transforma-se da fase slida para a fase lquida a tem-
peratura constante, o mesmo ocorrendo na passagem da fase lquida para a ga-
sosa. Em razo disso, o grfco 1 representa a curva de aquecimento de uma
substncia simples.
Observando o grfco 2 visvel a presena de um patamar durante a ebu-
lio do material em questo. Assim sendo, conclui-se que o material se trans-
forma da fase slida para a fase lquida a temperatura varivel, enquanto que
o mesmo muda da fase lquida para a gasosa em temperatura constante. Em
razo disso, o grfco 2 representa a curva de aquecimento de uma mistura aze-
otrpica.
>> Olimpada Brasileira de Qumica - 2013
39
Solues Escolhidas
Observando o grfco 3 visvel a ausncia de patamares. Assim a passagem
do material em questo da fase slida para a fase lquida e da lquida para a
gasosa ocorrem a uma temperatura varivel, o mesmo ocorrendo na passagem
da fase lquida para a gasosa. Em razo disso, o grfco 3 representa a curva de
aquecimento de uma mistura comum.
Pela anlise, conclui-se que o grfco que se relaciona com uma curva de
aquecimento representativa de uma mistura azeotrpica o grfco 2, uma vez
que apresenta apenas um patamar que ocorre durante a ebulio da mistura, ou
seja, a ebulio ocorre a uma temperatura constante, o mesmo no ocorrendo
durante a fuso da mistura.
A nica maneira de fazer um excelnte
trabalho amar o que voc faz.
Steve Jobs
Programa Nacional Olimpadas de Qumica <<
40
VI OLIMPADA BRASILEIRA DE QUMICA JNIOR
RESULTADO
Aluno UF Cidade UF ANO
NOTA
O U R O
Aleson Sstenes Valentim CE Caucaia Cenecista Luzardo Viana 9 100
Joo Alberto Moreira Serdio DF Braslia Militar de Braslia 8 100
Arthur Gabriel Moura Vieira MG Juiz de Fora Col. Militar 9 96
Gabriel Belizrio Amorim Silva ES Vitria Charles Darwin 9 96
Gabriel Moura Brana CE Fortaleza Ari de S Cavalcante 9 96
Lucas Salume Lima Nogueira ES Vitria C . E. Leonardo da Vinci 9 96
Rubens Henrique Ferreira dos Reis PI Teresina Madre Maria Villac 9 96

P R A T A
Ana Beatriz Rablo Evangelista PI Teresina Dom Barreto 9 94
Bruno de Macedo Grangeiro PB Joo Pessoa Motiva 9 94
Gerardo Albino Nogueira Filho CE Fortaleza Ari de S Cavalcante 9 94
Gustavo Balvedi Pimentel PR Curitiba Bom Jesus gua Verde 9 93
Victor Shinit Enari SP Pindamonhangaba Emlio Ribas 9 93
Amanda Camelo Paulino CE Fortaleza Ari de S Cavalcante 9 92
Claudio Jos F. de S F.Vasconcelos RJ Rio de Janeiro Militar 9 92
Davi Oliveira Arago CE Fortaleza Militar de Fortaleza 9 92
Francisco Luiz Isael Junyor CE Fortaleza Farias Brito 9 92
talo Rennan Lima Silva CE Fortaleza Sete de Setembro 9 92
Lucas Pontes Coutinho CE Fortaleza Santa Ceclia 9 92
Matheus Cardoso Arago CE Fortaleza Ari de S Cavalcante 9 92
Rogrio Aristida Guimares Jnior PI Teresina Dom Barreto 9 92
Ronaldo Viana Alencar Jnior CE Sobral Farias Brito Sobralense 9 92
Vitor Gomes Pires SP So Paulo Campos Salles 9 92
Gabriel de O. Mendona Soares PE Recife Col. Aplicao do Recife 9 91
Marcos Henrique Bandeira Ferreira PI Teresina Santa Maria Goretti 9 91
Otvio Henrique Ribas Guimares MG Belo Horizonte Bernoulli 9 91
Tiago Mansur Kobbaz SP Pindamonhangaba Emlio Ribas 9 91
Gabriel Arthur Teixeira Rodrigues CE Fortaleza Farias Brito 9 90
VI OBQ Jr
>> Olimpada Brasileira de Qumica - 2013
41
Honrio Ribeiro dos Santos Neto PI Teresina Santa Maria Goretti 9 90
Dione da Conceio Nunes MA Tutia Inst. Educ. Maria Madalena 9 89,5
Sandra Ayumi Nihama SP So Paulo Etapa 9 89
Arthur Rocha de Freitas DF Braslia Militar de Braslia 8 88
Danilo Vieira Gomes CE Juazeiro do Norte Paraiso 9 88
Dayanne Rolim Carvalho CE Juazeiro do Norte Modulo /Objetivo 9 88
Felipe Reyel Feitosa de Sousa PI Teresina Dom Barreto 9 88
Gabriel Fernandes Gonalves MG Juiz de Fora Col. Militar 9 88
Gustavo Neponucena de Farias DF Taguatinga Leonardo da Vinci 9 88
Joo Amncio Matos Oliveira Gama SE Aracaju Amadeus 9 88
Joo Guilherme Madeira Arajo CE Fortaleza Farias Brito 8 88
Lyza Hlen Silva Costa PI Teresina Santa Maria Goretti 9 88
Mateus Mouro Silva MG Belo Horizonte Coleguium 9 88
Rafael Juc Pinheiro CE Fortaleza 7 de Setembro 8 88
Samuel Henrique F. Pereira MG Juiz de Fora Col. Militar 9 88
B R O N Z E
Antnio Thiago Dias Arruda CE Sobral Luciano Feijo 9 86
Francisco Valdir Braga de Sousa Filho CE Sobral Sant'Ana 9 86
Gabriel Ferreira Gomes Amgarten CE Fortaleza Ari de S Cavalcante 9 86
Gabriela Barros Freitas CE Fortaleza Master 9 86
Joo Batista T. de Lima Jnior CE Fortaleza Ari de S Cavalcante 8 86
Lucas Silva Nogueira Gomes CE Fortaleza Teleyos 8 86
Maria Lusa Souza Barbosa BA Jequi Dom Pedro II 9 86
Rafael Peixoto Pagliaro SP So Paulo Albert Sabin 9 86
Brbara Simes Narciso MG Juiz de Fora Col. Militar 9 85
Gabriela Soares Cortez PI Teresina Santa Maria Goretti 9 85
Louse Maria Almeida de Carvalho PE Recife 17 de Agosto 9 85
Mateus Machado Decina MG Belo Horizonte Bernoulli 9 85
Matheus Henrique de A. Camacho SP So Paulo Objetivo Paulista 9 85
Tain Maria Alves de Sousa DF Taguatinga Leonardo da Vinci 9 85
Alexandre Maciel Ramos MG Campanha CEDEC 9 84
Alcia Fortes Machado PI Teresina Dom Barreto 9 84
Bruna Aguiar de Carvalho CE Fortaleza Master 9 84
Bruna Karoliny Sales Guimares SE Aracaju Amadeus 9 84
Caio Pessoa Cruz CE Fortaleza Sete de Setembro 8 84
Resultado
Programa Nacional Olimpadas de Qumica <<
42
Daniel Daher Moura CE Fortaleza Ari de S Cavalcante 9 84
Dennyse Arajo Andrade CE Sobral Farias Brito Sobralense 9 84
Dilson Francisco Bezerra Jnior PI Teresina Dom Barreto 9 84
Eduardo de Medeiros Carlos AL Macei Seb COC Macei 9 84
Eduardo Reis Cavalcante de Farias PI Teresina Dom Barreto 9 84
Francisco Maxwell Leite Barbosa CE Juazeiro do Norte Paraiso 9 84
Francisco Thalysson A. Rodrigus CE Quixeramobim Senso 9 84
Guilherme Goulart Kowalczuk RS Porto Alegre Militar 9 84
Gustavo Dehaini MG Belo Horizonte Magnum Agostiniano 9 84
Ingred Pimentel Guimares CE Fortaleza Ari de S Cavalcante 9 84
Jonathan Raniere P. de Oliveira CE Fortaleza Master Sul 8 84
Luiz Maurcio S. Ferreira Ribeiro DF Braslia Olimpo 9 84
Marcos Gabriel Souza Dantas CE Fortaleza Dom Felipe 9 84
Maria Stella B. de Freitas Neta CE Juazeiro do Norte Paraiso 9 84
Moiss Santana Oliveira BA Sto Ant de Jesus Santo Antonio de Jesus 9 84
Raquel Luz Sousa PI Teresina Dom Barreto 9 84
Rodolfo Nogueira Lima CE Fortaleza Ari de S Cavalcante 9 84
Sariel Arajo de Souza CE Iguatu Plos 9 84
Victor Massatoshi K. Tsuda SP So Paulo Etapa 9 84
Vincuis Koji Enari SP Pindamonhangaba Emlio Ribas 9 84
Vitria Dias Riguete Chaves MG Ipatinga Educao Criativa 9 84
A lista de alunos agraciados com MENO HONROSA e DEMAIS CLASSIFICADOS
na VI Olimpada Brasileira de Qumica Jnior encontra-se disponvel em www.obquimica.org
VI OBQ Jr
>> Olimpada Brasileira de Qumica - 2013
43
Modalidade A
A
Olimpada Brasileira de Qumica 2013
Fase III
PROVA MODALIDADE A Tempo de durao: 4 horas
QUESTO 1
A dose diria recomendada (DDR) de vitamina C (C
6
H
8
O
6
) para homens e mulheres,
a partir dos 15 anos, de 60 mg; no entanto, o cientista Linnus Pauling, um gran-
de defensor do consumo dessa vitamina, recomendava doses bem maiores. Um
seguidor de Linus Pauling ingeria diariamente 6,82 mols de vitamina C. Essa dose
quantas vezes maior que a recomendada? Dados: C = 12 g/mol, H = 1 g/mol, O =
16 g/mol.
a) 200 b) 1.000 c) 2.000 d) 10.000 e) 20.000
QUESTO 2
Para verifcar se o teor de lcool misturado na gasolina est de acordo com os
padres um teste muito simples realizado: Em um funil de separao adiciona-
-se uma quantidade conhecida de gua gasolina a ser testada, agita-se a mistura
at a formao de duas fases, em cima fca a fase rica em gasolina, embaixo fca
a fase rica na mistura gua + lcool. Subtrai-se o volume da fase rica em gasolina
do volume inicial da mistura gasolina + lcool e determina-se o teor de lcool na
gasolina. A respeito do texto acima assinale a alternativa correta:
a) A gua completamente miscvel no lcool e na gasolina
b) O processo de separao de misturas descrito a fltrao
c) Quando a gua adicionada forma-se uma mistura homognea dos trs com-
ponentes
d) O lcool se desloca da gasolina para a gua quando esta adicionada devido a
maior afnidade entre as molculas de lcool e gua por serem polares.
e) As operaes realizadas so insufcientes para determinar o teor de lcool na gasolina.
Programa Nacional Olimpadas de Qumica <<
44
OBQ-2013 Fase III
QUESTO 3
No incio deste ano, uma tragdia abalou a cidade universitria de Santa Maria -
RS. O incndio na Boate Kiss resultou na morte de mais de 240 jovens, que foram
provocadas principalmente pela ingesto de gases txicos provenientes da quei-
ma do material que revestia o teto da casa noturna. Os laudos da percia compro-
varam a presena de cianeto nos corpos das vtimas. Qual a frmula molecular do
produto da reao de combusto da espuma de poliuretano (EPU)?
a) HCN b) NaCN c) KCN d) CH
2
CN e) C
8
H
7
N
QUESTO 4
Em geral, os fogos de artifcios so utilizados em eventos esportivos, como na
Copa do Mundo. Sabe-se que a colorao resultante da queima dos mesmos est
relacionada com a emisso de radiao luminosa. Se a diferena de energia entre
dois nveis eletrnicos 1,2 eV, a frequncia da luz emitida quando um eltron cai
do nvel mais elevado para o nvel de menor energia, vale:
(Dados: h= 6,626 10
-34
J s; 1 eV = 1,6 10
-19
J)
a) 2,90 10
14
Hz b) 1,45 10
14
Hz
c) 5,80 10
14
Hz d) 2,44 10
14
Hz e) 3,80 10
14
Hz
QUESTO 5
Passaram-se muitos sculos, durante os quais o homem foi acumulando observa-
es e experincias. Diversos processos empricos foram realizados por distintos
cientistas, a fm de contribuir para a construo do modelo atmico atual. De
acordo com o exposto, considere as afrmaes abaixo:
I. A Lei de Lavoisier (Conservao das Massas) e a Lei de Proust (Propores De-
fnidas) serviram de base de sustentao para a Teoria Atmica de Dalton.
II. A massa atmica do elemento calculada pela mdia ponderada do nmero de
massa dos istopos naturais do elemento multiplicado pela respectiva abun-
dncia de cada istopo na natureza.
>> Olimpada Brasileira de Qumica - 2013
45
Modalidade A
III. O nmero mximo de eltrons em um subnvel dado pela seguinte expres-
so matemtica: 4L + 2, onde L o nmero quntico secundrio. Enquanto a
expresso N
2
, onde N o nmero quntico principal determina o nmero de
orbitais em um nvel
IV. A distribuio eletrnica do tomo de cobre (Z=29) no obedece ao diagrama
de Linus Pauling. Logo, pode-se afrmar que o conjunto de nmeros qunticos
para o eltron de diferenciao do tomo de cobre dado por: n = 4, L= 2, m = 2
e s = + ., tendo propriedades ferromagnticas.
V. A realizao de experincias com descargas eltricas em tubo de vidro fechado
contendo gs a baixa presso produz os raios catdicos que, por sua vez, so
constitudos por um feixe de eltrons cuja carga eltrica foi determinada por
Milikan.
Esto corretas somente as afrmaes:
a) I e V b) II, III e V c) II, IV e V d) III, IV e V e) I, II, III e V
QUESTO 6
Uma das preocupaes das autoridades policiais durante eventos esportivos o
aumento do consumo de bebidas alcolicas, principalmente a cerveja. A cerveja
, em geral, ingerida numa baixa temperatura para mascarar o seu gosto amargo.
Esse gosto devido presena do mirceno, que est presente nas folhas de lpulo,
um dos componentes da bebida. Considerando a sua frmula estrutural: C
10
H
16
Qual o total de ligaes sigma() + pi() presentes no mirceno?
a) 12 b) 17 c) 18 d) 25 e) 28
QUESTO 7
A prtica de mudar a cor do cabelo muito comum atualmente, mas ela j co-
nhecida h mais de 2000 anos. Os saxes, por exemplo, eram povos que gostavam
Programa Nacional Olimpadas de Qumica <<
46
OBQ-2013 Fase III
de tingir suas barbas de cores fortes e diferentes, tais como azul, verde e alaranja-
do. Os tipos de colorao podem ser classifcados de acordo com as formulaes e
as substncias qumicas presentes na tintura. Como um dos principais produtos,
destaca-se o hidrxido de amnio, que possui toxicidade aguda mesmo em baixas
concentraes e a sua inalao pode causar difculdades respiratrias.
Adaptao de http://www.brasilescola.com/quimica/quimica -no-tingimento-dos-
-cabelos.htm acessado em 08 de maro de 2013
Adicionando-se 0,12 mol de NH
4
OH a 0,09 mol de NH
4
+
de modo a obter 500 mL de
uma soluo tampo.
(Dados: log 1,8 = 0,26 e log 0,75 = - 0,12).
Se K
b
= 1,8 . 10
-5
, o pH deste tampo igual a:
a) 9,38 b) 4,62 c) 3,98 d) 5,44 e) 6,75
QUESTO 8
temperatura ambiente e presso atmosfrica ao nvel do mar, a gua encontra-
-se na fase lquida. Ela passa para a fase gasosa, numa temperatura que 200C
acima daquela que se esperaria, teoricamente, na ausncia de ligaes de hidrog-
nio. Pode-se concluir, portanto:
a) Essas ligaes so muito fortes entre tomos de molculas diferentes; por isso,
a gua encontra-se na fase lquida nessas condies.
b) A massa da molcula de gua grande em relao ao seu tamanho; por isso, a
gua compactada e torna-se lquida.
c) A densidade da gua maior que a soma da densidade do gs oxignio e do gs
hidrognio; por isso, a massa maior e faz com que as molculas se aproxi-
mem, formando a fase lquida.
d) As ligaes covalentes polares entre os tomos de hidrognio e de oxignio
so mais fortes do que as covalentes apolares entre os tomos de hidrognio;
assim, a repulso maior do que a atrao, formando a fase lquida.
e) Essas ligaes so chamadas de pseudo-inicas; ao invs de formarem uma fase
slida, formam uma fase lquida.
>> Olimpada Brasileira de Qumica - 2013
47
Modalidade A
QUESTO 9
O permanganato de potssio serve para eliminar as bactrias e os fungos da pele,
sendo tambm um bom cicatrizante para as feridas devido as suas propriedades
oxidantes. Comumente, utilizado diludo em gua, para tratamento de cataporas
e feridas generalizadas.
Diante desta afrmao, tem-se a seguinte equao de xido-reduo, que repre-
sentada apenas pelas espcies inicas que participam ativamente do fenmeno de
transferncia de eltrons:
MnO
4
-
+ NO
2
-
+ 6 H
+
Mn
2
+
+ NO
3
-
+ 3 H
2
O
Faa o balanceamento correto dessa equao e calcule a soma total dos coefcien-
tes mnimos e inteiros das espcies qumicas envolvidas.
a) 23 b) 20 c) 26 d) 47 e) 34
QUESTO 10
Na natureza encontram-se certos elementos qumicos que estabelecem um n-
mero de ligaes covalente maior do que aquele previsto pela sua confgurao
eletrnica no estado fundamental. o caso do carbono, berlio, boro e iodo, por
exemplo. O carbono que, na maioria dos compostos, estabelece 4 ligaes cova-
lentes comuns mesmo tendo apenas 2 eltrons desemparelhados no estado fun-
damental. Para explicar o que acontece, foi proposta a Teoria da Hibridizao, que
consiste na interpenetrao de orbitais puros. Considerando a teoria da repulso
dos pares eletrnicos, as polaridades das ligaes, as estruturas moleculares e a
hibridao, analise as afrmaes abaixo:
I. Podemos afrmar que a molcula de NH
3
: polar, piramidal e tem hibridao
sp
3
enquanto a molcula de CO
2
apolar, linear e possui hibridao sp
2
.
II. Os compostos BF
3
, PCl
5
, BeH
2
e IF
7
so molculas que desobedecem a regra do
octeto e apresentam hibridao, respectivamente: sp
3
, sp
3
d, sp e sp
3
d
2
.
III. No gs acetileno, C
2
H
2
muito usado em soldas, ocorre uma ligao sigma (dsp-
sp) entre os tomos de carbono.
IV. Quando colocamos um refrigerante no congelador, por um tempo prolongado,
ocorre a expanso de seu contedo como consequncia da reorganizao das
Programa Nacional Olimpadas de Qumica <<
48
OBQ-2013 Fase III
molculas de gua em uma estrutura cristalina hexagonal e da formao de
ligaes de hidrognio.
Aps a anlise, assinale a alternativa correta.
a) III e IV b) I, II e IV c) II, III e IV d) I, III e IV e) I, II, III e IV
QUESTO 11
IME - 1996 Uma fbrica, que produz cal (Ca(OH)
2
necessita reduzir o custo da
produo para se manter no mercado com preo competitivo para seu produto.
A direo da fbrica solicitou ao departamento tcnico o estudo da viabilidade de
reduzir a temperatura do forno de calcinao de carbonato de clcio, dos atuais
1500K, para 800K. Considerando apenas o aspecto termodinmico, pergunta-se: o
departamento tcnico pode aceitar a nova temperatura de calcinao? Em caso
afrmativo, o departamento tcnico pode fornecer uma outra temperatura de
operao que proporcione maior economia? Em caso negativo, qual a tempera-
tura mais econmica para operar o forno de calcinao?
Observaes: desconsidere a variao das propriedades com a temperatura.
Observaes: desconsidere a variao das propriedades com a temperatura.
Dados:
CaCO
3
(s) S = 92,9 H = -1206,9
CaO(s) S = 39,8 H = - 635,1
CO2(g) S = 213,6 H = - 393,5
Onde S representa entropia em J.mol
-1
.K
-1
e H entalpia em kJ.mol
-1
.
QUESTO 12
Duas fontes de cobre so cuprita, que tem o nome IUPAC de xido de cobre (I), e
malaquita, que tem a frmula Cu
2
CO
3
(OH)
2
. O cobre utilizado em fao predial
e motores eltricos porque tem boa condutividade eltrica. Outros usos do cobre
no relacionados com a sua condutividade eltrica so as moedas, encanamentos,
telhados e panelas de cozinha. Alumnio tambm utilizado para fabricao de
panelas. temperatura ambiente, a condutividade eltrica de um fo de cobre
1,6 vezes maior do que um fo de alumnio com o mesmo comprimento e seo
>> Olimpada Brasileira de Qumica - 2013
49
Modalidade A
transversal. temperatura ambiente, a condutividade trmica do cobre 1,8 ve-
zes maior do que a condutividade de calor de alumnio.
Em temperatura ambiente, a densidade do cobre 3,3 vezes maior do que a den-
sidade de alumnio.
a) Escreva a frmula qumica do cuprita.
b) Determine o nmero de oxidao do oxignio no on carbonato encontrado
em malaquita.
c) Identifque uma propriedade fsica de cobre que torna uma boa escolha para
usurios que no esto relacionados com a condutividade eltrica.
d) Identifque uma propriedade fsica do alumnio que poderia torn-lo uma op-
o melhor do que o cobre para uma panela.
QUESTO 13
O cido brico frequentemente utilizado como insecticida relativamente at-
xico, para matar baratas, cupins, formigas, pulgas e muitos outros insetos. Pode
ser utilizado diretamente sob a forma de p em pulgas, misturando-o com acar
de confeiteiro como atrativo para as formigas e baratas. Sobre esse cido e suas
propriedades, responda:
a) O cido brico pode reagir com a gua, gerando o nion tetrahidroxiborato.
Esse processo descrito pela seguinte equao:
B(OH)
3
(aq) + 2 H
2
O(l) H
3
O
+
(aq) + B(OH)
4
-
(aq)
Desenhe a estrutura de Lewis para todas as espcies apresentadas, incluindo em
sua resposta, a geometria e a hibridizao do tomo central em cada uma delas.
b) Quando o cido brico, B(OH)
3
, aquecido acima de 170 C, o mesmo desidrata
gerando o cido metabrico. Se o aquecimento continuar at cerca de 300C,
h uma nova desidratao e a consequente formao do cido pirobrico.
Caso haja aquecimento adicional, o cido pirobrico transforma-se em tri-
xido de boro. Escreva as trs reaes descritas nesta questo e apresente a
frmula estrutural das espcies em negrito.
Programa Nacional Olimpadas de Qumica <<
50
OBQ-2013 Fase III
c) Outro composto de boro relevante a borazina de frmula molecular B
3
H
6
N3.
Este composto tambm chamado de benzeno inorgnico, visto que isoe-
letrnico e isoestrutural ao benzeno. possvel, ainda, observar na estrutura
da borazina a presena de um cido de Lewis e de uma base de Lewis. Identif-
que-os na molcula em questo, utilizando um exemplo como embasamento
para sua resposta.
QUESTO 14
O trixido de boro um composto utilizado como aditivo da fbra ptica, na pro-
duo de vidro de borossilicato, entre outros. Esse composto obtido pela desi-
dratao do cido brico, porm tambm possvel consegui-lo a partir das se-
guintes etapas de reao.
Etapa 1: B
2
O
3
(s) + 3 H
2
O(g) 3 O
2
(g) + B
2
H
6
(g) 2035 H/kJ
Etapa 2: H
2
O(l) H
2
O(g) 44 H/kJ
Etapa 3: H
2
(g) + 1/2 O
2
(g) H
2
O(l) 286 H/kJ
Etapa 4: 2 B(s) + 3 H
2
(g) B
2
H
6
(g) 36 H/kJ
Tendo como base as reaes acima e suas respectivas entalpias, calcule a entalpia
geral de formao do trixido de boro e informe a sua equao global.
QUESTO 15
Considere o elemento
a) Qual o nmero de nutrons, prtons e eltrons que possui o on X
3+
?
b) Se o ion X
3+
se combinar com o oxignio, qual a frmula qumica mais provvel
para o composto formado?
c) Qual a massa de um tomo de X? Expresse o resultado em quilograma.
d) Escreva a confgurao eletrnica do ction formado com os 17 eltrons de
menor energia de X.
>> Olimpada Brasileira de Qumica - 2013
51
Modalidade A
QUESTO 16
A solubilidade de certos sais em soluo pode ser alterada drasticamente pela
adio de agentes complexantes, o que pode gerar srios problemas ambientais
devido a aumentos de solubilidade de alguns poluentes na presena de resduos
qumicos com propriedades complexantes.
a) Calcule a solubilidade do cianeto de prata, considerando apenas seu equilbrio
de precipitao. (K
ps
=2,2.10
-16
)
b) Calcule a solubilidade deste mesmo sal, considerando agora tambm seu equi-
lbrio de complexao entre os ons prata e cianeto e a razo
[Ag
+
]/[Ag(CN)
2-
] . (K
1
.K
2
=5,3.10
18
)
c) Calcule novamente a solubilidade deste mesmo sal, e a razo [Ag
+
] /[Ag(CN)
2-
],
sabendo que o pH de sua soluo saturada 7,15, o que no sufciente para
que precipite xido de prata. {Ka(HCN)= 4,0.10
-10
}
As pessoas nem sempre reconhecero
o que voc fez, muitas vezes elas s
enxergaro o que voc deixou de fazer.
Programa Nacional Olimpadas de Qumica <<
52
OBQ-2013 Fase III OBQ-2013 Fase III
B
Programa Nacional Olimpadas de Qumica
Olimpada Brasileira de Qumica 2013
Fase III
PROVA MODALIDADE B Tempo de durao: 4 horas
QUESTO 1
Sem fazer observaes, no nosso cotidiano convivemos com grande quantidade
de fenmenos qumicos, dentre esses se destacam as propriedades coligativas.
Isto pode ser observado no simples processo de cozimento de alimentos, pois aos
alimentos (carnes, legumes e verduras) se adiciona gua em quantidade e sal de
cozinha (cloreto de sdio, NaCl), seguido de aquecimento at fervura para amo-
lecer os alimentos e proporcionar sabor e aroma agradveis, resultando uma boa
refeio. Quanto s propriedades coligativas para o referido processo pode afr-
mar que:
I o sal adicionado pode aumentar a temperatura de ebulio da gua na mistura
em comparao com a temperatura ebulio normal da gua pura;
II os legumes como tubrculos (batatas, cenouras e outros), se comportam como
uma membrana semi-impermevel semelhante ao fenmeno da osmose;
III os alimentos e o sal adicionados interferem na presso de vapor da gua, ou
melhor, diminui a presso de vapor da gua;
IV se em vez de aquecer a mistura (alimentos, gua e sal), e essa fosse resfriada o
seu congelamento ocorrer a uma temperatura > 0
o
C.
Assinale a alternativa correta:
a) As afrmativas I e II, esto corretas. b) As afrmativas I e III, esto corretas.
c) As afrmativas I e IV, esto corretas. d) As afrmativas II e III, esto corretas.
e) As afrmativas III e IV, esto corretas.
>> Olimpada Brasileira de Qumica - 2013
53
Modalidade B
QUESTO 2
Na natureza encontram-se certos elementos qumicos que estabelecem um n-
mero de ligaes covalente maior do que aquele previsto pela sua confgurao
eletrnica no estado fundamental. o caso do carbono, berlio, boro e iodo por
exemplo. O carbono que, na maioria dos compostos, estabelece 4 ligaes cova-
lentes comuns mesmo tendo apenas 2 eltrons desemparelhados no estado fun-
damental. Para explicar o que acontece, foi proposta a Teoria da Hibridizao, que
consiste na interpenetrao de orbitais puros. Considerando a teoria da repulso
dos pares eletrnicos, as polaridades das ligaes, as estruturas moleculares e a
Hibridao, analise as afrmaes a baixo:
I. Podemos afrmar que a molcula de NH
3
: polar, piramidal e tem hibridao
sp
3
enquanto a molcula de CO
2
apolar, linear e possui hibridao sp
2
.
II. Os compostos BF
3,
PCl
5
, BeH
2
e IF
7
so molculas que desobedecem a regra do
octeto e apresentam hibridao, respectivamente: sp
3
, sp
3
d, sp e sp
3
d
2
.
III. No gs acetileno, C
2
H
2
muito usado em soldas, ocorre uma ligao s
sp - sp
entre
os tomos de carbono que apresenta hibridao sp.
IV. Quando colocamos um refrigerante no congelador, por um tempo prolonga-
do, ocorre a expanso de seu contedo como consequncia da reorganizao
das molculas de gua em uma estrutura cristalina hexagonal e da formao
de ligaes de hidrognio.
Aps a anlise, assinale a alternativa correta.
a) III e IV b) I, II e IV c) II, III e IV d) I, III e IV e) I, II, III e IV
QUESTO 3
A dose diria recomendada (DDR) de vitamina C (C
6
H
8
O
6
) para homens e mulhe-
res, a partir dos 15 anos, de 60 mg; no entanto, o cientista Linnus Pauling, um
grande defensor do consumo dessa vitamina, recomendava doses bem maiores.
Um seguidor de Linus Pauling ingeria diariamente 6,82 mols de vitamina C. Essa
dose quantas vezes maior que a recomendada? Dados: C = 12 g/mol, H = 1 g/mol,
O = 16 g/mol.
a) 200 b) 1.000 c) 2.000 d) 10.000 e) 20.000
Programa Nacional Olimpadas de Qumica <<
54
OBQ-2013 Fase III
QUESTO 4
Durante o exerccio, a liberao de energia na forma de calor e a conseqente
elevao da temperatura corporal envolvem mecanismos fsiolgicos potenciais
para promover a perda de calor. No entanto, para que a perda de calor ocorra, o
excesso de calor deve primeiro ser transportado da regio central do corpo para
a periferia (pele) onde pode ser perdido para o ambiente.
http://www.ativo.com/Canais/Pages/Aperdadecalor.aspxacessadoem08demarode2013.
Considerando que a liberao de calor do corpo humano durante um exerccio
fsico, de uma hora, ocorre de forma similar a uma reao exotrmica que produz
-500 kJ de calor, julgue os itens abaixo:
I. A reao direta tem entalpia de -500 kJ. Assim, se a variao da entropia for
maior que zero a temperatura constante, tem-se uma reao espontnea.
II. Se a reao direta possuir energia de ativao de valor igual a 350 kJ, obriga-
toriamente, a reao inversa ter energia de ativao de valor igual a -350 kJ.
III. Ao aumentar a temperatura deste sistema termodinmico, aumentar tam-
bm o nmero de colises e, consequentemente, a desordem do sistema. Pode-
-se afrmar, ento, que a entropia ser positiva e crescente.
IV. De acordo com os princpios de Le Chatelier, a velocidade da reao reduzida
com o aumento da temperatura.
V. A velocidade da reao decresce com o aumento da temperatura, pois aumenta
a energia cintica mdia das molculas e, consequentemente, a entropia da reao.
Esto corretas:
a) I e IV b) II e IV c) III e IV d) II e V

QUESTO 5
A produo do vinagre envolve dois tipos de alteraes bioqumicas: uma fermen-
tao alcolica de um carboidrato e uma oxidao deste lcool at cido actico;
que ajuda a evitar o acmulo de gordura nos vasos sanguneos e anti-infamat-
rio. A ingesto de cido actico estimula tambm a produo do suco gstrico me-
lhorando a digesto e o metabolismo. O vinagre de ma tem ainda o cido mlico,
que aumenta a temperatura das clulas queimando mais gorduras.
>> Olimpada Brasileira de Qumica - 2013
55
Modalidade B
Adaptao de http://g1.globo.com/jornal-hoje/noticia/2011/08/vinagre-faz-bem-para-saude-e-
-pode-ser-utilizado-ate-em-sobremesas.htmlacessadoem08demarode2013.
Utilizando-se 100 mL de cido actico 0,2 mol/L e adicionando-se a este volume
uma certa massa de acetato de sdio. A soluo tampo indicou um pH igual a 5.
A massa, em gramas, de acetato de sdio adicionado soluo foi de aproximada-
mente (Dados: pKa do cido actico = 4,76 ; antilog 0,24 = 1,74):
a) 3,3 g b) 1,3 g c) 2,9 g d) 1,9 g e) 3,6 g
QUESTO 6
No incio deste ano, uma tragdia abalou a cidade universitria de Santa Maria -
RS. O incndio na Boate Kiss resultou na morte de mais de 240 jovens, que foram
provocadas principalmente pela ingesto de gases txicos provenientes da quei-
ma do material que revestia o teto da casa noturna. Os laudos da percia compro-
varam a presena de cianeto nos corpos das vtimas. Qual a frmula molecular do
produto da reao de combusto da espuma de poliuretano (EPU)?
a) HCN b) NaCN c) KCN d) CH
2
CN e) C
8
H
7
N
QUESTO 7
O permanganato de potssio serve para eliminar as bactrias e os fungos da pele,
sendo tambm um bom cicatrizante para as feridas devido as suas propriedades
oxidantes. Comumente, utilizado diludo em gua, para tratamento de cataporas
e feridas generalizadas, por exemplo.
Diante desta afrmao, tem-se a seguinte equao de xido-reduo, que repre-
sentada apenas pelas espcies inicas que participam ativamente do fenmeno de
transferncia de eltrons:
MnO
4
-
+ NO
2
-
+ 6H
+
Mn
2+
+ NO
3
-
+ 3 H
2
O
Faa o balanceamento correto dessa equao e calcule a soma total dos coefcien-
tes mnimos e inteiros das espcies qumicas envolvidas.
a) 23 b) 20 c) 26 d) 47 e) 34
Programa Nacional Olimpadas de Qumica <<
56
OBQ-2013 Fase III
QUESTO 8
A reao de saponifcao no podia ter outro nome, uma vez que fcou muito co-
nhecida em razo de sua enorme utilizao na indstria, na produo de sabone-
tes, sabes e detergentes. Para que essa reao acontea, preciso haver um ster
misturado com uma base forte na presena de gua e aquecimento. O produto
fnal um sal orgnico e lcool. Observe a equao abaixo:

Adaptao de http://www.brasilescola.com/quimica/reacao-saponifcacao.htm acessado em 28 de feverei-
ro de 2013.
O processo de saponifcao citado acima trata-se de uma reao muito presente
no cotidiano. Sobre a reao ilustrada e as espcies qumicas presentes, julgue os
itens a seguir.
Os steres presentes nos leos so geralmente mais estveis que os hidrocarbone-
tos, devido a maior afnidade do carbono com o oxignio, o que os fazem ter um
ponto de ebulio relativamente elevado em comparao com os hidrocarbonetos.
Os cidos graxos so componentes orgnicos, pois possuem carbono e hidrognio
em sua formao, enquanto o hidrxido de sdio um composto inorgnico.
Na molcula de cido graxo h uma ligao dupla do carbono com oxignio, nesse
caso o carbono no tem hibridizao sp, pois esse mesmo carbono est fazendo
apenas ligao simples com o radical orgnico.
A reao ilustrada tambm pode ser considerada uma reao de neutralizao,
pois um cido e uma base esto reagindo, formando um sal, no caso o sabo.
O hidrxido de sdio uma base forte, isso se deve ao fato da fora de ligao ser
relativamente alta entre a hidroxila e o sdio, sendo assim tem maior capacidade
de ionizar.
Com base nas alternativas, marque abaixou quais esto certas.
a) II e IV b) IV e V c) I e II d) I, II e IV e) II, III e IV
>> Olimpada Brasileira de Qumica - 2013
57
Modalidade B
QUESTO 9
No olho humano, especifcamente na retina, o cis-11- retinal se transforma no
trans-11- retinal pela ao da luz e assim produz impulso eltrico para formar a
imagem. Por isso o ser humano precisa de luz para enxergar.
Adaptao de http://www.ufpe.br/cap/images/quimica/katiaaquino/3anos/aulas/aulaisome-
ria.pdf acessado em 08 de maro de 2013.
O texto acima demonstra umas das potenciais aplicaes da isomeria. Essa pro-
priedade da matria tem grandes aplicaes no segmento de frmacos e no trata-
mento de doenas. Assim sendo, considere as seguintes afrmaes em relao
isomeria e estabilidade de compostos orgnicos:
Os ismeros etanol e ter dimetlico so ismeros constitucionais entre eles.
O ciclobutano e o metil ciclopropano so ismeros de ncleo.
O alcano de frmula molecular C
6
H
14
apresenta seis possveis ismeros.
Existem quatro alcenos isomricos da frmula molecular C
4
H
8
.
Dos dois 1,3,5-trimetilciclo-hexanos estereoisomricos a seguir, o mais estvel o
trans-1,3,5- trimetilciclo-hexano.
cis- 1,3,5- trimetilciclo-hexano trans- 1,3,5- trimetilciclo-hexano
Das afrmaes acima, esto CORRETAS apenas:
a) I, II e IV b) I, II, III e IV. c) I, II, IV e V. d) II, IV e V. e) I, II e V.
Programa Nacional Olimpadas de Qumica <<
58
OBQ-2013 Fase III
QUESTO 10
Bombas de Sdio e Potssio
A diferena da concentrao in-
tracelular e extracelular de substncias
e ons atravs da membrana plasmtica
pode ser mantida por transporte passi-
vo (sem gasto de energia) ou por trans-
porte ativo (com gasto de energia).
Na bomba de sdio e potssio a
concentrao do sdio maior no meio
extracelular enquanto a de potssio
maior no meio intercelular. A manu-
teno dessas concentraes reali-
zada pelas protenas transportadoras
descritas anteriormente que capturam
ons sdio (Na
+
) no citoplasma e bom-
beia-os para fora da clula. No meio
extracelular, capturam os ons potssio
(K
+
) e os bombeiam para o meio interno.
Se no houvesse um transporte efciente, a concentrao destes ons iria se igualar.
http://www.infoescola.com/biologia/bomba-de-sodio-e-potassio/ acessado em 07 de maro de 2013
De acordo com o texto acima e seus conhecimentos sobre tabela peridica e os
tipos de transporte, assine a resposta correta:
a) A energia de ionizao dos metais alcalinos e alcalinos terrosos comparativa-
mente alta quando se analisa os outros componentes da tabela peridica. Desta
forma, pode-se afrmar que a energia de ionizao diretamente proporcional a
eletronegatividade.
b) O potssio um metal alcalino bivalente, ou seja, se ligar a no-metais que
realizem duas ligaes, alm de ser o menos eletropositivo de todos os elementos
presentes na tabela peridica.
c) O carter metlico do sdio garante a facilidade em se retirar um eltron deste
metal que, aps a perda, fcar com confgurao eletrnica similar a do nenio.
>> Olimpada Brasileira de Qumica - 2013
59
Modalidade B
d) Os ons de sdio encontrados na bomba de sdio e potssio possuem raios maio-
res que o sdio metlico, atuando no processo celular por meio do transporte ativo.
e) Quanto maior a eletronegatividade, menor ser a tendncia do elemento em
capturar eltrons.
PARTE II
QUESTO 11
Uma fbrica, que produz cal (Ca(OH)
2
necessita reduzir o custo da produo para
se manter no mercado com preo competitivo para seu produto. A direo da f-
brica solicitou ao departamento tcnico o estudo da viabilidade de reduzir a tem-
peratura do forno de calcinao de carbonato de clcio, dos atuais 1500K, para
800K. Considerando apenas o aspecto termodinmico, pergunta-se: o departa-
mento tcnico pode aceitar a nova temperatura de calcinao? Em caso afrma-
tivo, o departamento tcnico pode fornecer uma outra temperatura de operao
que proporcione maior economia? Em caso negativo, qual a temperatura mais
econmica para operar o forno de calcinao?
Observaes: desconsidere a variao das propriedades com a temperatura.
Dados:
CaCO
3
(s) S = 92,9 H = -1206,9
CaO(s) S = 39,8 H = - 635,1
CO
2
(g) S = 213,6 H = - 393,5
Onde S representa entropia em J.mol
-1
.K
-1
e H entalpia em kJ.mol
-1
.

QUESTO 12
A Anlise elementar quantitativa de um composto orgnico X forneceu o seguinte
resultado: 0,2035 g de X por combusto forneceu 0,4840 g de CO
2
e 0,2475 g de
H
2
O. Na determinao da densidade de vapor, de acordo com o processo de Victor
Meyer, 0,14 g deste composto deslocam 44,5 cm
3
de ar atmosfrico, medidos a 21C
e 798 mm Hg de presso. A presso de vapor dgua a 21C 19 mm de Hg. Calcular
Programa Nacional Olimpadas de Qumica <<
60
OBQ-2013 Fase III
a frmula molecular de X e escrever a sua frmula estrutural, bem como a de seus
ismeros, indicando qual deles apresenta estereoisomeria ptica.
QUESTO 13
O cido brico frequentemente utilizado como inseticida relativamente atxi-
co, para matar baratas, cupins, formigas, pulgas e muitos outros insetos. Pode ser
utilizado diretamente sob a forma de p em pulgas, misturando-o com acar de
confeiteiro como atrativo para as formigas e baratas. Sobre esse cido e suas pro-
priedades, responda:
a) O cido brico pode reagir com a gua, gerando o nion tetrahidroxiborato.
Esse processo descrito pela seguinte equao:
B(OH)
3
(aq) + 2 H
2
O (l) H
3
O
+
(aq) + B(OH)
4
-
(aq)
Desenhe a estrutura de Lewis para todas as espcies apresentadas, incluindo em
sua resposta, a geometria e a hibridizao do tomo central em cada uma delas.
b) Quando o cido brico, B(OH)
3
, aquecido acima de 170C, o mesmo desidrata
gerando o cido metabrico. Se o aquecimento continuar at cerca de 300C, h
uma nova desidratao e a consequente formao do cido pirobrico. Caso haja
aquecimento adicional, o cido pirobrico transforma-se em trixido de boro. Es-
creva as trs reaes descritas nesta questo e apresente a frmula estrutural das
espcies em negrito.
c) Outro composto de boro relevante a borazina de frmula molecular B
q
H
6
N
3
.
Este composto tambm chamado de benzeno inorgnico, visto que isoeletr-
nico e isoestrutural ao benzeno. possvel, ainda, observar na estrutura da borazi-
na a presena de um cido de Lewis e de uma base de Lewis. Identifque-os na mo-
lcula em questo, utilizando um exemplo como embasamento para sua resposta.
QUESTO 14
O trixido de boro um composto utilizado como aditivo da fbra ptica, na pro-
duo de vidro de borossilicato, entre outros. Esse composto obtido pela desi-
dratao do cido brico, porm tambm possvel consegui-lo a partir das se-
guintes etapas de reao.
>> Olimpada Brasileira de Qumica - 2013
61
Modalidade B
H/kJ
Etapa 1: B
2
O
3
(s) + 3 H
2
O(g) -> 3 O
2
(g) + B
2
H
6
(g) 2035
Etapa 2: H
2
O(l) -> H
2
O(g) 44
Etapa 3: H
2
(g) + 1/2 O
2
(g)
-> H
2
O(l) 286
Etapa 4: 2 B(s) + 3 H
2
(g) -> B
2
H
6
(g) 36
Tendo como base as reaes acima e suas respectivas entalpias, calcule a entalpia
geral de formao do trixido de boro e informe a sua equao global.
QUESTO 15

A composio de membranas celulares majoritariamente fosfolipdica, apresen-
tando tambm glicolipdios e colesterol. A fuidez da membrana de uma clula
animal controlada, de forma geral, de duas maneiras: pelos cidos graxos pre-
sentes na estrutura dos fosfolipdios e dos glicolipdios componentes, pela quanti-
dade de colesterol em sua composio.
a) Explique como o grau de saturao, o tamanho da cadeia carbnica e a exis-
tncia de ismeros cis e trans dos cidos graxos presentes na membrana afetam
a fuidez.
b) Explique o controle da fuidez pelo colesterol, utilizando as fguras a seguir
como suporte para sua resposta. No esquea de justifcar as suas respostas!

Figura1.Representaodoposicionamento
espacialdamolculadecolesterolna
membranacelular.
Figura2.Representaogrfcada
infunciadaconcentraodecolesterol
nafuidezdamembranacelular.
Programa Nacional Olimpadas de Qumica <<
62
QUESTO 16
Considere as ilustraes abaixo:

a. Complete os espaos acima com as espcies qumicas pertinentes s transfor-
maes dadas.
b. Quais so os mecanismos das duas primeiras etapas?
c. Que tipo de reao ocorre na terceira etapa?
d. Por que essa escolha de reagentes para esse processo?
OBQ-2013 Fase III
>> Olimpada Brasileira de Qumica - 2013
63
Resultados
MODALIDADE A
O U R O
NOME ESCOLA CIDADE CIDADE NOTA
Artur Souto Martins Ari de S Fortaleza CE 100,0
Giovanni Elson R. de Souza Farias Brito Fortaleza CE 98,32
Gianluca Carrilho Malta GGE Recife PE 96,66
Gabriel Queiroz Moura Dom Barreto Teresina PI 95,92
Glicia Rodrigues Ferreira Master Fortaleza CE 95,92
Lia de Oliveira Domingues Farias Brito Fortaleza CE 95,92
Gabriel Alves da Silva Diniz Etapa So Paulo SP 95,16
Pedro Teotnio de Sousa Ari de S Fortaleza CE 95,00
Maurocelio Rocha Pontes Filho Farias Brito Fortaleza CE 94,92
P R A T A
NOME ESCOLA CIDADE CIDADE NOTA
George Henrique N. M. Junior Farias Brito Fortaleza CE 93,66
George Henrique N. M. Junior Farias Brito Fortaleza CE 93,66
Danilo Aparecido C. Castillo CDC Ribeiro Preto SP 93,42
Isabelle de Sousa Pereira Farias Brito Fortaleza CE 93,42
Geisa Costa O. de M. Santana Farias Brito Fortaleza CE 93,25
Luciano Pinheiro Batista Farias Brito Fortaleza CE 93,21
Matheus Henrique Martins Costa Boa Viagem Recife PE 92,58
Chan Song Moon Colgio Etapa So Paulo SP 92,49
Sarah Machado Veldhui Farias Brito Fortaleza CE 91,36
Giberto Mitsuyoshi Yuki Jnior Olimpo Braslia DF 90,83
Joo Martins Cortez Filho Dom Barreto Teresina PI 90,16
Elcio Koodiro Yoshida Etapa So Paulo SP 90,08
Victor Sousa Silva Ari de S Fortaleza CE 90,08
Lucas Campos Barbosa e Silva CEFET-MG Contagem MG 90,00
David Silva Almeida Dom Barreto Teresina PI 89,53
Loureno Gianizeli Paganotti Guimares Rosa C. do Itapemirim ES 89,25
Leonardo Henrique Martins Florentino Objetivo Vergueiro So Paulo SP 89,16
Pedro Macedo Flores Militar Juiz de Fora MG 88,69
Eduardo Serpa Master Caucaia CE 88,67
Programa Nacional Olimpadas de Qumica <<
64
Iago Dantas Figueirdo Anchieta Salvador BA 88,66
Pedro Jorge Luz A. Cronemberger Sag. Corao de Jesus Teresina PI 87,72
Pedro Henrique Rocha de Freitas Olimpo Brasilia DF 87,18
Kevin Eiji Iwahita Colegio Etapa So Paulo SP 86,83
Vinicius Silva de Oliveira Motivo Recife PE 86,82
Josu Hofmeister de Azevedo Liberato Salzano Novo Hamburgo RS 86,75
Iana Soares Negreiros Farias Brito Fortaleza CE 86,64
Maria Paula Tellez Frias Sartre Coc - Nobel Salvador BA 86,44
Bruna Alves Ramalho Militar Rio de Janeiro RJ 86,33
Fbio Gabriel Costa Nunes Dom Barreto Teresina PI 86,20

B R O N Z E
NOME ESCOLA CIDADE CIDADE NOTA
Aline Bruno Figueiredo COC Ribeiro Preto Ribeiro Preto SP 85,08
Victor Machado Integral Teresina PI 84,90
Carlos Augusto F. F. de Carvalho Anchieta Salvador BA 84,58
Viviane Harue Higa Objetivo Gurupi TO 84,53
Jos Rodolfo de Farias Neto Contato Macei AL 84,42
Arthur Lasak Okuda Colegio Bandeirantes So Paulo SP 84,05
Seon Augusto de Souza Ferreira Militar Salvador BA 83,69
Daniel Tenrio C. Soares GGE J. dos Guararpes PE 83,35
Erick Tavares Marcelino Alves Contato Macei AL 83,34
Rodrigo Silva Ferreira Militar Salvador BA 83,08
Enrico Pascucci Lofel Etapa So Paulo SP 83,03
Joo Felipe Nascimento Mattos Olimpo Braslia DF 83,01
Ricardo Pinheiro Fernandes Lages Lato Sensu Manaus AM 82,96
Bruna Luiza Braga Pantoja 7 de setembro Fortaleza CE 82,68
Joo Paulo Mota Telles Anchieta Salvador BA 82,27
Sarah Barreto Ornellas Anchieta Salvador BA 82,20
Thiago da Silva Pinto Col. Dom Jaime Cmara So Jos SC 82,18
Mateus Almeida F. dos Santos Dom Barreto Teresina PI 81,91
Icaro de Azevedo Alexandre Ari de S Fortaleza CE 81,84
Victor Hugo Vianna Silva Militar Niteri RJ 81,73
Fernando Henrique Carlesso Notre Dame Passo Fundo RS 81,44
Gabriela Hidalgo V. dos Santos Albert Sabin So Paulo SP 81,29
Henrique Jos Agrizzi Alto Guimares Rosa C. do Itapemirim ES 81,25
OBQ-2013 Fase III
>> Olimpada Brasileira de Qumica - 2013
65
Resultados
Pedro Salazar Costa Adalberto Valle Manaus AM 81,17
Mateus Bezrutchka Colegio Etapa Taboo da Serra SP 80,92
Alessandra Dorigon Colgio Militar Porto Alegre RS 80,86
Breno Lima De Almeida Anchieta Salvador BA 80,83
Marcus Amaral Leite Santo Agostinho Belo Horizonte MG 80,76
Andr Lus de Alcntara Ramos Militar Braslia DF 80,52
Gustavo Henrique dos Santos Coesi Aracaju SE 80,50
Lia de Freitas Arajo Alves Farias Brito Fortaleza CE 80,50
Lucas Bastos Oliveira Dulia Bringel Fortaleza CE 80,50
Lucas Tadeu Rocha Santos Farias Brito Sobral CE 80,36
Pedro Henrique Vitorino SESI Barbacena MG 80,36
Daniel Marques Silva Farias Brito Eusebio CE 80,35
Pedro Filipe Medeiros Gomes Col. Aplic. da UFPE Recife PE 80,33
Leticia Campos de Oliveira Master Fortaleza CE 80,05
Eulalio Sotero Galvao Junior Lettera Teresina PI 80,04
Matheus Santos Santana EE La Sale Rondonpolis MT 79,94

Meno Honrosa e Demais Classifcados
Ver lista em www.obquimica.org
"Se voc tem uma mente calma,
Ser uma pessoa bela.
Se voc uma pessoa bela,
Criar um lar harmonioso.
Se o seu lar est em harmonia,
Sua nao se encontrar em ordem.
Se em sua nao h ordem,
Haver paz no mundo."
Lao Ts
Programa Nacional Olimpadas de Qumica <<
66
OBQ-2013 Fase III
MODALIDADE B
O U R O
NOME ESCOLA CIDADE CIDADE NOTA
Francisco Markan N. Souza Filho Farias Brito Fortaleza CE 100,0
Nathercia Castro Mota Master Fortaleza CE 99,77
Thiago Matheus Santos Rios IFBA Salvador BA 99,38
Luckeciano Carvalho Melo Lettera Teresina PI 98,22
Nicholas de Souza Costa Lima Ari de S Fortaleza CE 96,20
Jlia Cristina Soares de Sousa Dom Barreto Teresina PI 95,14
Felipe Celso Reis Pinheiro Dom Barreto Teresina PI 91,30
P R A T A
NOME ESCOLA CIDADE CIDADE NOTA
Kamayra Gomes Mendes EE Sen.r Joo Bosco Parintins AM 89,77
Giovani Florencio Scarpelli Junior IFRJ Rio de Janeiro RJ 89,31
Luis Fernando M. Poletti Valle Colegio Mater Amabuis Guarulhos SP 88,06
Gabriel Castro Tavares Colgio Militar Manaus AM 87,82
Thiago Silva Viana IFRJ Rio de Janeiro RJ 86,71
Victor Hugo Fernandes Breder Dom Barreto Teresina PI 86,60
Luiz De Paulo Santana IFBA Salvador BA 86,14
Matheus Felipe S.B.F. Rodrigues Militar Braslia DF 84,07
Joaquim Ivo V. Dantas Landim Farias Brito Fortaleza CE 83,58
Victor Domene Ribeiro Dos Santos Colegio Bandeirantes So Paulo SP 83,13
Isabella Vargas Baldon So Domingos Vitria ES 82,92
Guilherme Pires Barroso Colegio Bandeirantes So Paulo SP 82,67
Guilherme Renato Martins Unzer Colegio Etapa So Paulo SP 82,56
Rafael Tedeschi E. Pontes Barone C. Integrado Objetivo Araatuba SP 82,53
Narelli de Paiva Narciso 7 de Setembro Fortaleza CE 81,80
Andr Pozzobon Capeletti Politcnico da UFSM Santa Maria RS 81,32
Pedro Henrique Fonseca Duque IFRJ Rio de Janeiro RJ 81,18
Yuhz Uchigasaki Breyer Militar Braslia DF 80,31
Giancarlo Ferrigno Poli Ide Alves Colegio Bandeirantes So Paulo SP 79,83
Lucas Rebelo Vieira da Silva GGE Recife PE 79,79
Ana Letcia De Lima Dos Santos Positivo Ang. Sampaio Curitiba PR 79,51
>> Olimpada Brasileira de Qumica - 2013
67
Resultados
Lucas Henrique de Mendona Maia Olimpo Braslia DF 79,14
Luis Guilherme Gomes Aguiar Col. Ponto de Ensino Rio de Janeiro RJ 78,62
Paula Krein Colegio So Luis So Paulo SP 78,47
B R O N Z E
NOME ESCOLA CIDADE CIDADE NOTA
Ana Luisa Pereira Lima So Domingos Vitria ES 76,17
Joo Lucas Fernandes dos Santos GGE Recife PE 76,07
Gustavo Scarparo Pandolfo CIMAN Braslia DF 75,35
Rodrigo Reis Silva Sto. Antonio de Jesus Sto. Antonio de Jesus BA 75,31
Mateus Caracas Veras Farias Brito Caucaia CE 74,36
Marcelo dos Santos Bandeira Filho Dom Barreto Teresina PI 73,82
Sofa de Carvalho Britto Sousa Marista Dom Silvrio Belo Horizonte MG 73,67
Joo Luiz Vieira Maciel Borges Ideal Militar Belm PA 73,40
Andr Bubna Mirayama CIMAN Braslia DF 72,85
Murilo Leo Pereira Ideal Militar Belm PA 72,80
Fernanda Carneiro de C. Sacramento Militar Salvador BA 72,78
Fausto Daniel Mendes Morel Salesiano Dom Bosco Parnamirim RN 72,49
Nathan de Souza Mateus IFRJ Rio de Janeiro RJ 72,31
Camila Mendes Pontes Diocesano Caruaru PE 71,83
Manoel Maurcio Neves Gumes Anchieta Salvador BA 71,74
Estefani Tavares Jansen IF Sul-Rio-Grandense Pelotas RS 71,60
Rodrigo Rianelly de M. Evangelista Olimpo Braslia DF 71,53
Emilly Neves Souza Leonardo da Vinci Vitria ES 71,49
Jessica Henriques de Campos Militar Braslia DF 70,31
Juliana Sales do Nascimento Master Fortaleza CE 70,26
Gustavo Yuji Sakai Marista Cascavel PR 70,16
Yasmin Thaise Lisba Da Veiga IFPA Belm PA 69,83
Gabriel Sena Galvo Militar Braslia DF 69,51
Joo Teodoro Soares de Paula Olimpo Braslia DF 69,50
Livia Rodrigues de Araujo Farias Brito Fortaleza CE 69,12
Eder Leandro da Silva Dantas Overdose Natal RN 69,06
Thiago Edi Landim Lopes Antonio Vieira Salvador BA 68,81
Meno Honrosa e Demais Classifcados
Ver lista em www.obquimica.org
Programa Nacional Olimpadas de Qumica <<
68
Processo Seletivo
OLIMPADA BRASILEIRA DE QUMICA
2012 FASE IV
PROVA EXPERIMENTAL EM VDEO
Vdeoexibidoem26.01.2013,s14:00horas
Caro estudante,
Este exame de cunho experimental tem por fnalidade selecionar os 15 (quin-
ze) estudantes que participaro do Curso de Aprofundamento e Excelncia (Fase
V), que ser ministrado pela Universidade Federal do Piau (UFPI), para a futura
escolha dos representantes do Brasil nas olimpadas internacionais de Qumica.
Voc dispe de 3 (trs) horas para ver o vdeo e responder s questes deste exame.
I N S T R U E S
1. A prova contm 10 (dez) questes, que abrangem os 8 (oito) experimentos
contidos no vdeo.
2. Veja atentamente, no televisor ou monitor, as imagens do flme que contm
os fundamentos deste exame.
3. Seu coordenador, inicialmente, exibir a gravao completa do exame e, a
seguir, apresentar cada experimento separadamente. Caso seja necessrio,
ele repassar as imagens, at esclarecer as suas dvidas.
4. Leia as perguntas relativas a cada experimento, constantes nesta folha, e es-
creva as respostas nas folhas ofciais de respostas, nos espaos destinados a
cada questo.
5. Os resultados desse exame sero encaminhados para o seu coordenador (e
tambm diretamente para voc, caso tenha e-mail). Veja o resultado, tam-
bm, na internet em www.obquimica.org (clique em notcias).
>> Olimpada Brasileira de Qumica - 2013
69
OBQ-2012 Fase IV
QUESTIONRIO
QUESTO 1 (EXPERIMENTO 1)
Utilizando equaes de reaes qumicas, explique a mudana da colorao observada no bquer. Ver vdeo.
QUESTO 2 (EXPERIMENTO 2)
Utilizando equaes de reaes qumicas, explique a mudana da colorao observada nas superfcies das
moedas.
QUESTO 3 (EXPERIMENTO 3)
Considerando que em cada erlenmeyer foram adicionados 4,0 g de bicarbonato de sdio, discuta se a utilizao
de 65 mL de vinagre acarretaria na produo de um volume de gs maior do que o verifcado no balo de
tonalidade dourada.
QUESTO 4 (EXPERIMENTO 4)
Com base no fator de reteno, avalie se o eluente utilizado foi adequado para a anlise das amostras.
QUESTO 5 (EXPERIMENTO 4)
Discuta se esse eluente adequado para uma separao entre os componentes da amostra A com os da amos-
tra B em uma coluna de cromatografa.
QUESTO 6 (EXPERIMENTO 5)
Explique a variao de volume observada no balo de maior capacidade.
QUESTO 7 (EXPERIMENTO 6)
Indique o nome e a estrutura qumica do gs produzido.
QUESTO 8 (EXPERIMENTO 7)
De acordo com os resultados obtidos nessa anlise, avalie se possvel afrmar que existe pelo menos 400 mg
de cido acetilsaliclico no comprimido utilizado.
QUESTO 9 (EXPERIMENTO 8)
Escreva a equao geral da reao qumica que ocorre no bquer, indicando o nome do slido produzido.
QUESTO 10 (EXPERIMENTO 8)
Aps a reao, que procedimento deveria ser utilizado para a obteno de cristais maiores.
Programa Nacional Olimpadas de Qumica <<
70
Processo Seletivo
Olimpada Brasileira de Qumica 2012 Fase V
Curso de Aprofundamento e Excelncia em Qumica
Os estudantes classifcados na Fase IV da Olimpada participaram do Curso de
Aprofundamento e Excelncia em Qumica ministrado por docentes do Progra-
ma de Ps-graduao em Qumica da Universidade Federal do Piau. Durante
15 dias tiveram oportunidade de aperfeioar seus embasamentos tericos e se
aprofundar nos temas sugeridos pelos organizadores da 45
th
International Che-
mistry Olympiad, realizada em Moscou. A cada ano, os organizadores no pas
sede da Olimpada Internacional divulgam o tema central do certame e os as-
suntos defnidos de maior interesse na concepo do organizador.
A Quinzena Olmpica tem por fnalidade trabalhar esses conceitos, oferecer ativi-
dades culturais, mostrar como os grupos de pesquisa da universidade geram novos
conhecimentos e, por fm, promover a integrao desses jovens talentosos. Eles
almejam conquistar uma das vagas na equipe que representar o pas, ao tempo
em que se preparam para obter bons resultados nas olimpadas internacionais.
ReitordaUFPI,Prof.ArimateiaLopes,dboasvindasaosparticipantesnaaberturadaQuinzena
Olmpica-2013.
>> Olimpada Brasileira de Qumica - 2013
71
OBQ-2012 Fase V
Durante a Quinzena Olmpica so debatidos os temas listados no programa das
olimpadas internacionais, assuntos esses pouco abordados nas escolas de ensi-
no mdio e, assim, precisam ser discutidos com profundidade por especialistas
da universidade promotora do curso.
Aps a fnalizao da Quinzena Olmpica, os participantes retornaram aos seus
lugares de origem e aguardaram a realizao da etapa seguinte ocorrida, 30 dias
depois, com a aplicao dos exames da Fase VI da Olimpada.
Grupo de estudantes participantes da Quinzena Olmpica. Da esquerda para a direita: Nicholas
Lima,PedroDuque,ThiagoViana,NathanMateus,ThiagoRios,RafaelBarone,LiaDomingues,
LusValle,NathrciaMota,LviaArajo,VitriaMedeiros,MauroclioRochaeJoaquimLandim.
Sentados,FranciscoMarkanePedrotalo.
Programa Nacional Olimpadas de Qumica <<
72
Processo Seletivo
Olmpiada Brasileira de Qumica 2012 FASE VI
06.04.2013
Questo 1 (18 pontos)
Os complexos de nquel normalmente se apresentam no estado de oxidao +2.
Os amino complexos de nquel so preparados usando-se um sal de nquel +2 e
uma soluo da amina em questo.
Uma soluo aquosa de um sal de nquel +2 foi misturada com uma soluo
concentrada de amnia + soluo amoniacal (NH
4
Cl + NH
4
OH). Ao se adicionar a
soluo concentrada de amnia soluo inicial do sal de nquel +2 (complexo
A) essa mudou de cor (complexo B).
a) Escreva a frmula do complexo A e do complexo B e a equao de obteno
do complexo B.
b) Explique por que o complexo A mudou de cor.
c) Os espectros de absoro dos complexos A e B so mostrados na Figura 1.
Quais so as cores absorvidas e observadas dos complexos A e B?
d) Considerando que foram usados 5 g do composto A e que o mesmo possui
12% de impureza, que massa do composto B poder ser obtida?
e) 3 mL de etilenodiamina (NH
2
CH
2
CH
2
NH
2
) a 98% em massa foram adicionadas
a uma soluo contendo 3 g em 20 mL do composto B. A mistura foi evapo-
rada at se obter uma volume de 10 mL e ento mais uma gota da soluo
de etilenodiamina foi adicionada. Essa soluo foi resfriada e cristais foram
obtidos. Sabendo-se que foram obtidos 3,5 g do composto C, qual foi o rendi-
mento da reao?
f) Qual a cor absorvida e a cor observada para o complexo C.
g) Escreva a equao balanceada de obteno do complexo C, bem como seu
nome.
h) Os complexos A, B e C, podem apresentar algum tipo de isomeria? Se posi-
tivo, que tipo de isomeria eles podero apresentar?
i) A fgura 3 mostra os espectros de IV dos complexos B e C. Quais as principais
diferenas que tais espectros apresentam? Justifque sua resposta.
>> Olimpada Brasileira de Qumica - 2013
73
OBQ-2012 Fase VI
2

Figura 1 Espectro de Absoro dos Complexos A, B e C

Figura 2 Disco de Newton


Figura 3 Espectro de Infravermelho dos complexos B e C
Programa Nacional Olimpadas de Qumica <<
74
Questo 2 (12 pontos)
A Termodinmica uma subrea do conhecimento que fundamenta grande
parte dos conhecimentos estudados na Qumica. Por exemplo, as variaes
de energia envolvidas nas transformaes da matria podem ser determina-
das por meio das trs leis da Termodinmica, possibilitando, assim, uma maior
compreenso de tais processos. Sendo de fundamental importncia para a com-
preenso dos processos qumicos, imprescindvel que se tenha conhecimen-
tos bsicos sobre as leis da Termodinmica, para dar continuidade aos estudos
dentro da rea de Qumica. Desta forma, responda corretamente as seguintes
questes:
a) Baseado nas informaes apresentadas abaixo, determine a variao de en-
talpia, a 200C, da reao:
CH
3
COOH (g) + 2 O
2
(g) 2 CO
2
(g) + 2 H
2
O (g)
CH
3
COOH (l) + 2 O
2
(g) 2 CO
2
(g) + 2 H
2
O (l) H = - 871,5 kJ/mol (25 C)
H
2
O (l) H
2
O (g) H = + 44,0 kJ/mol (25 C)
CH
3
COOH (l) CH
3
COOH (g) H = + 30,2 kJ/mol (25 C)
Substncia CH
3
COOH (g) O
2
(g) CO
2
(g) H
2
O (l) H
2
O (g)
Cp/R 14,9 3,53 4,46 9,055 4,038
b) Calcule a variao de entropia total quando 80 g de gua (Cp = 75,5 J/K.mol),
a 70 C, so despejados sobre 100 g de gua, a 5 C, num vaso termicamente
isolado.
c) O tetrxido de nitrognio est 25% dissociado a 30 C e 1 atm no equilbrio:
N
2
O
4
(g) 2 NO
2
(g). Baseado nesta informao e sabendo que H = + 57,2
kJ/mol no intervalo de temperatura considerado, determine a constante de
equilbrio (Kp) da reao a 90 C.
Processo Seletivo
>> Olimpada Brasileira de Qumica - 2013
75
OBQ-2012 Fase VI
Questo 3 (16 pontos)
Para colocar o sistema de replicao de E. coli (4,6 milhes de pares de bases)
em escala humana, vamos imaginar que o DNA-B (3,4 /par de base) de 20 de
dimetro foi ampliado a 1 m de dimetro. Se tudo foi expandido proporcional-
mente, cada DNA polimerase holoenzima III teria aproximadamente o tamanho
de um caminho.
Em um sistema to expandido, responda:
a) Qual a velocidade que cada replissoma se move?
b) Qual a distncia que cada replissoma ir percorrer durante um ciclo de rep-
licao completa?
c) Qual ser o comprimento de um fragmento de Okazaki (1000 a 2000 pares de
bases)?
d) Qual ser a distncia mdia percorrida por um replissoma (um erro a cada 10
milhes de bases replicadas corretamente) a cada erro cometido? Fornea
suas respostas em km/h e em km.
Dado: Velocidade do replissoma = 1000 pares de bases/segundo.
Questo 4 (16 pontos)
Uma amostra de soro fsiolgico (NaCl 0,9% m/v em gua) est sendo inves-
tigada, sob a suspeita de contaminao com KNO
3
. Enviada ao laboratrio de
controle de qualidade, o analista no encontrou sequer traos de potssio ou
on nitrato. Uma alquota de 20 mL da amostra foi tratada com um excesso con-
fvel de AgNO
3
, precipitando todo o Cl- (do NaCl) na forma de cloreto de prata
slido. Com base no exposto responda:
a) Qual deve ser a massa de AgCl produzido na reao, para que a suspeita de
contaminao seja considerada improcedente?
b) Supondo que a formulao indicada no rtulo (0,9%) esteja correta, qual se-
ria o volume mnimo de AgNO
3
0,10 mol/L, necessrio para que o resultado
encontrado esteja correto?
c) Que titulante padro deve ser usado na determinao do excesso de on prata?
Programa Nacional Olimpadas de Qumica <<
76
Questo 5 (20 pontos)
a) 1,2-diis so oxidados em cetonas e/ou aldedos por cido peridico (HIO
4
)
quando reage com o diol para formar um intermedirio cclico. A reao
ocorre porque o iodo est num estado de oxidao altamente positivo (7+) de
modo que facilmente aceita eltrons. Quando o carbono que est ligado a um
grupo OH est tambm ligado a dois grupos R, o produto ser uma cetona. Se
o carbono est ligado a um R e um H, o produto ser um aldedo. Esta reao
de oxidao, onde o reagente corta a molcula em duas partes, chamada de
uma clivagem oxidativa.
Um alceno (X) tratado com OsO
4
seguido por H
2
O
2
. Quando o diol resultante
tratado com HIO
4
, o nico produto obtido uma cetona com a frmula molecu-
lar C
6
H
10
O. Qual a estrutura do alceno (X)?
b) Quando pirrol adicionado a uma soluo diluda de D
2
SO
4
em D
2
O, o 2-deu-
teriopirrol formado. Propor um mecanismo para explicar a formao deste
composto (2- deuteriopirrol).
c) Desenhar um segmento de 4 monmero do polmero formado a partir de
polimerizao catinica de 3,3 dimetiloxaciclobutano com BF
3
.
d) Quando o fenol tratado com Br
2
uma mistura de monobromo, dibromo e
tribromophenol so obtidos. Faa o mecanismo da reao de sntese que
possa obter apenas ortobromofenol.
e) Buchwald e colaboradores utilizaram o paldio para catalisar acoplamento
de hidrazonas com brometos de arila formando N-arilhidrazonas como na
reao de indolizao de Fischer formando (A), conforme a equao qumica
abaixo. A subsequente hidrlise de A na presena uma cetona, em condies
cidas, leva ao indol. D as frmulas estruturais de A e B.
5
e) Buchwald e colaboradores utilizaram o paldio para catalisar acoplamento de hidrazonas
com brometos de arila formando N-arilhidrazonas como na reao de indolizao de Fischer
formando (A), conforme a equao qumica abaixo. A subsequente hidrlise de A na
presena uma cetona, em condies cidas, leva ao indol. D as frmulas estruturais de A e
B.

N
NH
2
Ph Ph
+
Cl
Br
Pd(OAc)
2
1 mol%
BINAP tol
NaOBu 80 C
A
TsOH
EtOH e aquecimento
B
Ciclohexanona


Questo 6 (12 pontos)
O volume molar de um determinado slido 161,0 cm
3
/mol a 1,00 atm e sua temperatura de
fuso 350,75 K. O volume molar de um lquido nessa temperatura e presso 163,30
cm
3
/mol. Em 100 atm a temperatura de fuso muda para 351,26 K. Calcule:
a) A entalpia de fuso do slido.
b) A entropia de fuso do slido.
c) O ponto normal de fuso.
d) O ponto de fuso do solido sob a presso de 150 bar, assumindo que a densidade do
slido nessa condio aproximadamente 0,9 g/cm
3
e do lquido 1,12 g/cm
3
.

Questo 7 (10 pontos)
a) A 0,5 gramas de uma mistura que contm bicarbonato de sdio, carbonato de sdio e
cloreto de potssio foi adicionado 50 mL de gua at completa dissoluo da mistura. Aps
iniciar uma titulao da mistura com cido clordrico 0,115 molar, a soluo teve uma queda
acentuada de pH aps a adio de 11 mL do cido. A segunda queda acentuada do pH
ocorreu aps a adio de 35 mL do cido. Calcule a porcentagem de cada componente na
mistura analisada.
b) Uma amostra de 2.198 g contm uma mistura de XO e X
2
O
3
, requer 0,015 mol de K
2
Cr
2
O
7

para oxidar completamente a amostra para formar XO
4
-
e Cr
3+
. Se 0,0187 mol de XO
4
-

formado, qual a massa atmica de X?
Processo Seletivo
>> Olimpada Brasileira de Qumica - 2013
77
Questo 6 (12 pontos)
O volume molar de um determinado slido 161,0 cm
3
/mol a 1,00 atm e sua
temperatura de fuso 350,75 K. O volume molar de um lquido nessa tempera-
tura e presso 163,30 cm
3
/mol. Em 100 atm a temperatura de fuso muda para
351,26 K. Calcule:
a) A entalpia de fuso do slido.
b) A entropia de fuso do slido.
c) O ponto normal de fuso.
d) O ponto de fuso do solido sob a presso de 150 bar, assumindo que a den-
sidade do slido nessa condio aproximadamente 0,9 g/cm
3
e do lquido
1,12 g/cm
3
.
Questo 7 (10 pontos)
a) A 0,5 gramas de uma mistura que contm bicarbonato de sdio, carbonato
de sdio e cloreto de potssio foi adicionado 50 mL de gua at completa
dissoluo da mistura. Aps iniciar uma titulao da mistura com cido clo-
rdrico 0,115 molar, a soluo teve uma queda acentuada de pH aps a adio
de 11 mL do cido. A segunda queda acentuada do pH ocorreu aps a adio
de 35 mL do cido. Calcule a porcentagem de cada componente na mistura
analisada.
b) Uma amostra de 2.198 g contm uma mistura de XO e X
2
O
3
, requer 0,015 mol
de K
2
Cr
2
O
7
para oxidar completamente a amostra para formar XO
4

-
e Cr
3+
. Se
0,0187 mol de XO
4
-
formado, qual a massa atmica de X?
OBQ-2012 Fase VI
Programa Nacional Olimpadas de Qumica <<
78
Olimpada Brasileira de Qumica 2012 Fase VI
Resultado da prova Final - 2013
(Relao dos estudantes convocados para a Olimpada
Internacional de Qumica - Moscou)
Nome Escola Cidade Estado
Nota
Fase VI
Resultado
seletiva
Nicholas de Souza Costa Lima Ari de S Fortaleza CE 75,04 98,33
Lvia Rodrigues de Arajo Farias Brito Fortaleza CE 56,52 77,47
Mauroclio Rocha Pontes Filho Farias Brito Fortaleza CE 54,93 74,30
Nathrcia Castro Mota Master Fortaleza CE 45,80 69,84
Pedro talo Oliveira Gomes Ari de S Fortaleza CE 48,38 65,81
Thiago Silva Viana IFRJ R. de Janeiro RJ 42,76 63,21
Nathan de Souza Mateus IFRJ R. de Janeiro RJ 37,51 61,11
Luis Fernando Machado Poletti Valle Mater Amabilis Guarulhos SP 39,80 58,15
Pedro Henrique Fonseca Duque IFRJ R. de Janeiro RJ 34,76 57,16
Rafael Tedeschi E. Pontes Barone COC Araatuba SP 35,46 54,30
Thiago Matheus Santos Rios IFBA Salvador BA 31,96 54,01
Lia de Oliveira Domingues Farias Brito Fortaleza CE 31,35 51,16
Mateus Caracas Veras Farias Brito Fortaleza CE 30,17 50,05
Francisco Markan N. de Souza Filho Farias Brito Fortaleza CE 29,41 49,00
Joaquim Ivo Vasques D. Landim Farias Brito Fortaleza CE Faltou -
1. Os trs primeiros classifcados foram convocados para representar o Bra-
sil na olimpada internacional que aconteceu em Moscou. A quarta vaga,
conforme regulamento, neste ano, foi ocupada por Vitria Nunes Medeiros,
convocada hors concours.
2. Conforme o art. 3 do Regulamento da OBQ, o quarto e o quinto colocados
foram convocados para integrar a equipe brasileira na olimpada Iberoame-
ricana de qumica que ocorreu na Bolvia. Todos eles participaram de treina-
mento laboratorial, na 1 semana de julho, na UNICAMP.
Resultados Final
>> Olimpada Brasileira de Qumica - 2013
79
Treinamento em laboratrio na UNICAMP
Estudantes selecionados para representar o Brasil na 45
th
International Che-
mistry Olympiad (Rssia) e na 18. Olimpada Ibero-americana de Qumica (Bo-
lvia) recebem treinamento laboratorial no Instituto de Qumica da UNICAMP.
A parceria estabelecida entre o Programa Nacional Olimpadas de Qumica e o
Instituto de Qumica da UNICAMP tem por objetivo suprir as defcincias que os
estudantes do ensino mdio tm no manuseio de instrumentos de laboratrio
qumico. Ela destinada exclusivamente aos alunos classifcados na Fase VI da
olimpada (ltima etapa classifcatria), aqueles que representaro o Brasil nas
duas olimpadas internacionais no ano em curso.
Tanto da IChO quanto na OIAQ os exames constam de avaliaes tericas e
prticas em laboratrio, da a necessidade de os estudantes passarem por um
perodo de treinamento em laboratrio. Neste ano, o curso foi ministrado em
Campinas, no perodo de 01 a 05 de julho de 2013 sob a orientao do Prof. Ed-
valdo Sabadini, e no teve nenhum carter avaliativo.
Treinamento
Programa Nacional Olimpadas de Qumica <<
80
Exame Terico
45
th
International
Chemistry Olympiad
Face your challenge,
Be smart
Exame terico
20 de JULHO de 2013
MOSCOU, R SSIA
R E G R A S G E R A I S
Escreva seu nome e cdigo em cada pgina.
Voc tem 5 h para realizar o exame. Se voc no atender o comando STOP
fcar com zero ponto neste exame.
Se voc precisar de papel para rascunho, use o verso das folhas de sua prova.
No ser dada pontuao por esses rascunhos.
Escreva todas as respostas e clculos dentro dos boxes designados. Desenvol-
va a resoluo de seu problema onde for necessrio.
Use somente a caneta e a calculadora fornecidas.
Este caderno contm 43 pginas, incluindo os boxes para resposta, pgina
da capa e a Tabela Peridica.
A verso ofcial em Ingls estar disponvel, devendo ser solicitada somente
para tirar dvidas.
Se tiver necessidade de ir ao banheiro levante a sua mo. Voc ser acom-
panhado at l.
Aps o sinal de STOP, coloque o caderno de exame no envelope (no o la-
cre), deixe-o na mesa. No saia da sala sem permisso.
>> Olimpada Brasileira de Qumica - 2013
81
45
th
IChO
Constantes fsicas, Unidades, Frmulas e Equaes
Constante de Avogadro
N
A
= 6,0221 x 10
23
mol
1
Constante Universal dos gases R = 8,3145 JK
1
mol
1
Velocidade da luz c = 2,9979 x 108 ms1
Constante de Planck h = 6,6261 x 10
34
Js
Presso padro p
o
= 1 bar = 105 Pa = 750 mm Hg
Presso atmosfrica 1 atm = 1,013 x 10
5
Pa = 760 mm Hg
Zero na escala Celsius 273,15 K
1 nanmetro (nm) = 10
9
m
1 eltron volt (eV) = 1,602210
19
J = 96485 Jmol
1
Relao entre energia da luz e comprimento de onda (l)
E = hc / l
Energia de um mol de ftons
E = hcNA / l
Energia de Gibbs
G = H TS
Relao entre constante de equilbrio e energia padro de
Gibbs

3

- Constantes fsicas, Unidades, Frmulas e Equaes

Constante de Avogadro
N
A
= 6,0221 10
23
mol
1

Constante Universal dos
gases
R = 8,3145 JK
1
mol
1

Velocidade da luz
c = 2,9979 10
8
ms
1

Constante de Planck
h = 6,6261 10
34
Js

Presso padro
p = 1 bar = 10
5
Pa = 750 mm Hg
Presso atmosfrica
1 atm = 1,013 10
5
Pa = 760 mm Hg
Zero na escala Celsius 273,15 K

1 nanmetro (nm) = 10
9
m
1 eltron volt (eV) = 1,602210
19
J = 96485 Jmol
1

Relao entre energia da luz e comprimento de
onda ()
E = hc /
Energia de um mol de ftons
E = hcNA /
Energia de Gibbs G = H TS
Relao entre constante de equilbrio e energia
padro de Gibbs
= exp
G
K
RT
| |
A
|
|
\ .

Relao entre energia padro de Gibbs e fem
= G nFE A

Equao de Clapeyron para transio de fases =
dp H
dT T V
A
A

Equao de Clausius-Clapeyron para transio de
fases envolvendo vapor
2
1 1 2
1 1
ln =
p H
p R T T
| |
A

|
\ .

Dependncia da energia de Gibbs com a
concentrao ou presso da reao
prod
reag
= ln
a
G G RT
a
A A +
,
a = c / (1 mol/l) for the substances in
solution, a = p / (1 bar) for gases
Volume da esfera de raio R
3
4
3
V R = t

rea da superfcie de uma esfera de raio R S = 4tR2
Presso hidrosttica p = gh
Relao entre energia padro de Gibbs e fem

3

- Constantes fsicas, Unidades, Frmulas e Equaes

Constante de Avogadro
N
A
= 6,0221 10
23
mol
1

Constante Universal dos
gases
R = 8,3145 JK
1
mol
1

Velocidade da luz
c = 2,9979 10
8
ms
1

Constante de Planck
h = 6,6261 10
34
Js

Presso padro
p = 1 bar = 10
5
Pa = 750 mm Hg
Presso atmosfrica
1 atm = 1,013 10
5
Pa = 760 mm Hg
Zero na escala Celsius 273,15 K

1 nanmetro (nm) = 10
9
m
1 eltron volt (eV) = 1,602210
19
J = 96485 Jmol
1

Relao entre energia da luz e comprimento de
onda ()
E = hc /
Energia de um mol de ftons
E = hcNA /
Energia de Gibbs G = H TS
Relao entre constante de equilbrio e energia
padro de Gibbs
= exp
G
K
RT
| |
A
|
|
\ .

Relao entre energia padro de Gibbs e fem
= G nFE A

Equao de Clapeyron para transio de fases =
dp H
dT T V
A
A

Equao de Clausius-Clapeyron para transio de
fases envolvendo vapor
2
1 1 2
1 1
ln =
p H
p R T T
| |
A

|
\ .

Dependncia da energia de Gibbs com a
concentrao ou presso da reao
prod
reag
= ln
a
G G RT
a
A A +
,
a = c / (1 mol/l) for the substances in
solution, a = p / (1 bar) for gases
Volume da esfera de raio R
3
4
3
V R = t

rea da superfcie de uma esfera de raio R S = 4tR2
Presso hidrosttica p = gh
Equao de Clapeyron para transio de fases

3

- Constantes fsicas, Unidades, Frmulas e Equaes

Constante de Avogadro
N
A
= 6,0221 10
23
mol
1

Constante Universal dos
gases
R = 8,3145 JK
1
mol
1

Velocidade da luz
c = 2,9979 10
8
ms
1

Constante de Planck
h = 6,6261 10
34
Js

Presso padro
p = 1 bar = 10
5
Pa = 750 mm Hg
Presso atmosfrica
1 atm = 1,013 10
5
Pa = 760 mm Hg
Zero na escala Celsius 273,15 K

1 nanmetro (nm) = 10
9
m
1 eltron volt (eV) = 1,602210
19
J = 96485 Jmol
1

Relao entre energia da luz e comprimento de
onda ()
E = hc /
Energia de um mol de ftons
E = hcNA /
Energia de Gibbs G = H TS
Relao entre constante de equilbrio e energia
padro de Gibbs
= exp
G
K
RT
| |
A
|
|
\ .

Relao entre energia padro de Gibbs e fem
= G nFE A

Equao de Clapeyron para transio de fases =
dp H
dT T V
A
A

Equao de Clausius-Clapeyron para transio de
fases envolvendo vapor
2
1 1 2
1 1
ln =
p H
p R T T
| |
A

|
\ .

Dependncia da energia de Gibbs com a
concentrao ou presso da reao
prod
reag
= ln
a
G G RT
a
A A +
,
a = c / (1 mol/l) for the substances in
solution, a = p / (1 bar) for gases
Volume da esfera de raio R
3
4
3
V R = t

rea da superfcie de uma esfera de raio R S = 4tR2
Presso hidrosttica p = gh
Equao de Clausius-Clapeyron para transio de fases en-
volvendo vapor
3

- Constantes fsicas, Unidades, Frmulas e Equaes

Constante de Avogadro
N
A
= 6,0221 10
23
mol
1

Constante Universal dos
gases
R = 8,3145 JK
1
mol
1

Velocidade da luz
c = 2,9979 10
8
ms
1

Constante de Planck
h = 6,6261 10
34
Js

Presso padro
p = 1 bar = 10
5
Pa = 750 mm Hg
Presso atmosfrica
1 atm = 1,013 10
5
Pa = 760 mm Hg
Zero na escala Celsius 273,15 K

1 nanmetro (nm) = 10
9
m
1 eltron volt (eV) = 1,602210
19
J = 96485 Jmol
1

Relao entre energia da luz e comprimento de
onda ()
E = hc /
Energia de um mol de ftons
E = hcNA /
Energia de Gibbs G = H TS
Relao entre constante de equilbrio e energia
padro de Gibbs
= exp
G
K
RT
| |
A
|
|
\ .

Relao entre energia padro de Gibbs e fem
= G nFE A

Equao de Clapeyron para transio de fases =
dp H
dT T V
A
A

Equao de Clausius-Clapeyron para transio de
fases envolvendo vapor
2
1 1 2
1 1
ln =
p H
p R T T
| |
A

|
\ .

Dependncia da energia de Gibbs com a
concentrao ou presso da reao
prod
reag
= ln
a
G G RT
a
A A +
,
a = c / (1 mol/l) for the substances in
solution, a = p / (1 bar) for gases
Volume da esfera de raio R
3
4
3
V R = t

rea da superfcie de uma esfera de raio R S = 4tR2
Presso hidrosttica p = gh
Dependncia da energia de Gibbs com a concentrao ou
presso da reao
3

- Constantes fsicas, Unidades, Frmulas e Equaes

Constante de Avogadro
N
A
= 6,0221 10
23
mol
1

Constante Universal dos
gases
R = 8,3145 JK
1
mol
1

Velocidade da luz
c = 2,9979 10
8
ms
1

Constante de Planck
h = 6,6261 10
34
Js

Presso padro
p = 1 bar = 10
5
Pa = 750 mm Hg
Presso atmosfrica
1 atm = 1,013 10
5
Pa = 760 mm Hg
Zero na escala Celsius 273,15 K

1 nanmetro (nm) = 10
9
m
1 eltron volt (eV) = 1,602210
19
J = 96485 Jmol
1

Relao entre energia da luz e comprimento de
onda ()
E = hc /
Energia de um mol de ftons
E = hcNA /
Energia de Gibbs G = H TS
Relao entre constante de equilbrio e energia
padro de Gibbs
= exp
G
K
RT
| |
A
|
|
\ .

Relao entre energia padro de Gibbs e fem
= G nFE A

Equao de Clapeyron para transio de fases =
dp H
dT T V
A
A

Equao de Clausius-Clapeyron para transio de
fases envolvendo vapor
2
1 1 2
1 1
ln =
p H
p R T T
| |
A

|
\ .

Dependncia da energia de Gibbs com a
concentrao ou presso da reao
prod
reag
= ln
a
G G RT
a
A A +
,
a = c / (1 mol/l) for the substances in
solution, a = p / (1 bar) for gases
Volume da esfera de raio R
3
4
3
V R = t

rea da superfcie de uma esfera de raio R S = 4tR2
Presso hidrosttica p = gh
a = c / (1 mol/l) for the substances
in solution, a = p / (1 bar) for gases
Volume da esfera de raio R

3

- Constantes fsicas, Unidades, Frmulas e Equaes

Constante de Avogadro
N
A
= 6,0221 10
23
mol
1

Constante Universal dos
gases
R = 8,3145 JK
1
mol
1

Velocidade da luz
c = 2,9979 10
8
ms
1

Constante de Planck
h = 6,6261 10
34
Js

Presso padro
p = 1 bar = 10
5
Pa = 750 mm Hg
Presso atmosfrica
1 atm = 1,013 10
5
Pa = 760 mm Hg
Zero na escala Celsius 273,15 K

1 nanmetro (nm) = 10
9
m
1 eltron volt (eV) = 1,602210
19
J = 96485 Jmol
1

Relao entre energia da luz e comprimento de
onda ()
E = hc /
Energia de um mol de ftons
E = hcNA /
Energia de Gibbs G = H TS
Relao entre constante de equilbrio e energia
padro de Gibbs
= exp
G
K
RT
| |
A
|
|
\ .

Relao entre energia padro de Gibbs e fem
= G nFE A

Equao de Clapeyron para transio de fases =
dp H
dT T V
A
A

Equao de Clausius-Clapeyron para transio de
fases envolvendo vapor
2
1 1 2
1 1
ln =
p H
p R T T
| |
A

|
\ .

Dependncia da energia de Gibbs com a
concentrao ou presso da reao
prod
reag
= ln
a
G G RT
a
A A +
,
a = c / (1 mol/l) for the substances in
solution, a = p / (1 bar) for gases
Volume da esfera de raio R
3
4
3
V R = t

rea da superfcie de uma esfera de raio R S = 4tR2
Presso hidrosttica p = gh
rea da superfcie de uma esfera de raio R
S = 4pR2
Presso hidrosttica
p = rgh
Programa Nacional Olimpadas de Qumica <<
82
PROBLEMA 1. CLATHRATE GUN (8 PONTOS)
Questo 1 2 3 4 5 6 Total
Pontos 2 1 3 5 6 2 19
The only gun that is able to kill all living people in one shot
No fundo dos oceanos e mares existe uma vasta reser-
va de metano na forma de compostos clatratos, cha-
mados hidratos de metano. Estas reservas podem ser
exploradas e servem como fonte de energia ou como
matria-prima para sntese orgnica. Entretanto, os
cientistas esto seriamente preocupados com a pos-
sibilidade da decomposio espontnea desses hidra-
tos, causada pelo aumento da temperatura do oceano. Acredita-se que, se uma
sufciente quantidade de metano for liberada para atmosfera, os oceanos tero
um rpido aumento de temperatura devido ao efeito estufa, acelerando, por
sua vez, a decomposio de clatratos. Devido a exploso resultante da mistura
metano-ar e/ou s mudanas na composio da atmosfera, todos os seres vivos
podero ser extintos. Este cenrio apocalptico chamado de clathrate gun.
Durante a decomposio de 1,00 g de hidrato de metano, com uma composio
fxa, a 25 C e presso atmosfrica (101,3 kPa), 205 ml de metano liberado.
1. Determine n (no necessariamente um nmero inteiro) na frmula do hidra-
to de metano, CH
4
.nH
2
O.
Clculos:
Resposta:
Exame Terico
>> Olimpada Brasileira de Qumica - 2013
83
O hidrato de metano real tem uma composio no estequiomtrica prximo
de CH
4
6H
2
O. presso atmosfrica, hidrato de metano decompe-se a -81C.
No entanto, a altas presses (por exemplo, no fundo do oceano) ele estvel a
temperaturas mais altas. A decomposio de hidrato de metano produz metano
gasoso e gua slida ou lquida, dependendo da temperatura.
2. Escreva a equao de decomposio de 1 mol de CH
4
6H
2
O produzindo gua
slida (gelo) H
2
O(s).
A entalpia do presente processo igual a 17,47 kJmol
-1
. Assuma que as entalpias
no dependem da temperatura e da presso, que a variao de volume aps a
decomposio do hidrato igual ao volume de metano liberado, e que o metano
um gs ideal.
3. Em que presso externa ocorre a decomposio do hidrato de metano em gs
metano e gelo, a -5 C?
Clculos:
Resposta:
4. Qual a possvel profundidade mnima de gua lquida pura em que os hidra-
tos de metano podem ser estveis?
272,9 K 273,15K 273,4 K
45
th
IChO
Programa Nacional Olimpadas de Qumica <<
84
Para responder a esta pergunta, voc deve primeiro deduzir qual a temperatura
mnima em que o hidrato de metano pode coexistir com gua lquida. Assinale
a resposta correta.
Clculos:
Resposta:
Reservas de grandes quantidades de hidrato de metano no fundo do lago Baikal,
o maior lago de gua doce da Rssia e do mundo, foram descobertas, em julho
de 2009, pela tripulao de um veculo de profunda submerso Mir-2. Durante
a subida, a partir da profundidade de 1.400 m, amostras de hidrato de metano
comearam a decompor-se a uma profundidade de 372 m.
5. Determine a temperatura no lago Baikal, a uma profundidade de 372 m. A
entalpia de fuso do gelo 6,01 kJmol
-1
.
Clculos:
Resposta:
A quantidade total de metano em hidratos de metano na Terra no menos que
510
11
tons.
6. De quantos graus poderia a temperatura da atmosfera da Terra aumentar, se
tal quantidade de metano fosse queimada ao reagir com o oxignio atmosf-
rico? A entalpia de combusto do metano -889 kJmol
-1
, a capacidade total
de calor da atmosfera da Terra de cerca de 41021 JK
-1
.
Clculos:
Resposta:
Exame Terico
>> Olimpada Brasileira de Qumica - 2013
85
PROBLEMA 2. BREAK DOWN FOTOSSNTESE
REAO DO HILL (7 PONTOS)
Questo 1 2
3 4
5 6 Total
a b c a b
Pontos 1 2 2 2 3.5 1 2 3 2.5 19
Na histria da pesquisa em break down fotossntese, existiram alguns experi-
mentos que acrescentaram muito ao nosso conhecimento sobre esse processo
to complexo. Um desses experimentos foi realizado em 1930 pelo bioqumico
ingls Robert Hill. Neste problema, consideraremos alguns de seus dados juntos
com dados de experimentos mais recentes.

1. Nas plantas, sob iluminao, dixido de carbono reduzido a carboidratos
(denominado como {CH
2
O}) e oxignio produzido. Escreva a equao geral
da fotossntese em plantas.
A maior parte da fotossntese acontece nos cloroplastos - organelas encontra-
dos nas clulas de plantas que contm clorofla substncia que absorve a luz.
Hill isolou cloroplastos das clulas, macerando as folhas em solues de sucrose.
As clulas livres de cloroplastos no produziram oxignio sob iluminao, nem
mesmo na presena de CO
2
. Entretanto, adicionando ferrioxalato de potssio,
K
3
[Fe(C
2
O
4
)
3
] (com excesso de oxalato de potssio) na suspenso de cloroplasto,
Hill observou a liberao de oxignio sob iluminao, mesmo sem CO
2
.
2. O experimento de Hill permitiu determinar a fonte de oxignio durante a fo-
tossntese. Escreva as frmulas do oxidante e do agente redutor na fotossn-
tese dentro das clulas das plantas e na clula livre de cloroplastos (reao
de Hill).
45
th
IChO
Programa Nacional Olimpadas de Qumica <<
86
Fotossntese natural Reao de Hill
Oxidante Agente redutor Oxidante Agente redutor
Hill mediu a quantidade de oxignio liberado, usando a hemoglobina do mscu-
lo (Hill denominou de Hb) o qual liga todo oxignio molecular na razo de 1:1
para formar HbO
2
. A concentrao inicial de Hb foi 0,6
.
10
4
M. Curvas cinticas
correspondentes s diferentes concentraes de ferrioxalato so mostradas na
fgura (a curva superior corresponde a 2,0
.
10
4
M).
AfraodahemoglobinaHbO
2
ligada(comrelaoaquantidadeinicialdeHb)
emfunodotempo.Ascruzes(X)indicamofnaldareao.
(Figure 2a da publicao original de Hills: R. Hill. Oxygen produced by isolated
chloroplasts.Proc.R.Soc.B,1939,v.127,pp.192-210)
3. a. A partir da fgura, estime a razo molar de Fe/O
2
no fnal da reao. No
leve em considerao o ferro do Hb.
b. Escreva a reao de Hill assumindo que ela procede com um alto rendi-
mento.
c. Usando a tabela de potenciais de eletrodo padro, determine a energia
de Gibbs da reao de Hill a T = 298 K, presso de oxignio 1 mmHg, pH = 8 e
Exame Terico
>> Olimpada Brasileira de Qumica - 2013
87
concentraes padro das outras espcies. Essa reao espontnea nessas
condies?
Nome:_________________________________________________Cdigo do Estudante: BRA-S___
9


A frao da hemoglobina HbO
2
ligada (com relao a quantidade inicial
de Hb) em funo do tempo. As cruzes (X) indicam o final da reao.
(Figure 2a da publicao original de Hills: R. Hill. Oxygen produced by
isolated chloroplasts. Proc. R. Soc. B, 1939, v. 127, pp. 192-210)

3. a. A partir da figura, estime a razo molar de Fe / O
2
no final da reao. No leve em
considerao o ferro do Hb.
b. Escreva a reao de Hill assumindo que ela procede com um alto rendimento.
c. Usando a tabela de potenciais de eletrodo padro, determine a energia de Gibbs da
reao de Hill a T = 298 K, presso de oxignio 1 mmHg, pH = 8 e concentraes padro das
outras espcies. Essa reao espontnea nessas condies?

Meia-reao
E, V
O
2
+ 4H
+
+ 4e 2H
2
O
+1,23
CO
2
+ 4H
+
+ 8e {CH
2
O} + H
2
O
0,01
Fe
3+
+ e Fe
2+
+0,77
Fe
3+
+ 3e Fe
0
0,04
[Fe(C
2
O
4
)
3
]
3
+ e [Fe(C
2
O
4
)
3
]
4
+0,05
[Fe(C
2
O
4
)
3
]
4
+ 2e Fe + 3C
2
O
4
2
0,59

a. Clculos








a. Clculos
n(Fe) / n(O
2
) =
b. Equao da reao:

c. Clculos

G =
A reao espontnea no espontnea
Atualmente, o nome reao de Hill denomina a oxidao fotoqumica da gua
por qualquer oxidante que no seja dixido de carbono o qual sintetizado pe-
las clulas das plantas ou cloroplastos isolados.
45
th
IChO
Programa Nacional Olimpadas de Qumica <<
88
Em outro experimento (1952), quinona em soluo cida foi usada como um
oxidante na reao de Hill iniciada por fashes de luz nas algas Chlorella. Dados
experimentais so mostrados na fgura. O volume de oxignio (em mm
3
, na tem-
peratura de 10
o
C e presso 740 mmHg) por um grama de clorofla por fash foi
determinado como uma funo da intensidade da luz para fotossntese natural
e para cloroplastos isolados. Foi encontrado que o rendimento mximo do oxi-
gnio o mesmo para a fotossntese natural e reao de Hill.

(Figure1from:H.Ehrmantraut,E.Rabinovitch.KineticsofHillreaction.Archives
ofBiochemistryandBiophysics,1952,v.38,pp.67-84)
4 a. Determine a ordem de reao da reao fotoqumica de Hill em relao a
intensidade de luz, em intensidades baixa e alta. Em cada caso escolha um
dos trs valores:
Ordem de reao:
Intensidade baixa Intensidade alta
0 1 2 0 1 2
Exame Terico
>> Olimpada Brasileira de Qumica - 2013
89
b. Quantas molculas de clorofla (Chl) participam na formao de uma mo-
lcula de oxignio no limite da reao saturada de Hill? (A massa molecular
da clorofla 900 Da).
Clculos:
n(Chl) / n(O
2
) =
O requisito quntico das reaes redox de luz defnido como o nmero mdio
de ftons de luz (no necessariamente inteiro), necessrio para a transferncia
de um eltron de um agente redutor para um oxidante. Os cloroplastos isolados
foram irradiados durante 2 horas com uma luz monocromtica (comprimento
de onda de 672 nm) com a energia de entrada 0,503 mJ/s, e o volume total de
oxignio formado foi de 47,6 mm3 (sob as mesmas condies da questo 4).
5. Calcular o requisito quntico para a reao de Hill.
Clculos:
Requisito quntico:
45
th
IChO
Programa Nacional Olimpadas de Qumica <<
90
6. Tente tirar concluses dos experimentos acima (questes 2-5). Para cada um
das seguintes sentenas escolha Sim ou No
Sim No
Na fotossntese natural, a oxidao da gua e a reduo do
CO2 so separadas no espao.
Nos cloroplastos, O
2
produzido a partir de CO
2
.
Oxidao da gua em cloroplastos requer iluminao de
luz.
A maior parte da clorofla nos cloroplastos participam di-
retamente na produo fotoqumica de O
2
.
Em cloroplastos isolados, cada fton absorvido leva
transferncia de um eltron.

PROBLEMA 3. REAO DE MEERWEIN - SCHMIDT -
PONNDORF - VERLEY (8 PONTOS)
Questo
1
2 3 4 Total
a b
Pontos 7 3 8.5 6 8 32.5
A reao de Schmidt-Meerwein-Ponndorf-Verley (MSPV) uma ferramenta til
para a reduo de compostos carbonlicos a lcoois. Essa reao consiste na re-
duo de compostos carbonlicos por lcoois de baixa massa molecular, na pre-
sena de alcxidos de alumnio ou outros metais:
Nome:_________________________________________________Cdigo do Estudante: BRA-S___
13

Problema 3. Reao de Meerwein-Schmidt-Ponndorf-Verley (8 pontos)

Questo 1 2 3 4 Total
a b
Pontos 7 3 8.5 6 8 32.5

A reao de Schmidt-Meerwein-Ponndorf-Verley (MSPV) uma ferramenta til para a
reduo de compostos carbonlicos a lcoois. Essa reao consiste na reduo de compostos
carbonlicos por lcoois de baixa massa molecular, na presena de alcxidos de alumnio ou
outros metais:
OH
R
1
R
2
O OH O
R
1
R
2
Al(OiPr)
3

(1)
O mecanismo dessa reao inclui a coordenao do composto carbonlico com o
alcxido de alumnio, a transferncia de hidreto na esfera interna do complexo e a subsequente
transalcoxilo. Ela pode ser esquematicamente representada como segue (para simplificar, a
transalcoxilao mostrada como um processo de uma s etapa):
Al
O
O O
O
R
1
R
2
O
R
1
R
2
Al
O
O O
H
O
R
1
R
2
Al
O
O
H
O
iPrOH
Al
O
O O
OH
R
1
R
2
O

(2)
A reao reversvel e o deslocamento do equilbrio para o produto desejado, requer
um excesso do agente redutor. Em alguns casos (por exemplo, no caso da reduo de aldedos
aromticos e cetonas) a constante de equilbrio to grande que a reao inversa pode ser
negligenciada.

A tabela abaixo contm as entropias e entalpias padres de formao de substncias lquidas a
298 K. Tambm so dados os pontos de ebulio das substncias a 1 bar.
Substncia
f
H
o
298
, kJ/mol S
o
298
, J/(molK) t
vap
,


Acetona 248,4 200,4 56
Isopropanol 318,1 180,6 82
Ciclohexanona 271,2 229,0 156
Ciclohexanol 348,2 203,4 161

1a. Calcule a razo mnima em massa, entre isopropanol:ciclohexanona, necessria para
alcanar um rendimento de 99% da reao a 298 K. Assuma que: a) a mistura de reao,
eventualmente, atinge o equilbrio e b) no h produtos presentes inicialmente.


O mecanismo dessa reao inclui a coordenao do composto carbonlico com o
alcxido de alumnio, a transferncia de hidreto na esfera interna do complexo
e a subsequente transalcoxilo. Ela pode ser esquematicamente representada
como segue (para simplifcar, a transalcoxilao mostrada como um processo
de uma s etapa):
Exame Terico
>> Olimpada Brasileira de Qumica - 2013
91
Nome:_________________________________________________Cdigo do Estudante: BRA-S___
13

Problema 3. Reao de Meerwein-Schmidt-Ponndorf-Verley (8 pontos)

Questo 1 2 3 4 Total
a b
Pontos 7 3 8.5 6 8 32.5

A reao de Schmidt-Meerwein-Ponndorf-Verley (MSPV) uma ferramenta til para a
reduo de compostos carbonlicos a lcoois. Essa reao consiste na reduo de compostos
carbonlicos por lcoois de baixa massa molecular, na presena de alcxidos de alumnio ou
outros metais:
OH
R
1
R
2
O OH O
R
1
R
2
Al(OiPr)
3

(1)
O mecanismo dessa reao inclui a coordenao do composto carbonlico com o
alcxido de alumnio, a transferncia de hidreto na esfera interna do complexo e a subsequente
transalcoxilo. Ela pode ser esquematicamente representada como segue (para simplificar, a
transalcoxilao mostrada como um processo de uma s etapa):
Al
O
O O
O
R
1
R
2
O
R
1
R
2
Al
O
O O
H
O
R
1
R
2
Al
O
O
H
O
iPrOH
Al
O
O O
OH
R
1
R
2
O

(2)
A reao reversvel e o deslocamento do equilbrio para o produto desejado, requer
um excesso do agente redutor. Em alguns casos (por exemplo, no caso da reduo de aldedos
aromticos e cetonas) a constante de equilbrio to grande que a reao inversa pode ser
negligenciada.

A tabela abaixo contm as entropias e entalpias padres de formao de substncias lquidas a
298 K. Tambm so dados os pontos de ebulio das substncias a 1 bar.
Substncia
f
H
o
298
, kJ/mol S
o
298
, J/(molK) t
vap
,


Acetona 248,4 200,4 56
Isopropanol 318,1 180,6 82
Ciclohexanona 271,2 229,0 156
Ciclohexanol 348,2 203,4 161

1a. Calcule a razo mnima em massa, entre isopropanol:ciclohexanona, necessria para
alcanar um rendimento de 99% da reao a 298 K. Assuma que: a) a mistura de reao,
eventualmente, atinge o equilbrio e b) no h produtos presentes inicialmente.


A reao reversvel e o deslocamento do equilbrio para o produto desejado,
requer um excesso do agente redutor. Em alguns casos (por exemplo, no caso da
reduo de aldedos aromticos e cetonas) a constante de equilbrio to gran-
de que a reao inversa pode ser negligenciada.
A tabela abaixo contm as entropias e entalpias padres de formao de subs-
tncias lquidas a 298 K. Tambm so dados os pontos de ebulio das substn-
cias a 1 bar.
Substncia f H
o
298
, kJ/mol S
o
298
, J/(molK) t
vap
,
o
C
Acetona 248,4 200,4 56
Isopropanol 318,1 180,6 82
Ciclohexanona 271,2 229,0 156
Ciclohexanol 348,2 203,4 161
1 a. Calcule a razo mnima em massa, entre isopropanol:ciclohexanona, ne-
cessria para alcanar um rendimento de 99% da reao a 298 K. Assuma
que: a) a mistura de reao, eventualmente, atinge o equilbrio e b) no h
produtos presentes inicialmente.
Clculos:
Resposta:
m(C
3
H
8
O) : m(C
6
H
1
0O) =
45
th
IChO
Programa Nacional Olimpadas de Qumica <<
92
1 b. Escolha a(s) forma(s) adequada(s) para aumentar o rendimento de ciclohe-
xanol.
Aviso:respostasassinaladaserroneamenteiroresultarempontosdepenalizao
Aumentar a temperatura at 50
o
C usando refuxo
Aumentar a temperatura at 60
o
C, evaporando (destilando) a acetona
Adicionar um pouco de etanol mistura reacional
Adicionar um pouco de etanal mistura reacional
2. Frequentemente, a etapa limitante da velocidade na reao MSPV a trans-
ferncia do hidreto ou a alcolise do alcxido aps a transferncia de hi-
dreto. Para estes dois casos, utilizando o mecanismo (2) acima, derive uma
expresso para a velocidade de reao como uma funo da concentrao
corrente de um composto carbonlico, de isopropanol e de catalisador. Em
ambos os casos, determinar as ordens de reao dos reagentes e do catalisa-
dor. Assuma que todos as etapas da reao, antes da etapa limitante, so r-
pidas e reversveis. Use aproximao do equilbrio, caso seja necessrio. Para
simplifcar use a seguinte notao: A para um composto carbonlico, B para
o isopropanol, C para o catalisador. Use para os intermedirios a notao que
desejar.
A etapa limitante da velocidade a transferncia de hidreto
Derivao:
r =
Resposta
Ordem para o composto carbonlico: _________________
Ordem para o isopropanol: _________________
Ordem para o catalisador: _________________
A etapa limitante da velocidade a transalcoxilao do alcoolato pelo isopropanol
Exame Terico
>> Olimpada Brasileira de Qumica - 2013
93
Derivao:
r =
Resposta
Ordem para o composto carbonlico: _________________
Ordem para o isopropanol: _________________
Ordem para o catalisador: _________________
A reao MSPV pode ser usada para se obter lcoois quirais, se um catalisador
quiral empregado. Por exemplo, Campbell et al. utilizou um catalisador base
de 2,2 -di-hidroxi-1, 1-binaphtyl (BINOL) quiral, que sintetizado in situ a par-
tir binaftol e trimetilalumnio:
Nome:_________________________________________________Cdigo do Estudante: BRA-S___
16

A etapa limitante da velocidade a transalcoxilao do alcoolato pelo isopropanol
Derivao:



















r =


Resposta
Ordem para o composto carbonlico: ________
Ordem para o isopropanol: ________
Ordem para o o catalisador: ________


A reao MSPV pode ser usada para se obter lcoois quirais, se um catalisador quiral
empregado. Por exemplo, Campbell et al. utilizou um catalisador base de 2,2 '-di-hidroxi-1,
1'-binaphtyl (BINOL) quiral, que sintetizado in situ a partir binaftol e trimetilalumnio:

O
O
Al O
OH
OH
Al(CH
3
)
3
O
O
Al
iPrOH
(BINOL)Al(OiPr)

(3)
A quiralidade do BINOL deve-se ao impedimento estrico da rotao da ligao
CC. Embora perfeitamente estvel temperatura ambiente, o BINOL pode race-
mizar quando aquecido.
45
th
IChO
Programa Nacional Olimpadas de Qumica <<
94
3. Qual dos fenis abaixo podem formar ( temperatura ambiente) enantime-
ros estveis, de modo que eles possam ser utilizados da mesma forma para
produzir um catalisador quiral?
Aviso:respostaassinaladaerroneamenteirresultarempontosdepenalizao
Substncia Pode ser usada Substncia Pode ser usada

Nome:_________________________________________________Cdigo do Estudante: BRA-S___
17

A quiralidade do BINOL deve-se ao impedimento estrico da rotao da ligao CC. Embora
perfeitamente estvel temperatura ambiente, o BINOL pode racemizar quando aquecido.

3. Qual dos fenis abaixo podem formar ( temperatura ambiente) enantimeros estveis, de
modo que eles possam ser utilizados da mesma forma para produzir um catalisador quiral?
Aviso: resposta assinalada erroneamente ir resultar em pontos de penalizao

Substncia Pode ser usada Substncia Pode ser usada
OH
OH
OCH
3
OCH
3



OH
OH



OH
OH
OCH
3
OCH
3



OCH
3
OH
HO
CH
3
O



OH
OH



OH
OH




4. O excesso enantiomrico, ee, utilizado para caracterizar a pureza enantiomrica de uma
substncia. Esta quantidade igual razo entre a diferena e a soma nas concentraes dos
enantimeros R e S:
[ ] [ ]
[ ] [ ]
R S
ee
R S


O excesso enantiomrico do ismero R puro 1, o ee da mistura racmica zero.

Quando se utiliza o enantiomericamente puro (BINOL) Al (OiPr) como um catalisador para a
reduo de -bromoacetofenona, o ee do produto igual a 81%. Qual ser o ee do produto, se
o ee do catalisador for igual a 50%? Apresente seu clculo com uma ilustrao ou derivao
da frmula final.
Nome:_________________________________________________Cdigo do Estudante: BRA-S___
17

A quiralidade do BINOL deve-se ao impedimento estrico da rotao da ligao CC. Embora
perfeitamente estvel temperatura ambiente, o BINOL pode racemizar quando aquecido.

3. Qual dos fenis abaixo podem formar ( temperatura ambiente) enantimeros estveis, de
modo que eles possam ser utilizados da mesma forma para produzir um catalisador quiral?
Aviso: resposta assinalada erroneamente ir resultar em pontos de penalizao

Substncia Pode ser usada Substncia Pode ser usada
OH
OH
OCH
3
OCH
3



OH
OH



OH
OH
OCH
3
OCH
3



OCH
3
OH
HO
CH
3
O



OH
OH



OH
OH




4. O excesso enantiomrico, ee, utilizado para caracterizar a pureza enantiomrica de uma
substncia. Esta quantidade igual razo entre a diferena e a soma nas concentraes dos
enantimeros R e S:
[ ] [ ]
[ ] [ ]
R S
ee
R S


O excesso enantiomrico do ismero R puro 1, o ee da mistura racmica zero.

Quando se utiliza o enantiomericamente puro (BINOL) Al (OiPr) como um catalisador para a
reduo de -bromoacetofenona, o ee do produto igual a 81%. Qual ser o ee do produto, se
o ee do catalisador for igual a 50%? Apresente seu clculo com uma ilustrao ou derivao
da frmula final.
Nome:_________________________________________________Cdigo do Estudante: BRA-S___
17

A quiralidade do BINOL deve-se ao impedimento estrico da rotao da ligao CC. Embora
perfeitamente estvel temperatura ambiente, o BINOL pode racemizar quando aquecido.

3. Qual dos fenis abaixo podem formar ( temperatura ambiente) enantimeros estveis, de
modo que eles possam ser utilizados da mesma forma para produzir um catalisador quiral?
Aviso: resposta assinalada erroneamente ir resultar em pontos de penalizao

Substncia Pode ser usada Substncia Pode ser usada
OH
OH
OCH
3
OCH
3



OH
OH



OH
OH
OCH
3
OCH
3



OCH
3
OH
HO
CH
3
O



OH
OH



OH
OH




4. O excesso enantiomrico, ee, utilizado para caracterizar a pureza enantiomrica de uma
substncia. Esta quantidade igual razo entre a diferena e a soma nas concentraes dos
enantimeros R e S:
[ ] [ ]
[ ] [ ]
R S
ee
R S


O excesso enantiomrico do ismero R puro 1, o ee da mistura racmica zero.

Quando se utiliza o enantiomericamente puro (BINOL) Al (OiPr) como um catalisador para a
reduo de -bromoacetofenona, o ee do produto igual a 81%. Qual ser o ee do produto, se
o ee do catalisador for igual a 50%? Apresente seu clculo com uma ilustrao ou derivao
da frmula final.
Nome:_________________________________________________Cdigo do Estudante: BRA-S___
17

A quiralidade do BINOL deve-se ao impedimento estrico da rotao da ligao CC. Embora
perfeitamente estvel temperatura ambiente, o BINOL pode racemizar quando aquecido.

3. Qual dos fenis abaixo podem formar ( temperatura ambiente) enantimeros estveis, de
modo que eles possam ser utilizados da mesma forma para produzir um catalisador quiral?
Aviso: resposta assinalada erroneamente ir resultar em pontos de penalizao

Substncia Pode ser usada Substncia Pode ser usada
OH
OH
OCH
3
OCH
3



OH
OH



OH
OH
OCH
3
OCH
3



OCH
3
OH
HO
CH
3
O



OH
OH



OH
OH




4. O excesso enantiomrico, ee, utilizado para caracterizar a pureza enantiomrica de uma
substncia. Esta quantidade igual razo entre a diferena e a soma nas concentraes dos
enantimeros R e S:
[ ] [ ]
[ ] [ ]
R S
ee
R S


O excesso enantiomrico do ismero R puro 1, o ee da mistura racmica zero.

Quando se utiliza o enantiomericamente puro (BINOL) Al (OiPr) como um catalisador para a
reduo de -bromoacetofenona, o ee do produto igual a 81%. Qual ser o ee do produto, se
o ee do catalisador for igual a 50%? Apresente seu clculo com uma ilustrao ou derivao
da frmula final.
Nome:_________________________________________________Cdigo do Estudante: BRA-S___
17

A quiralidade do BINOL deve-se ao impedimento estrico da rotao da ligao CC. Embora
perfeitamente estvel temperatura ambiente, o BINOL pode racemizar quando aquecido.

3. Qual dos fenis abaixo podem formar ( temperatura ambiente) enantimeros estveis, de
modo que eles possam ser utilizados da mesma forma para produzir um catalisador quiral?
Aviso: resposta assinalada erroneamente ir resultar em pontos de penalizao

Substncia Pode ser usada Substncia Pode ser usada
OH
OH
OCH
3
OCH
3



OH
OH



OH
OH
OCH
3
OCH
3



OCH
3
OH
HO
CH
3
O



OH
OH



OH
OH




4. O excesso enantiomrico, ee, utilizado para caracterizar a pureza enantiomrica de uma
substncia. Esta quantidade igual razo entre a diferena e a soma nas concentraes dos
enantimeros R e S:
[ ] [ ]
[ ] [ ]
R S
ee
R S


O excesso enantiomrico do ismero R puro 1, o ee da mistura racmica zero.

Quando se utiliza o enantiomericamente puro (BINOL) Al (OiPr) como um catalisador para a
reduo de -bromoacetofenona, o ee do produto igual a 81%. Qual ser o ee do produto, se
o ee do catalisador for igual a 50%? Apresente seu clculo com uma ilustrao ou derivao
da frmula final.

Nome:_________________________________________________Cdigo do Estudante: BRA-S___
17

A quiralidade do BINOL deve-se ao impedimento estrico da rotao da ligao CC. Embora
perfeitamente estvel temperatura ambiente, o BINOL pode racemizar quando aquecido.

3. Qual dos fenis abaixo podem formar ( temperatura ambiente) enantimeros estveis, de
modo que eles possam ser utilizados da mesma forma para produzir um catalisador quiral?
Aviso: resposta assinalada erroneamente ir resultar em pontos de penalizao

Substncia Pode ser usada Substncia Pode ser usada
OH
OH
OCH
3
OCH
3



OH
OH



OH
OH
OCH
3
OCH
3



OCH
3
OH
HO
CH
3
O



OH
OH



OH
OH




4. O excesso enantiomrico, ee, utilizado para caracterizar a pureza enantiomrica de uma
substncia. Esta quantidade igual razo entre a diferena e a soma nas concentraes dos
enantimeros R e S:
[ ] [ ]
[ ] [ ]
R S
ee
R S


O excesso enantiomrico do ismero R puro 1, o ee da mistura racmica zero.

Quando se utiliza o enantiomericamente puro (BINOL) Al (OiPr) como um catalisador para a
reduo de -bromoacetofenona, o ee do produto igual a 81%. Qual ser o ee do produto, se
o ee do catalisador for igual a 50%? Apresente seu clculo com uma ilustrao ou derivao
da frmula final.

4. O excesso enantiomrico, ee, utilizado para caracterizar a pureza enantio-
mrica de uma substncia. Esta quantidade igual razo entre a diferena
e a soma nas concentraes dos enantimeros R e S:
Nome:_________________________________________________Cdigo do Estudante: BRA-S___
17

A quiralidade do BINOL deve-se ao impedimento estrico da rotao da ligao CC. Embora
perfeitamente estvel temperatura ambiente, o BINOL pode racemizar quando aquecido.

3. Qual dos fenis abaixo podem formar ( temperatura ambiente) enantimeros estveis, de
modo que eles possam ser utilizados da mesma forma para produzir um catalisador quiral?
Aviso: resposta assinalada erroneamente ir resultar em pontos de penalizao

Substncia Pode ser usada Substncia Pode ser usada
OH
OH
OCH
3
OCH
3



OH
OH



OH
OH
OCH
3
OCH
3



OCH
3
OH
HO
CH
3
O



OH
OH



OH
OH




4. O excesso enantiomrico, ee, utilizado para caracterizar a pureza enantiomrica de uma
substncia. Esta quantidade igual razo entre a diferena e a soma nas concentraes dos
enantimeros R e S:
[ ] [ ]
[ ] [ ]
R S
ee
R S


O excesso enantiomrico do ismero R puro 1, o ee da mistura racmica zero.

Quando se utiliza o enantiomericamente puro (BINOL) Al (OiPr) como um catalisador para a
reduo de -bromoacetofenona, o ee do produto igual a 81%. Qual ser o ee do produto, se
o ee do catalisador for igual a 50%? Apresente seu clculo com uma ilustrao ou derivao
da frmula final.
O excesso enantiomrico do ismero R puro 1, o ee da mistura racmica zero.
Quando se utiliza o enantiomericamente puro (BINOL) Al (OiPr) como um cata-
lisador para a reduo de -bromoacetofenona, o ee do produto igual a 81%.
Exame Terico
>> Olimpada Brasileira de Qumica - 2013
95
Qual ser o ee do produto, se o ee do catalisador for igual a 50%? Apresente seu
clculo com uma ilustrao ou derivao da frmula fnal.
Derivao:
ee =
PROBLEMA 4. UM EXPERIMENTO INORGNICO SIMPLES
(6 PONTOS)
Questo 1 2 3 Total
Pontos 5 12 7 24
1. Determine o metal X e os compostos A, B.
Suas dedues (your work):
X =____________ A =____________ B =____________
45
th
IChO
Programa Nacional Olimpadas de Qumica <<
96
2. Aps a adio de uma certa quantidade de tiossulfato de sdio soluo de A
a cor torna-se, imediatamente, vermelho, depois muda para castanho-aver-
melhada, e depois de alguns minutos, forma-se um precipitado marrom-es-
curo de C (reao 1). A soluo sobrenadante incolor. Quando aquecido ao
ar, a 600 C, C d um p cinzento X (reao 2), assim como 0,90 g de resduo
pode ser obtido a partir de 1,10 g de C. Um gs liberado pelo aquecimento de
C, a vcuo, (reao 3) pode ser absorvido por uma suspenso de hidrxido
clcio (reao 4). Sendo armazenado durante um longo tempo sob uma so-
luo saturada de perclorato de brio em HClO
4
, 0,1 M, a cor do precipitado
se torna mais clara, enquanto o uso de perclorato de magnsio no d esse
efeito. O que o C? Escreva as equaes das reaes (1 - 4).
Suas dedues (your work):
C = _______
Equaes das reaes:
3. O composto C sendo armazenado sob a gua-me (contendo um excesso de
A) muda de cor para amarelo, devido transformao em D. Se ions brio
so adicionados suspenso de C no gua-me, forma-se uma mistura de D
e de um precipitado branco. Proponha a frmula D, tendo em conta que ela
contm 77,5% (em massa) de X. Escreva a equao de formao de D.
Suas dedues (your work):
D = _______
Equaes das reaes:
Exame Terico
>> Olimpada Brasileira de Qumica - 2013
97
PROBLEMA 5. ESTIMATIVA SIMPLES DAS PROPRIEDADES
DO GRAFENO (7 PONTOS)
Questo
1
2 3
Total
a b
Pontos 2 2.5 4 5.5 14
O grafeno um material bidimensional da espessura de um tomo de carbono
(Fig.1 a). Muitas camadas de grafeno juntas formam grafte (Fig. 1b).
Fig. 1. (a) A estrutura do grafeno. As esferas so tomos de carbono. Eles esto
dispostos em hexgonos. A rea de um hexgono de carbono de 5,1610
-20
m
2
(b) Estrutura cristalina de grafte. Trs camadas de grafeno so mostradas
Tal estrutura atmica foi considerada por muito tempo como sendo instvel. No
entanto, em 2004, Andrey Geim e Konstantin Novoselov relataram a produo
das primeiras amostras desse material incomum. Esta inveno revolucionria
foi premiada com o Prmio Nobel em 2010.
Estudos experimentais de grafeno ainda so restritos. A produo de grandes
pores dessa nova substncia ainda um problema sinttico desafador. Mui-
tas propriedades do grafeno foram estimadas. Normalmente, no h informa-
45
th
IChO
Programa Nacional Olimpadas de Qumica <<
98
es sufcientes para clculos rigorosos, por isso temos de fazer suposies e
negligenciar fatores sem importncia. Neste problema, voc vai estimar as pro-
priedades de adsoro de grafeno.
1 a. Estime a superfcie especfca de grafeno livre para a adsoro em unida-
des de m2 /g. Considere que o grafeno plano est separado de qualquer outra
substncia slida ou lquida.
Clculos: S = _________ m
2
/g
Uma simples camada de molculas de nitrognio adsorvidas sobre a superfcie
exterior de grafte mostrado na Fig. 2. Assuma que o mesmo arranjo de mol-
culas de nitrognio formado sobre uma superfcie de grafeno.
Fig. 2. Molculas de nitrognio, N
2
(crculos cinzento), sobre a superfcie exte-
rior de grafte
1 b. Quantos gramas de nitrognio pode ser adsorvido sobre um grama de gra-
feno, assumindo que a camada de grafeno colocada sobre a superfcie de
um suporte slido? Estime o volume ocupado por estas molculas de nitro-
gnio aps a completa dessoro proveniente de 1 g de grafeno (presso de 1
bar, temperatura de 298 K).
Clculos:

m
N
2
= _______ g
v
N
2
= _______ g
Exame Terico
>> Olimpada Brasileira de Qumica - 2013
99
Vamos considerar adsoro como um equilbrio qumico comum

Nome:_________________________________________________Cdigo do Estudante: BRA-S___
24







2
N
_______ V .

Vamos considerar adsoro como um equilbrio qumico comum

gas ads
A A

, (1)
(A
gas
so molculas A, no estado gasoso, A
ads
so as mesmas molculas na superfcie)
com a constante de equilbrio K:

ads
gas
2
A
A
(mol/m )
(bar)
n

p

(tal suposio vlida se um pequeno nmero de molculas adsorvida na superfcie)

Propriedades de adsoro de grafeno podem ser estimadas a partir dos dados de adsoro sobre
uma grafite tridimensional regular. A entalpia de adsoro (H
o
da reao (1)) de qualquer
molcula de A no grafeno , em mdia, 10% menos negativa quando comparada com a de
grafite. Na grafite, a molcula adsorvida est ligado mais fortemente devido interao com as
camadas inferiores de grafeno na estrutura (Fig. 1b) e, portanto, a entalpia de adsoro mais
negativa. As entropias padro de adsoro sobre grafeno e grafite so assumidas serem as
mesmas.

2. Quantos moles, n, de CCl
4
so adsorvidos em 1 g de grafeno com p(CCl
4
) = 10
4
bar se
2,010
7
mol de CCl
4
so adsorvidos em 1 m
2
de grafite com p(CCl
4
) = 6,610
5
bar? Assuma
que o grafeno colocado sobre a superfcie de um suporte slido e a interao de CCl
4
com o
suporte no altera a entalpia de adsoro de CCl
4
no grafeno. A temperatura em ambos os
casos de 293 K. O H
o
de adsoro de CCl
4
em grafite - 35,1 kJ / mol.

Clculos:












n(CCl
4
) = _______
(1)
(A
gas
so molculas A, no estado gasoso, A
ads

so as mesmas molculas na superfcie)
com a constante de equilbrio K:
Nome:_________________________________________________Cdigo do Estudante: BRA-S___
24







2
N
_______ V .

Vamos considerar adsoro como um equilbrio qumico comum

gas ads
A A

, (1)
(A
gas
so molculas A, no estado gasoso, A
ads
so as mesmas molculas na superfcie)
com a constante de equilbrio K:

ads
gas
2
A
A
(mol/m )
(bar)
n

p

(tal suposio vlida se um pequeno nmero de molculas adsorvida na superfcie)

Propriedades de adsoro de grafeno podem ser estimadas a partir dos dados de adsoro sobre
uma grafite tridimensional regular. A entalpia de adsoro (H
o
da reao (1)) de qualquer
molcula de A no grafeno , em mdia, 10% menos negativa quando comparada com a de
grafite. Na grafite, a molcula adsorvida est ligado mais fortemente devido interao com as
camadas inferiores de grafeno na estrutura (Fig. 1b) e, portanto, a entalpia de adsoro mais
negativa. As entropias padro de adsoro sobre grafeno e grafite so assumidas serem as
mesmas.

2. Quantos moles, n, de CCl
4
so adsorvidos em 1 g de grafeno com p(CCl
4
) = 10
4
bar se
2,010
7
mol de CCl
4
so adsorvidos em 1 m
2
de grafite com p(CCl
4
) = 6,610
5
bar? Assuma
que o grafeno colocado sobre a superfcie de um suporte slido e a interao de CCl
4
com o
suporte no altera a entalpia de adsoro de CCl
4
no grafeno. A temperatura em ambos os
casos de 293 K. O H
o
de adsoro de CCl
4
em grafite - 35,1 kJ / mol.

Clculos:












n(CCl
4
) = _______
(tal suposio vlida se um pequeno nmero de molculas adsorvida na su-
perfcie)
Propriedades de adsoro de grafeno podem ser estimadas a partir dos dados de
adsoro sobre uma grafte tridimensional regular. A entalpia de adsoro (Ho
da reao (1)) de qualquer molcula de A no grafeno , em mdia, 10% menos
negativa quando comparada com a de grafte. Na grafte, a molcula adsorvida
est ligado mais fortemente devido interao com as camadas inferiores de
grafeno na estrutura (Fig. 1b) e, portanto, a entalpia de adsoro mais negati-
va. As entropias padro de adsoro sobre grafeno e grafte so assumidas serem
as mesmas.
2. Quantos moles, n, de CCl
4
so adsorvidos em 1 g de grafeno com p(CCl
4
) =
10
4
bar se 2,0
.
10
7
mol de CCl
4
so adsorvidos em 1 m
2
de grafte com p(CCl4)
= 6,6
.
10
5
bar? Assuma que o grafeno colocado sobre a superfcie de um
suporte slido e a interao de CCl
4
com o suporte no altera a entalpia de
adsoro de CCl
4
no grafeno. A temperatura em ambos os casos de 293 K. O
Ho de adsoro de CCl
4
em grafte - 35,1 kJ / mol.
Clculos: n(CCl
4
) = _______
Espera-se que os flmes de grafeno sejam sensveis detectores de gs. Se 10
9
par-
tculas de um gs est adsorvido em 1 cm
2
de uma superfcie de grafeno, isso
sufciente para medir uma mudana de resistividade eltrica da camada de
grafeno e detectar a presena de gs no ambiente.
45
th
IChO
Programa Nacional Olimpadas de Qumica <<
100
3. Determine o teor mnimo de etano, C
2
H
6
, no ar (em mol.%) presso atmos-
frica (T = 293 K), no qual um sensor de grafeno detecta este gs. Os dados
conhecidos para a adsoro de alcanos em grafte so mostrados na Fig. 3.
Suponha que o ar no afeta as propriedades de adsoro de etano.
Fig. 3. As propriedades termodinmicas de adsoro de alcanos em uma super-
fcie de grafte. (a) ln K {mol/m
2
/bar} como uma funo de ln M (M - massa
molecular do alcano, em g/mol), (b) Ho de adsoro em funo de ln M. Depen-
dncias lineares so assumidos em ambos casos.
Exame Terico
>> Olimpada Brasileira de Qumica - 2013
101
Clculos:
Contedo de C
2
H
6
= _________ mol.%
PROBLEMA 6. CICLOPROPANOS. TO SIMPLES.
TO ELEGANTE (8 PONTOS)
Questo 1 2 3 Total
Pontos 8 22 70 100
Ciclopropanos contendo substituintes dadores e aceitadores ligados a tomos de
C vizinhos, como por exemplo - A, apresentam elevada reatividade semelhante
a uma espcie 1,3-zwiterinica do tipo B.
Nome:_________________________________________________Cdigo do Estudante: BRA-S___
27

Problema 6. Ciclopropanos. To simples. To elegante (8 pontos)
Questo
1 2
3 Total
Pontos
8 22
70 100
Ciclopropanos contendo substituintes dadores e aceitadores ligados a tomos de C vizinhos,
como por exemplo - A, apresentam elevada reatividade semelhante a uma espcie 1,3-
zwiterinica do tipo B.

Assim, o A1 (X = 4-OMe) sofre abertura do anel de trs membros numa reao catalizada por
um cido de Lewis, com o nuclefilo 1,3-dimetoxibenzeno, originando o produto C.
1. Escreva a frmula estruturla de C.
Frmula estrutural de C:










A1 participa em reaes de cicloadio, annulations, oligomerizaes e outros processos.
Deste modo, a reaco de cicloadio [3+2] entre A1 e 4-metoxibenzaldeido origina o
composto D que contem um anel de cinco-membros. A descarboxilao de todos os grupos
carboxlicos existentes em D origina o composto E (C
18
H
20
O
3
), cuja estrutura apresenta um
plano de simetria.

2. Escreva as estruturass do composto D e E indicando as suas estereoqumicas.

Assim, o A1 (X = 4-OMe) sofre abertura do anel de trs membros numa reao
catalizada por um cido de Lewis, com o nucleflo 1,3 dimetoxibenzeno,
originando o produto C.
1. Escreva a frmula estruturla de C.
Frmula estrutural de C:
A1 participa em reaes de cicloadio, annulations, oligomerizaes e outros
processos. Deste modo, a reaco de cicloadio [3+2] entre A1 e 4-metoxibenza-
45
th
IChO
Programa Nacional Olimpadas de Qumica <<
102
ldeido origina o composto D que contem um anel de cinco-membros. A descar-
boxilao de todos os grupos carboxlicos existentes em D origina o composto E
(C
18
H
20
O
3
), cuja estrutura apresenta um plano de simetria.
Nome:_________________________________________________Cdigo do Estudante: BRA-S___
27

Problema 6. Ciclopropanos. To simples. To elegante (8 pontos)
Questo
1 2
3 Total
Pontos
8 22
70 100
Ciclopropanos contendo substituintes dadores e aceitadores ligados a tomos de C vizinhos,
como por exemplo - A, apresentam elevada reatividade semelhante a uma espcie 1,3-
zwiterinica do tipo B.

Assim, o A1 (X = 4-OMe) sofre abertura do anel de trs membros numa reao catalizada por
um cido de Lewis, com o nuclefilo 1,3-dimetoxibenzeno, originando o produto C.
1. Escreva a frmula estruturla de C.
Frmula estrutural de C:










A1 participa em reaes de cicloadio, annulations, oligomerizaes e outros processos.
Deste modo, a reaco de cicloadio [3+2] entre A1 e 4-metoxibenzaldeido origina o
composto D que contem um anel de cinco-membros. A descarboxilao de todos os grupos
carboxlicos existentes em D origina o composto E (C
18
H
20
O
3
), cuja estrutura apresenta um
plano de simetria.

2. Escreva as estruturass do composto D e E indicando as suas estereoqumicas.

2. Escreva as estruturass do composto D e E indicando as suas estereoqumicas.
D E
Composto de tipo A podem tambm sofrer transformaes apenas na presena
de catalisadores e sem a adio de outros reagentes. Algumas transformaes
tpicas de A1 esto indicadas no esquema abaixo.
Para determinar a estrutura dos compostos F-J, um conjunto de parmetros
fsico-qumicos foram obtidos (ver na tabela 1 alguns desses resultados). Foi en-
contrado que:
a) F e G tem a mesma frmula molecular que A1;
b) G o estereoismero mais estvel;
c) H e I so ismeros estruturais (constitucionais);
d) H formado como um nico diastereoismero com um eixo de simetria C
2
(a
molcula igual aps uma rotao de 180
o
);
e) I uma mistura de 2 diastereoismeros;
Exame Terico
>> Olimpada Brasileira de Qumica - 2013
103
f) J um derivado do naftaleno.
No processo de sntese de I, uma das molculas de A1 apresenta o tipo de reativi-
dade anteriormente referida (isto , comportando-se como se fosse B). A outra
molcula de A1 tem um comportamento diferente. Este ltimo comportamento
tambm demostrado pelo ciclopropano A2 [2-(3,4,5-trimetoxifenil)ciclopro-
pano-1,1-dicarboxilato de metilo; ou seja, o X em A = 3,4,5-(MeO)
3
] quando trata-
do com SnCl
4
origina K que uma mistura de dois diastereoismeros. O ismero
principal tem um centro de simetria. Uma reao semelhante ocorre quando A2
e G reagem na presena do catalisador Sn(OTf)
2
orginando L.
Tabela 1. Informao sobre os compostos.
Razo do nmero de grupos contendo hidrognio
Frmula
empirica
No-aromticos Aromticos
CH
CH CH2 CH3 OH
A1 1 1 1+1+1 0 2+2
(C
14
H
16
O
5
)n
F 1 1 1+1+1 0 2+2
(C
14
H
16
O
5
)n
G 1+1+1 0 2+1 0 2+2
(C
14
H
16
O
5
)n
H 1 1 1+1+1 0 2+2
(C
14
H
16
O
5
)n
I 1+1+1 1+1 2+1+1+1+1 0 2+2+1+1+1
(C
14
H
16
O
5
)n
J 0 0 1+1 1 1+1+1+1+1
(C
13
H
12
O
4
)n
K 1+1 1 2+1+1+1 0 1
(C
16
H
20
O
7
)n
L 1+1+1+1+1 1 2+2+1+1+1+1 0 2+2+1
(C
5
H
6
O
2
)n
3. Escreva as estruturas dos compostos F-J, L e do ismero predominante de K.
45
th
IChO
Programa Nacional Olimpadas de Qumica <<
104
F G
H I
J K (ismero predominante)
L
Exame Terico
>> Olimpada Brasileira de Qumica - 2013
105
PROBLEMA 7. PERMANGANOMETRIAS (8 PONTOS)
Questo 1
5 4 5 Total
a b c d a b
Pontos 2 2 4 2 2 6 7 7 2 34
A quantidade de muitos agentes redutores pode ser determinada por titulao
permanganomtrica, em meio alcalino, para possibilitar a reduo do on per-
manganato a manganato.
1. Escreva a equao inica da reao da titulao do on formato com perman-
ganato em soluo aquosa, contendo ~0,5 M de NaOH.
A titulao do permanganato em meio alcalino frequentemente efetuada com
a adio de um sal de brio que leva precipitao do manganato na forma de
BaMnO
4
.
2. Que processo redox paralelo, envolvendo manganato, impedido pelo sal de
brio?
Escreva abaixo um exemplo de equao da reao correpondente
Em cada um dos frascos A, B e C foram colocados 10,00 mL (V
Mn
) de uma solu-
o a 0,0400 mol.dm
-3
(c
Mn
) de KMnO
4
. E diferentes reaes ocorreram em cada
frasco.
45
th
IChO
Programa Nacional Olimpadas de Qumica <<
106
3. Ao frasco A, foram adicionados uma amostra contendo uma quantidade des-
conhecida de cido crotnico (CA) CH
3
CH=CHCOOH (m
CA
), uma base e ni-
trato de brio (ambos em excesso). A mistura reacional foi incubada durante
45 min. Sabe-se que cada molcula de cido crotnico libera 10 eltrons nes-
tas condies experimentais. A massa molar do CA 86,09 g/mol.
a) Escreva a equao qumica global da reao.
8,00 mL (V
CN
) de uma soluo a 0,0100 mol.dm
-3
(c
CN
) de cianeto de potssio
foram adiconados, em seguida, mistura incubada. Em consequncia ocorreu a
seguinte reao qumica:
2 Ba
2+
+ 2 MnO
4

+ CN

+ 2 OH

= 2 BaMnO
4
+ CNO

+ H
2
O
O BaMnO
4
precipita e removido por fltrao. O excesso de cianeto no fltrado
titulado com uma soluo 0,0050 mol.dm
-3
(c
Ag
) de AgNO
3
at se detectar a
formao de um procipitado. Note que os ons CN e CNO so semelhantes aos
ons haletos, mas o CNO origina sais de prata solveis.
b) Escreva a frmula do complexo formado no incio da titulao com Ag+ (an-
tes de se formar precipitado).
c) Escreva a frmula do precipitado formado.
Exame Terico
>> Olimpada Brasileira de Qumica - 2013
107
d) Calcule a massa de cido crotnico (em mg) se 5,40 mL (V
Ag
) da soluo de sal
de prata foram consumidos at o ponto fnal da titulao.
4. Outra amostra com uma concentrao diferente de cido crotnico e um ex-
cesso de uma soluo alcalina, foi adicionado ao frasco B. Esta mistura no
tem sal de brio. Um excesso de KI (em vez do cianeto) foi adicionado como
agente redutor. A mistura foi em seguida acidifcada e o iodo liberado foi
titulado com uma soluo 0,1000 mol.dm
-3
(c
S
) de tiossulfato.
Calcule a massa de cido crotnico (em mg) se 4,90 mL (V
Ag
) da soluo de titu-
lante tiverem sido consumidos at o ponto fnal da titulao.
5. Uma amostra contendo estanho(II) foi adicionada ao frasco , e o pH do meio
foi ajustado de forma a fcar ligeiramente alcalino. O estanho(II) foi quantita-
tivamente oxidado a Sn(OH)
6
2
, enquanto se formava um precipitado resul-
tante da reduo do permanganato. O precipitado foi isolado, lavado, seco a
250
O
C, pesado [a massa obtida do precipitado anidro (m
prec
), que um com-
posto binrio de Mn
x
O
y
, foi de 28,6 mg], e dissolvido em H
2
SO
4
na presena
de um excesso de iodeto de potssio. O iodo liberado foi titulado com uma
soluo a 0,1000 mol.dm
-3
de tiossulfato e foram utilizados 2,50 mL (V
S2
) para
atingir o ponto fnal da titulao.
a) Determine os coefcientes x e y e escreva a reao da formao do precipitado.
Reao:
b) Calcule a massa de estanho na amostra (em mg).
45
th
IChO
Programa Nacional Olimpadas de Qumica <<
108
Problema 8. A vida especial da archaea (8 points)
Questo 1 2 3 4 5 6 7 8
9
Total
a b
Pontos 2 7 3 8 4 4 5 4 3 5 45
A Archaea (ou archaebacteria) so microorganismos unicelulares signifcativa-
mente diferentes a nvel molecular de bactrias e clulas eucariotas.
A reao enzimtica da metilamina com gua a maior fonte de energia de algu-
mas archaeas. Numa experincia particular, um determinado tipo de archaea foi
cultivada a pH = 7 em condies anaerbicas (sem oxignio) num meio nutriente
contendo
13
CH
3
NH
2
como nica fonte de energia. Aps um determinado perodo
de incubao, foi recolhida uma amostra do gs existente acima da cultura de
archaea e posteriormente analisado. Verifcou-se que o gs contem 2 substncia
A e B numa relao molar de 1,00 para 3,00 respectivamente. A densidade rela-
tiva da amostra para com o H
2
12,0.
1. Calcule as porcentagens em volume das substncias A e B na mistura.
2. Determine A e B considerando que no existem tomos de N no gs recolhido.
Clculos:
A B
3. Escreva a equao da reao enzimtica da metilamina com gua anterior-
mente descrita, . usando a forma predominante de cada espcie.
Exame Terico
>> Olimpada Brasileira de Qumica - 2013
109
As enzimas contendo um dado resduo -aminocido X so encontradas em
muitas archaea. Sabe-se que X:
composto por tomos de 4 elementos;
Que tem 18,8% de oxignio em massa;
Possui um tRNA especfco e incorporado nas protenas durante a trans-
lao.
O aminocido L-lisina (ver estrutura no esquema abaixo) foi identifcado como
precursor de X na archaea. Todos os tomos de C e N encontrados em X provm
de duas molculas de L-lisina. Diferentes molculas de L-lisina marcadas iso-
topicamente foram usadas num sistema modelo para clarifcar os passos biosin-
ttico de X . Os resultados encontram-se sumarizados na tabela.
Composio isotpica da L-lisina Massa molecular (arredondada ao n-
mero inteiro) do resduo X [RCH(NH
2
)
CO], ligado ao tRNA, g/mol
Normal 238
Todos os carbono
13
C, todos os nitro-
gnios
15
N
253
Grupo amino da cadeia lateral com
15
N 239
4. Determine a frmula molecular de X.
Clculos:
C
45
th
IChO
Programa Nacional Olimpadas de Qumica <<
110
X:
A biossntese de X na archaea ocorre como descrito no esquema seguinte (En-
zimas E
1
E
3
):
Nome:_________________________________________________Cdigo do Estudante: BRA-S___
39
















X:





A biossntese de X na archaea ocorre como descrito no esquema seguinte (Enzimas E
1
E
3
):
HOOC
NH
2
NH
2
E
2
E
3
C
D E
X
E
1
-H
2
O
-H
2
O
?
HOOC
NH
2
NH
2
L-lisina
reao
espontnea
L-lisina

No primeiro passo, a L-lisina transformada num ismero estrutural (o aminocido C), enquanto
que D contm uma ligao pptidica e E um grupo formilo [
C
O
H
]. Todos os coeficientes na reao
apresentados no esquema so iguais a 1.
5. Indique as frmulas moleculares de C, D e E.
Dos tipos de reaes indicados abaixo, assinale apenas aquela que corresponde reao
catalisada pela enzima
3
.
Clculos:









No primeiro passo, a L-lisina transformada num ismero estrutural (o ami-
nocido C), enquanto que D contm uma ligao pptidica e E um grupo formilo
Nome:_________________________________________________Cdigo do Estudante: BRA-S___
39
















X:





A biossntese de X na archaea ocorre como descrito no esquema seguinte (Enzimas E
1
E
3
):
HOOC
NH
2
NH
2
E
2
E
3
C
D E
X
E
1
-H
2
O
-H
2
O
?
HOOC
NH
2
NH
2
L-lisina
reao
espontnea
L-lisina

No primeiro passo, a L-lisina transformada num ismero estrutural (o aminocido C), enquanto
que D contm uma ligao pptidica e E um grupo formilo [
C
O
H
]. Todos os coeficientes na reao
apresentados no esquema so iguais a 1.
5. Indique as frmulas moleculares de C, D e E.
Dos tipos de reaes indicados abaixo, assinale apenas aquela que corresponde reao
catalisada pela enzima
3
.
Clculos:









. Todos os coefcientes na reao apresentados no esquema so iguais a 1.
5. Indique as frmulas moleculares de C, D e E.
Dos tipos de reaes indicados abaixo, assinale apenas aquela que corresponde
reao catalisada pela enzima E
3
.
Clculos:
Exame Terico
>> Olimpada Brasileira de Qumica - 2013
111
C X: X:
Desaminao oxidativa;
Descarboxilao;
Desaminao intermolecular;
Hidroxilao;
Hidrlise da ligao pptidica.
X contm o fragmento:

Nome:_________________________________________________Cdigo do Estudante: BRA-S___
40






C

D

E

Desaminao oxidativa;
Descarboxilao;
Desaminao intermolecular;
Hidroxilao;
Hidrlise da ligao pptidica.


contm o fragmento:
N
(R,Me,H)
(H,Me,R)
(R,Me,H)
3 4
5

onde R um substituinte grande (M>100 g/mol); o 3 tomo de C simtrico; os tomos de C 4 e 5
so centros estereognicos; todos os C do anel esto ligado a pelo menos um tomo de H; cada um
dos substituintes H, Me e R s aparece uma vez.
6. Indique a localizao dos substituintes H, Me e R no fragmento.
Suas dedues (your work)






















onde R um substituinte grande (M>100 g/mol); o 3 tomo de C simtrico;
os tomos de C 4 e 5 so centros estereognicos; todos os C do anel esto ligado
a pelo menos um tomo de H; cada um dos substituintes H, Me e R s aparece
uma vez.
6. Indique a localizao dos substituintes H, Me e R no fragmento.
Suas dedues (your work)
7. Escreva as frmulas estruturais de C e X apresentando detalhes estereo-
qumicos. Tenha em considerao que nenhum estereocentro afetado na
transformao de C em X. Indique ainda a confgurao absoluta (R/S) dos
estereocentros de X.
45
th
IChO
Programa Nacional Olimpadas de Qumica <<
112
C X:
Somente um cdo responsvel pela incorporao do resduo X na protena da
archaea. As bases nitrogenadas que formam esse cdo contm 2 grupos amino
exocclicos e trs oxignios exocclicos no total.
Nome:_________________________________________________Cdigo do Estudante: BRA-S___
41

7. Escreva as frmulas estruturais de C e X apresentando detalhes estereoqumicos. Tenha em
considerao que nenhum estereocentro afetado na transformao de C em X. Indique ainda a
configurao absoluta (R/S) dos estereocentros de X.
C










X

Somente um cdo responsvel pela incorporao do resduo X na protena da archaea. As bases
nitrogenadas que formam esse cdo contm 2 grupos amino exocclicos e trs oxignios
exocclicos no total.

8. Preencha a tabela abaixo para determinar a composio nucleotdica do cdo que codifica.
Assinale apenas um box por linha.
Suas dedues (your work)










Base
nitrogenada
Nmero de bases no codo
1 2 3 0 or 1 1 or 2
A

C

G

U

8. Preencha a tabela abaixo para determinar a composio nucleotdica do
cdo que codifca. Assinale apenas um box por linha.
Suas dedues (your work)
Base
nitrogenada
Nmero de bases no codo
1 2 3 0 or 1 1 or 2
A
C
G
U
A sequncia dada abaixo o fragmento do mRNA que contm os cdes que
codifcam a incorporao do resduo X na enzima da archaea:
5AAUAGAAUUAGCGGAACAGAGGGUGAC3
9 a. Utilizando a tabela do cdigo gentico, indique quantos resduos de ami-
nocidos so incorporados na cadeia da enzima pela translao deste frag-
mento.
Nmero de aminocidos: __________
Exame Terico
>> Olimpada Brasileira de Qumica - 2013
113
9 b. Indique a sequncia de aminocidos obtida atravs deste fragmento. Note
que o fragmento tem mais do que um resduo X. Escreva nos boxes as abre-
viaturas dos aminocidos comeando no terminal N e terminando no C. Note
que o nmero de caixas excessivo. Se tiver mais do que uma possibilidade,
escreva todas elas separadas por /. Se a translao for interrompida numa
posio particular escreva STOP e deixe os restantes boxes direita vazios.
Nome:_________________________________________________Cdigo do Estudante: BRA-S___
42


A sequncia dada abaixo o fragmento do mRNA que contm os cdes que codificam a
incorporao do resduo X na enzima da archaea:
5AAUAGAAUUAGCGGAACAGAGGGUGAC3
9a. Utilizando a tabela do cdigo gentico, indique quantos resduos de aminocidos so
incorporados na cadeia da enzima pela translao deste fragmento.

Nmero de aminocidos: __________


9b. Indique a sequncia de aminocidos obtida atravs deste fragmento. Note que o fragmento tem
mais do que um resduo X. Escreva nos boxes as abreviaturas dos aminocidos comeando no
terminal N e terminando no C. Note que o nmero de caixas excessivo. Se tiver mais do que uma
possibilidade, escreva todas elas separadas por /. Se a translao for interrompida numa posio
particular escreva STOP e deixe os restantes boxes direita vazios.








(a) RNA Cdons para os 20 aminocidos


Abreviaturas dos aminocidos:
segunda base

Ala = Alanine
U C A G

Arg = Arginine
U
Phe Ser Tyr Cys U
T
e
r
c
e
i
r
a
b
a
s
e


Asn = Asparagine
Phe Ser Tyr Cys C

Asp = Aspartic acid
Leu Ser STOP STOP A

Cys = Cysteine
Leu Ser STOP Trp G

Glu = Glutamic acid
C
Leu Pro His Arg U

Gln = Glutamine
Leu Pro His Arg C

Gly = Glycine
Leu Pro Gln Arg A

His = Histidine
Leu Pro Gln Arg G

Ile = Isoleucine
A
Ile Thr Asn Ser U

Leu = Leucine
Ile Thr Asn Ser C

Lys = Lysine
Ile Thr Lys Arg A

Met = Methionine
Met(start) Thr Lys Arg G

Phe = Phenylalanine
G
Val Ala Asp Gly U

Pro = Proline
Val Ala Asp Gly C

Ser = Serine
Val Ala Glu Gly A

Thr = Threonine
Val Ala Glu Gly G

Trp = Tryptophan

Tyr = Tyrosine

Val = Valine
45
th
IChO
Programa Nacional Olimpadas de Qumica <<
114
45
th
International
Chemistry Olympiad
Face your challenge,
Be smart
Exame prtico
JULHO 18, 2013
MOSCOU, RSSIA
R E G R A S G E R A I S
Regras de segurana: siga aquelas dadas no caderno de problemas preparatrios. No coma nem beba
no laboratrio.
Violao regras de segurana: na primeira vez voc ser chamado ateno; caso repita, voc ser
desclassifcado.
Problemas e caderno resposta: 27 pginas (incluindo a pgina capa e a Tabela Peridica dos elementos)
contendo trs problemas. Comea com o problema 1.
Tempo: 5 h; 15 min para leitura antes do incio. Ser dado um aviso 30 min antes do trmino.
Seu cdigo de estudante: escreva-o em todas as pginas.
Respostas: escreva somente nas caixas de respostas do caderno, nada fora ser considerado. Clculos rel-
evantes devem ser apresentados.
Use somente a caneta e a calculadora fornecida.
Bureta: faa a leitura o mais preciso possvel.
Necessita de mais reagentes? Pergunte ao assistente de laboratrio. Nenhuma penalidade ser aplicada
por solicitar mais reagentes, com excesso aos itens a seguir.
Cada poro extra de aldedo, de 2,4-dinitrofen il-hidrazine, de 50 mL of HCl, de titulante EDTA ou de
soluo de polmero: penalidade de 1 ponto em 40.
Seja bem cuidadoso! No haver reposio do viscosmetro caso voc o quebre!
Perguntas relacionadas a segurana, equipamentos, reagentes e ida ao toilete: pergunte ao seu as-
sistente de laboratrio.
Descarte de reagente: descarte somente no bquer de 800 mL rotulado WASTE.
Verso ofcial em ingls disponvel caso solicitada para maior clareza. Pea ao seu assistente de laboratrio.
Aps o sinal de pare coloque o seu caderno e todos os papis de grfco dentro do envelope; (no o feche),
deixe-o em sua bancada.
Voc deve parar o seu experimento imediatamente aps o sinal de stop ser dado. Um atraso de 5 min
resultar em zero pontos para o exame.
Durante o exame prtico algumas vidrarias e plsticos sero utilizados mais de uma vez. Lave-os cuida-
dosamente.
Exame Prtico
>> Olimpada Brasileira de Qumica - 2013
115
Lista de Reagentes
Reagente Quantidade Recipiente Rotulado Segurana
Problema 1
2,4-Dinitrofenil-
hidrazina
200 mg cada,
2 vidros
frasco pequeno
rosqueado
,4-dinitrophenylhydrazine H228, H302
cido sulfrico,
concentrado
1 mL cada,
2 vidros
frasco de plstico
rosqueado
H2SO4 concentrated H314
Soluo de aldedo 1
mmol em etanol
4 mL cada,
2 vidros
30 mL frasco de
vidro pequeno
com tampa
Aldehyde 1 and Aldehyde2 H319 e
H302
Etanol 30 mL frasco de vidro
com tampa
Ethanol H225
Soluo de NaOH
(usada nos
problemas 1e 2)
27 mL 60 mL frasco de
vidro com tampa
NaOH 2M H314
Acetona 30 mL Frasco rosqueado
de vidro ambar
Acetone H225, H319,
H336
Problema 2
Soluo padro de
EDTA, 0,0443M*
70 mL frasco de vidro
com tampa de
125 mL
EDTA 0.05M H319
Soluo padro de
HCl, 0,0535M*
70 mL frasco de vidro
com tampa de
125 mL
HCl H314, H335
Alaranjado de
metila, 0,1% em
gua
25 mL frasco
conta-gotas
Methyl orange H301
Indicator Murexida,
mistura slida com
NaCl (1:250 em
massa)
10 mL frasco pequeno
rosqueado
Murexide
Amostra de gua 500 mL frasco de plstico
de 0,5 L
Water sample
45
th
IChO
Programa Nacional Olimpadas de Qumica <<
116
Problema 3
lcool polivinlico,
poli(vinil) lcool
40 mL cada,
5 vidros
Vidro rosqueado
ambar
P1, P2, P3, P4 e X
A ser usada em todos os problemas
gua destilada 500 mL Frasco de
plstico para
lavagem
H2O
A compartilhar pelos estudantes, na mesa em comum
Hidrocarbonato de
sdio
800 mL 800 mL bquer NaHCO3
*Aconcentraoindicadanortuloaproximada.Osvaloresexatosestoindicadosnatabela.
Utenslios e equipamentos
Item Quantidade
Em cada local de trabalho
Frasco de plstico rosqueado de 5 mL, rotulado 1 com seu cdigo de estudante 1
Frasco de plstico rosqueado de 5 mL, rotulado 2 com seu cdigo de estudante 1
Suporte universal 1
Bquer de 50 mL 2
Bquer de 25 mL 2
Bquer de 25 ou 50 mL 1
Agitador magntico 1
Barra magntica 2
Filtro de vidro com placa sinterizada 2
Adaptador 1
balo de fundo redondo de 50 mL 1
Trompa de gua para vcuo 1
Pipeta de 2 mL 2
Exame Prtico
>> Olimpada Brasileira de Qumica - 2013
117
Pipeta de 5 mL 2
Pr-pipeta 1
Esptula 2
frasco de lavagem de plstico de 500 mL 1
bquer para descarte de 800 mL 1
Proveta de 10 mL 1
Papel de fltro, redondo 2
Tesoura 1
Papel de fltro 2
Basto de vidro 1
Papel indicador de pH (no saco com ziper ) 3
Viscosmetro 1
Cronmetro 1
bulbo de borracha (pera) de 30 mL 1
Rgua 1
Marcador 1
Bureta de 25 mL 1
Pipeta de 25 mL 1
Funil de plstico 1
Frasco Erlenmeyer 2
Tiras para teste para determinar o contedo de slidos (no saco com ziper) 1
Leno de papel (no canto de cada mesa, a ser compartilhado por 3 estudantes) 1 pacote
Cesta de plstico 1
Papel milimetrado 4 folhas
Escala de pH (no saco com ziper) 1
Nas mesas para uso comum
Papel de fltro, redondo
Papel de fltro
Luvas
45
th
IChO
Programa Nacional Olimpadas de Qumica <<
118
Balanas
Vidro H
2
O dist.
Termmetro imerso em H2O
Proveta de 100 mL
pH-metro
*Casovocnecessitedemaispapeldefltro,vocpodeencontrarnamesadeusocomum.
Problema 1
Cdigo do estudante
BRA ________
Quest. 1 2 3 4 5 Total
Pontos 3.5 1.5 1 3 35 44
Problema 1. Snteses das 2,4-dinitrofenil-hidrazonas (13 pontos)
As hidrazonas pertencem a classe das iminas que contm uma ligao simples
nitrognio-nitrognio adjacente a uma ligao dupla carbono-nitrognio. As hi-
drazonas so formadas quando hidrazinas reagem com aldedos ou cetonas, sob
condies apropriadas. Como as hidrazonas so derivados de compostos carbo-
nlicos, elas so, frequentemente, compostos estveis, cristalinos, slidos muito
coloridos e so usadas para identifcar aldedos e cetonas.
Neste problema, voc ter que identifcar dois benzaldedos substitudos (mos-
trados abaixo) pela anlise dos produtos obtidos nas suas reaes com 2,4-dini-
trofenil-hidrazina.

Exame Prtico
>> Olimpada Brasileira de Qumica - 2013
119
Procedimento
Preparao de 2,4-dinitrofenil-hidrazonas
Ateno!Norealizeasduassntesessimultaneamente,porqueumbquer
podecairdoagitadormagnticoevocpoderperderamisturareacional.
Coloque uma barra magntica em um bquer de 50 mL. Fixe o bquer no agita-
dor magntico usando o anel de metal conectado ao suporte universal. Coloque
o contedo do frasco (200 mg of 2,4-dinitrofenil-hidrazina) no bquer e inicie
agitando cuidadosamente. Somente na presena do assistente de laboratrio, adi-
cione cuidadosamente uma amostra de cido sulfrico concentrado (1 mL) ao
slido. Usando pipetas adicione 1,6 mL de gua e 4 mL of etanol mistura rea-
cional. Ento, usando uma pipeta, adicione gota a gota, a soluo de aldedo con-
tida no fraco (cada frasco aldehyde 1 ou aldehyde 2, contm 1,00 mmol do
aldedo). Um precipitado brilhante comea a formar-se. Continue agitando por
10 min e ento, adicione 10 mL de gua e agite por mais 3 min.
Separao e purifcao do produto
Usando a tesoura corte cuidadosamente um papel de fltro circular de 1 cm a
mais que o dimetro do funil de placa sinterizada. Molhe o papel de fltro circu-
lar com gua, e fxe-o cuidadosamente na superfcie da placa do funil. O papel
fltro cortado, deve encaixar perfeitamente no funil. Se ao cortar o papel fl-
tro, voc no obtiver um crculo bem feito, repita o corte pegando outro
papel fltro na mesa de material de uso comum. Remova a barra magntica
do bquer, usando a esptula e transfra o produto da reao para o funil de
placa sinterizada. Ligue a trompa de vcuo (se voc tiver difculdade, solicite
ajuda do assistente de laboratrio) e fltre o precipitado. Coloque um pouco
de gua no bquer e transfra o resto de produto para o funil de placa sinteri-
zada. Lave o slido com gua at que o pH das gotas do fltrado se torne neutro.
(Use o bquer rotulado com WASTE para descartar o contedo do balo de fun-
do redondo). Ento lave o slido duas vezes com etanol, usando no mais que 3
mL em cada vez (Nota: a hidrazona ligeiramente solvel em etanol). Seque
o slido no funil de placa sinterizada, usando a trompa de vcuo, mexendo o
slido cuidadosamente, de vez em quando, com o basto de vidro. Aps cerca de
20-30 min, transfra cuidadosamente o p seco para um papel de fltro dobrado
45
th
IChO
Programa Nacional Olimpadas de Qumica <<
120
por voc, no formato de box, para secagem fnal ao ar. Coloque esse box com
o produto em um lugar seguro (por ex. na prateleira). Desligue a trompa de
vcuo quando no estiver usando! To logo seus produtos estejam secos, ns
recomendamos que voc os pese, para evitar a formao de fla para uso das ba-
lanas. Coloque seus produtos nos tubos plsticos com seu cdigo de estudante
e complete as respostas no box abaixo. Nota: Os produtos que voc sintetizou
sero posteriormente re-examinados pelo staff do laboratrio.
Repita o procedimento acima, com o outro aldedo.
Tubo plstico 1 Tubo plstico 2
Massa do tubo vazio________________mg Massa do tubo vazio_______________ mg
Massa do tubo com produto___________ mg Massa do tubo com produto___________mg
Massa do produto _________________ mg Massa do produto _________________mg
Assinatura do assistente de laboratrio __________________________________
1.1. Escreva abaixo as estruturas da 2,4-dinitrofenil-hidrazina e de ambos os
produtos.
1.2. Que espcie de estereoisomerismo (se algum) possvel para estas hidrazo-
nas? Assinale o box apropriado.
R/S E/Z treo/eritro mano/gluco D/L
Exame Prtico
>> Olimpada Brasileira de Qumica - 2013
121
2.1. Qual o papel do cido sulfrico na formao da 2,4-dinitrofenil-hidrazona?
Assinale o box apropriado.
reagente estequiomtrico catalisador agente redutor agente oxidante
2.2. Como mudaria a velocidade de reao, se a sntese fosse realizada em meio
neutro? Assinale o box apropriado.
aumentaria muito
no mudaria
aumentaria ligeiramente
a reao ocorreria muito lentamente
2.3. Como mudaria a velocidade de reao, se a sntese fosse realizada em meio
alcalino? Assinale o box apropriado.
aumentaria muito
no mudaria
aumentaria ligeiramente
a reao ocorreria muito lentamente
Caracterizao
Coloque uma pequena quantidade (uma ponta de espatula) de cada produ-
to, cada um em um bquer 25 mL. Adicione 10 mL de acetona a cada bquer. O
melhor resultado ter sido obtido se a cor e a intensidade de cor de cada
bquer for similarmente amarela. Adicione 5 mL de soluo de NaHCO
3
em
cada bquer. Agite a mistura resultante em cada bquer, com o basto de vidro,
usando as diferentes pontas do basto.
3.1. Assinale suas observaes sobre as mudanas de cor das solues, no box
abaixo.
A cor no muda em nenhum bquer
A cor muda signifcativamente em ambos os bqueres
A cor muda signifcativamente somente em um bquer
Adicione 2 mL de soluo de NaOH a cada uma das misturas resultantes da ques-
to 3.1. Agite a mistura reacional com o basto de vidro.
45
th
IChO
Programa Nacional Olimpadas de Qumica <<
122
3.2. Assinale suas observaes sobre as mudanas de cor das solues, no box
abaixo.
A cor no muda em nenhum bquer
A cor muda signifcativamente em ambos os bqueres
A cor muda signifcativamente somente em um bquer
4.1. Que aspecto estrutural de seus produtos justifca a mudana de cor obser-
vada na reao com NaHCO
3
? Assinale o box apropriado.
presena do grupo MeO na posio 4 do anel benznico;
presena do grupo MeO na posio 3 do anel benznico;
presena do grupo OH na posio 4 do anel benznico;
presena de ambos os grupos MeO e OH.

4.2. Qual dos processos listados abaixo responsvel pela mudana de cor ob-
servada na reao das 2,4-dinitrofenil-hidrazonas com soluo aquosa de
NaOH? Assinale o box apropriado.
hidrlise alcalina desidratao

hidratao
desprotonao desidrogenao
4.3. Desenhe as estruturas das principais espcies orgnicas presentes em cada
meio reacional citado nos boxes abaixo.
Aldedo inicial Aldedo inicial
Exame Prtico
>> Olimpada Brasileira de Qumica - 2013
123
Soluo de NaHCO
3
Soluo de NaHCO
3
Soluo de NaOH Soluo de NaOH
5. Coloque os nmeros 1 ou 2 sob cada estrutura. Calcule o rendimento per-
centual de ambas as hidrazonas.

Nmero:_____________
Clculo do rendimento:

Nmero:_____________
Clculo do rendimento:
Rendimentos:
45
th
IChO
Programa Nacional Olimpadas de Qumica <<
124
Nmero 1_________________________________ %
Nmero 2_________________________________ %
Reagentes substitudos
ou extra
Assinatura do assistente
de laboratrio
Penalidade
Problema 2. Determinao do ndice de saturao de Langelier da
gua de uma piscina (12 pontos)
O ndice de saturao de Langelier (LI) uma medida da corrosividade da gua
de uma piscina bem como da sua capacidade para dissolver ou depositar carbo-
nato de clcio.
Se o LI for aproximadamente zero, a gua considerada balanceada. Se o LI
for um nmero postitivo a gua tende a depositar carbonato de clcio e forma
um depsito. Se o LI for um nmero negativo a gua corrosiva e dissolve car-
bonato de clcio. O LI uma combinao dos valores das grandezas fsicas e dos
fatores a ela associados (ver tabela 1) e pode ser calculado pela frmula:
Exame Prtico
>> Olimpada Brasileira de Qumica - 2013
125
Temperatura,

FT
Dureza em
clcio (CH),
mg/L CaCO
3
FD
Alcalinidade
Total (TA),
mg/L CaCO
3
FA
Total de Slidos
Dissolvidos
(TDS), mg/L NaCl
FTDS
0 0,0 5 0,3 5 0,7 0 12,0
3 0,1 25 1,0 25 1,4 - -
8 0,2 50 1,3 50 1,7 1000 12,1
12 0,3 75 1,5 75 1,9 - -
16 0,4 100 1,6 100 2,0 2000 12,2
19 0,5 150 1,8 125 2,1 - -
24 0,6 200 1,9 150 2,2 3000 12,25
29 0,7 250 2,0 200 2,3 - -
34 0,8 300 2,1 300 2,5 4000 12,3
41 0,9 400 2,2 400 2,6 - -
53 1,0 600 2,35 800 2,9 5000 12,35
- - 800 2,5 1000 3.0 - -
- - 1000 2,6 - - 6000 12,4
Neste experimento voc vai determinar o valor de LI de uma amostra de gua
fornecida. Note que a dureza expressa como equivalente da concentrao de
CaCO
3
(mg/L). A alcalinidade total dada pelo equivalente cido, que corres-
ponde quantidade total de carbonato e hidrogenocarbonato, tambm expressa
em mg/L de CaCO
3
. O valor de TDS recalculado em funo da concentrao de
NaCl (mg/L).
Procedimento
A Dureza em Clcio determinada por titulao complexomtrica com EDTA
(Na
2
H
2
Y). Esta titulao efetuada em meio fortemente alcalino para evitar a
interferncia do magnsio (grandes quantidades de Mg
2+
interferem pois co-
-precipitam com o clcio na forma de Mg(OH)
2
; alm disso, o indicador usado na
complexometria tambm adsorvido em Mg(OH)
2
, impedindo a observao da
alterao de cor). A titulao deve ser feita imediatamente depois da adio da
soluo alcalina para evitar a deposio do CaCO
3
.
45
th
IChO
Programa Nacional Olimpadas de Qumica <<
126
1.1. Escreva a equao qumica da reao que ocorre durante a titulao com
Na
2
H
2
Y:
Procedimento para a determinao de clcio
a) Coloque a soluo padro de EDTA (concentrao exata de 0,0443 mol.dm
-3
)
na bureta.
b) Pipete 20 mL da amostra de gua (Water sample) para um Erlenmeyer.
c) Adicione 3 mL da soluo NaOH , 2 mol.dm
-3
, usando a proveta.
d) Adicione o indicador murexide com a esptula de forma a obter uma soluo
nitidamente cor-de-rosa.
e) Rapidamente titule com o EDTA at o indicador mudar de cor-de-rosa para
roxo.
1.2. Preencha a tabela 2.
Tabela 2
Titulao de clcio
N da titulo
Leitura Inicial da bureta, mL
Leitura Final da bureta, mL
Volume consumido, mL
Volume considerado, _________ mL

Exame Prtico
>> Olimpada Brasileira de Qumica - 2013
127
2. Calcule a dureza da amostra de gua em mg/L de CaCO
3
. Escreva o resultado
obtido na tabela 4 (ver na questo 7).
Clculos:
Medio do pH. Localize o medidor de pH no laboratrio (ou pergunte ao as-
sistente do laboratrio).
a) Coloque entre 70-90 mL da amostra de gua num Erlenmeyer limpo.
b) Remova a tampa protetora do medidor de pH (mantenha a tampa de forma a
no derramar a soluo nela contida).
c) Lave o eletrodo com gua destilada usando a pisseta (frasco lavador).
d) Ligue o medidor de pH atravs do boto ON/OFF.
e) Mergulhe o eletrodo na amostra de gua e agite suavemente o Erlenmeyer.
f) Coloque o Erlenmeyer na mesa e espere que o valor estabilize (no mais de 1
minuto).
g) Leia e registe o valor do pH.
h) Desligue o medidor de pH, lave o eletrodo com gua destilada e recoloque-o
na tampa protectora (em caso de haver fla passe o eletrodo ao estudante
seguinte).
3.1. Escreva o valor de pH na tabela 4 (ver na questo 7).
3.2. Qual forma do cido carbnico predominante na sua amostra de gua?
Confrme a sua escolha com clculos e assinale o quadrado correspondente.
Nota. As constantes de dissociao do cido carbnico so: K
1
= 4,5 x 10
7
; K
2
=
4,8 x 10
11
.
45
th
IChO
Programa Nacional Olimpadas de Qumica <<
128
Clculos:
Carbonato Hidrogenocarbonato cido Carbnico
3.3. Escreva a equao inica predominante da reao de titulao da amostra
de gua com HCl.
Determinao da alcalinidade total. Para obter o valor da alcalinidade total, a
amostra de gua deve ser titulada em relao ao H
2
CO
3
. O indicador cido-base
usado o alaranjado de metila (methyl orange), cuja mudana de cor de am-
arelo para laranja, em um pH prximo de 4,5.
a) Lave a bureta com gua destilada e encha-a com a soluo padro de HCl
(concentrao exata de 0,0535 mol.dm
-3
).
b) Pipete 50,0 mL de amostra de gua para um Erlenmeyer e adicione 3 gotas da
soluo de indicador (methyl orange).
c) Se a soluo estiver laranja antes da adio de cido signifca que a alcalini-
dade total zero. Se a soluo fcar amarela, titule com a soluo padro de
cido at notar a mudana de cor para laranja. Registe os volumes de titu-
lante na tabela 3.
Exame Prtico
>> Olimpada Brasileira de Qumica - 2013
129
4.1. Preencha a tabela 3.
Tabela 3
Determinao da
alcalinidade total
N da titulao
Leitura Inicial da bureta, mL
Leitura Final da bureta, mL
Volume consumido, mL
Volume considerado, _________ mL

4.2. Calcule a alcalinidade total (em mg/L de CaCO
3
). Escreva o resultado obtido
na tabela 4 (ver na questo 7).
Clculos:
5. Medida da Temperatura. Leia a temperatura no termmetro colocado na
mesa de uso comum e escreva o valor na tabela 4 (ver na questo 7).
6. Determinao de TDS na amostra de gua com a fta de teste fornecida.
a) Encha um bquer com a amostra de gua at um nvel de aproximadamente
3 cm de altura. Mergulhe a fta na amostra; certifque-se que a zona amarela
no topo da fta est a cima do nvel do lquido.
b) Espere entre 34 min at que a zona amarela fque totalmente castanha. Faa
a leitura do valor obtido com uma casa dcimal, tal como se mostra na fgura
abaixo.
c) Registre o valor:
d) Determine o valor da concentrao de TDS da sua amostra expresso em NaCl
(mg/L) usando a tabela que se encontra do lado direito da fgura.
45
th
IChO
Programa Nacional Olimpadas de Qumica <<
130
e) Escreva o valor da concentrao de NaCl na tabela 4 (ver na questo 7).

Reading NaCl conc., mg/L
1.4 360
1.6 370
1.8 420
2.0 430
2.2 470
2.4 530
2.6 590
2.8 660
3.0 730
3.2 800
3.4 880
3.6 960
3.8 1050
4.0 1140
4.2 1240
4.4 1340
4.6 1450
4.8 1570
5.0 1700
7. Preencha completamente a tabela 4. Calcule o valor de LI e escreva o resul-
tado na tabela 4. Considere os valores dos fatores como tendo uma preciso
de 2 casas decimais.
Tabela 4. Clculo do LI da amostra de gua
Nmero da amostra de gua (Water Sample Number) ______________
CH,
mg/L CaCO
3
TA,
mg/L CaCO
3
t, C pH TDS,
mg/L NaCl
LI
FD FA FT FTDS

Exame Prtico
>> Olimpada Brasileira de Qumica - 2013
131
Questes Tericas. Correo do balanceamento de gua.
Se o valor de LI for signifcativamente diferente de zero necessrio ajust-lo
a zero.
Imagine que lhe dada uma amostra de gua de uma piscina que foi analisa-
da por um procedimento igual ao que acabou de efetuar. Os resultados desta
anlise foram: CH = 550 mg/L, FD = 2,31, TA = 180 mg/L, FA=2,26, t = 24C, FT =
0,6; TDS = 1000 mg/L, FTDS = 12,1, pH = 7,9, LI = 0,97.
O responsvel pela piscina recolhe vrias amostras de 200 mL de gua. A cada
uma destas amostras adiciona 10 mL de uma soluo 0,0100 mol.dm
-3
dos rea-
gentes [NaHCO
3
, NaOH, NaHSO
4
, CaCl
2
, EDTA (sal dissdico di-hidratado) e HCl]
(1 reagente por amostra).
8. Determine se h depsito de CaSO
4
aps a adio de NaHSO
4
.
Nota: O produto de solubilidade do CaSO
4
5 x10
5
. Assum a que no se forma
precipitado de CaCO
3
aps a adio de qualquer uma das solues acima referidas.
Clculos:
Assinale a sua resposta Sim No
9. Preencha a tabela 5 indicando as alteraes resultantes da adio de cada
reagente s amostras de gua (utilize + se espera que o fator aumente, se
espera que o factor diminua, e 0 se no espera que ocorra alterao).
Tabela 5
Reagente pH FA FD FTDS LI
NaHCO
3
NaOH
NaHSO
4
CaCl
2
Na
2
H
2
Y
HCl
45
th
IChO
Programa Nacional Olimpadas de Qumica <<
132
Substituio de material
ou reagentes
Assinatura do Assistente
de laboratrio
Penalidade
Problema 3. Determinao da massa molecular
por viscosimetria (15 pontos)
O coefciente de viscosidade a medida da resistncia de um fuido a escorrer.
Pode ser determinada medindo a velocidade com que o lquido escorre atravs
de um tubo capilar fno. A viscosidade de uma soluo de um polmero aumenta
com o aumento da concentrao. A uma concentrao constante, as interaes
solvente-polmero so mais fortes como consequncia de uma maior expanso
da estrutura polimrica e consequentemente aumenta a viscosidade. Consid-
erando que a densidade de uma soluo diluda do polmero igual do sol-
vente, a viscosidade reduzida red da soluo de polmero com uma concen-
trao c (g/mL) defnida pela expresso:
Problem
3

Cdigo de estudante
BRA ________
Quest. 1 2 3 4 5 6 7 8 9 Total
Marks 3 2 0 27.5 5 0 19.5 4 1 64

19
Problema 3. Determinao da massa molecular por viscosimetria (15 pontos)

O coeficiente de viscosidade a medida da resistncia de um fluido a escorrer. Pode ser
determinada medindo a velocidade com que o lquido escorre atravs de um tubo capilar fino. A
viscosidade de uma soluo de um polmero aumenta com o aumento da concentrao. A uma
concentrao constante, as interaes solvente-polmero so mais fortes como consequncia de
uma maior expanso da estrutura polimrica e consequentemente aumenta a viscosidade.
Considerando que a densidade de uma soluo diluda do polmero igual do solvente, a
viscosidade reduzida
red
da soluo de polmero com uma concentrao c (g/mL) definida
pela expresso:
c t
t t
red
0
0


] / [ g mL ,
onde t e t
0
so os tempos de escoamento da soluo e do solvente puro, respectivamente.
A viscosidade reduzida para solues diludas de polmeros depende da concentrao de acordo
com a expresso:
kc c
red
) (
,
sendo k um factor (mL
2
/g
2
) e [] a viscosidade intrnseca (mL/g). A viscosidade intrnseca []
determinada por extrapolao da viscosidade reduzida quando a concentrao do polmero
zero. De um modo geral a viscosidade intrnseca est relacionada com a massa molecular M do
polmero segundo a equao de Mark-Kuhn-Houwink:



KM
,
onde K e so constantes para um par solvente-polmero especfico, a determinada temperatura.
Assim, M pode ser obtido atravs da equao de Mark-Kuhn-Houwink usando o valor
determinado experimentalmente [] e os valores tericos K e .

onde t e t
0
so os tempos de escoamento da soluo e do solvente puro, respec-
tivamente.
A viscosidade reduzida para solues diludas de polmeros depende da concen-
trao de acordo com a expresso:
Problem
3

Cdigo de estudante
BRA ________
Quest. 1 2 3 4 5 6 7 8 9 Total
Marks 3 2 0 27.5 5 0 19.5 4 1 64

19
Problema 3. Determinao da massa molecular por viscosimetria (15 pontos)

O coeficiente de viscosidade a medida da resistncia de um fluido a escorrer. Pode ser
determinada medindo a velocidade com que o lquido escorre atravs de um tubo capilar fino. A
viscosidade de uma soluo de um polmero aumenta com o aumento da concentrao. A uma
concentrao constante, as interaes solvente-polmero so mais fortes como consequncia de
uma maior expanso da estrutura polimrica e consequentemente aumenta a viscosidade.
Considerando que a densidade de uma soluo diluda do polmero igual do solvente, a
viscosidade reduzida
red
da soluo de polmero com uma concentrao c (g/mL) definida
pela expresso:
c t
t t
red
0
0


] / [ g mL ,
onde t e t
0
so os tempos de escoamento da soluo e do solvente puro, respectivamente.
A viscosidade reduzida para solues diludas de polmeros depende da concentrao de acordo
com a expresso:
kc c
red
) (
,
sendo k um factor (mL
2
/g
2
) e [] a viscosidade intrnseca (mL/g). A viscosidade intrnseca []
determinada por extrapolao da viscosidade reduzida quando a concentrao do polmero
zero. De um modo geral a viscosidade intrnseca est relacionada com a massa molecular M do
polmero segundo a equao de Mark-Kuhn-Houwink:



KM
,
onde K e so constantes para um par solvente-polmero especfico, a determinada temperatura.
Assim, M pode ser obtido atravs da equao de Mark-Kuhn-Houwink usando o valor
determinado experimentalmente [] e os valores tericos K e .

sendo k um factor (mL
2
/g
2
) e [] a viscosidade intrnseca (mL/g). A viscosidade
intrnseca [] determinada por extrapolao da viscosidade reduzida quando
a concentrao do polmero zero. De um modo geral a viscosidade intrnseca
est relacionada com a massa molecular M do polmero segundo a equao de
Mark-Kuhn-Houwink:
Exame Prtico
>> Olimpada Brasileira de Qumica - 2013
133
Problem
3

Cdigo de estudante
BRA ________
Quest. 1 2 3 4 5 6 7 8 9 Total
Marks 3 2 0 27.5 5 0 19.5 4 1 64

19
Problema 3. Determinao da massa molecular por viscosimetria (15 pontos)

O coeficiente de viscosidade a medida da resistncia de um fluido a escorrer. Pode ser
determinada medindo a velocidade com que o lquido escorre atravs de um tubo capilar fino. A
viscosidade de uma soluo de um polmero aumenta com o aumento da concentrao. A uma
concentrao constante, as interaes solvente-polmero so mais fortes como consequncia de
uma maior expanso da estrutura polimrica e consequentemente aumenta a viscosidade.
Considerando que a densidade de uma soluo diluda do polmero igual do solvente, a
viscosidade reduzida
red
da soluo de polmero com uma concentrao c (g/mL) definida
pela expresso:
c t
t t
red
0
0


] / [ g mL ,
onde t e t
0
so os tempos de escoamento da soluo e do solvente puro, respectivamente.
A viscosidade reduzida para solues diludas de polmeros depende da concentrao de acordo
com a expresso:
kc c
red
) (
,
sendo k um factor (mL
2
/g
2
) e [] a viscosidade intrnseca (mL/g). A viscosidade intrnseca []
determinada por extrapolao da viscosidade reduzida quando a concentrao do polmero
zero. De um modo geral a viscosidade intrnseca est relacionada com a massa molecular M do
polmero segundo a equao de Mark-Kuhn-Houwink:



KM
,
onde K e so constantes para um par solvente-polmero especfico, a determinada temperatura.
Assim, M pode ser obtido atravs da equao de Mark-Kuhn-Houwink usando o valor
determinado experimentalmente [] e os valores tericos K e .

onde K e so constantes para um par solvente-polmero especfco, a determi-
nada temperatura.
Assim, M pode ser obtido atravs da equao de Mark-Kuhn-Houwink usando o
valor determinado experimentalmente [] e os valores tericos K e .
Como trabalhar com o viscosmetro
1 Vaso coletor
2, 3 Tubos suplementares
4 Vaso de medida
5 Vaso coletor
6 Marcas de medio
7 - Capilar
a) Fixe o viscosmetro no suporte de forma que o tubo (3) fque na vertical e o
vaso colector (1) fque apoiado na bancada. Ajuste a garra o mais baixo pos-
svel.
b) Atravs do tubo (2), coloque no vaso colector (1) 10 mL do lquido a analisar,
usando uma pipeta.
c) Coloque a pera no topo do tubo (3) e sugue o lquido at ao vaso colector (5)
de forma a que o menisco do lquido fque 10 mm acima da marca superior
(6) e evitando bolhas de ar no capilar (7) (estas podem causar erros experi-
mentais signifcativos).
d) Zere o cronmetro e remova pr-pipeta ou pera do tubo (3), o lquido comea
a escorrer para o vaso coletor (1).
e) Mea o tempo de escoamento: inicie (start) o cronmetro quando o menis-
co do lquido atingir a marca (6) superior e pare (stop) o cronmetro quando
o menisco do lquido atingir a marca (6) inferior.
45
th
IChO
Programa Nacional Olimpadas de Qumica <<
134
ATENO: Utilize o viscosmetro como muito cuidado!
No h forma de substituir o viscosmetro se for quebrado!
Se quebrar o seu viscosmetro diga ao assistente de laboratrio. Se
isso acontece, pode tentar realizar a experincia usando a pipeta de
25 mL e um bquer.
Lave o viscosmetro trs vezes com gua da torneira e, depois, uma vez com
gua destilada antes de passar uma outra amostra de polmero. No necessrio
lavar o viscosmetro com a soluo de polmero. O erro causado desprezvel.
NO necessrio preencher totalmente a tabela de respostas. Efetue os ensaios
que achar necessrios para ter uma mdia de valores precisa.
Procedimento
Voc recebeu um conjunto de solues aquosas de polmeros (0,01 g/mL). Trs
das solues de polmero P1-P4 so de poli(vinil) lcool, enquanto que a outra
soluo de um poli(vinil)acetato hidrolisado, contendo cerca de 10% de uni-
dades no hidrolisadas.
No se sabe qual das solues estoque P1-P4 deste polmero. As massas mo-
leculares dos polmeros P1-P4 esto indicadas na tabela que se segue.
Massa molecular aproximada Cdigo da amostra
26650 P2
50850 P1
65300 P4
91900 P3
A amostra X um poli(vinil) lcool de massa molecular desconhecida.
Nesta experincia voc ter que identifcar qual das solues P1-P4 corresponde
ao poli(vinil) acetato parcialmente hidrolisado e determinar a massa molecular
do polmero X.
1. Complete o esquema abaixo, da reao da preparao do poli(vinil) lcool
por hidrlise do poli(vinil) acetato.
Exame Prtico
>> Olimpada Brasileira de Qumica - 2013
135
2. Escolha (assinale o box apropriado) o polmero que mostra interao mais
forte com a gua e compare as viscosidades das solues aquosas dos
polmeros poli(vinil) acetato total ou parcialmente hidrolisados. Considere
que as concentraes das solues e as massas moleculares dos polmeros
so as mesmas
Poli(vinil) lcool
Poli(vinil) acetato parcialmente hidrolisado
Comparao das viscosidades:
poli(vinil lcool) _________ Poli(vinil acetato) parcialmente hidrolisado (coloque no espao < , > , ou )
3. Mea o tempo de escoamento do solvente puro (gua destilada). NO
necessrio preencher totalmente a tabela de respostas.
Valor considerado: ___________ s

4. Mea o tempo de escoamento das solues estoque P1-P4, e da amostra X.
Calcule a viscosidade reduzida. NO necessrio preencher totalmente a
tabela de respostas. Efetue os ensaios que achar necessrios para ter uma
mdia de valores precisa.
45
th
IChO
Programa Nacional Olimpadas de Qumica <<
136
Amostra P2 (26650) P1 (50850) P4 (65300) P3 (91900) X
Tempo de
escoamento, s
Tempo de
escoamento
considerado:
_____ s _____ s _____ s _____ s _____ s
Clculos:
Amostra P2 (26650) P1 (50850) P4 (65300) P3 (91900)
X
viscosidade
reduzida
das solues
estoque dos
polmeros, mL/g
5. Assinale com um crculo, dentre as solues P1-P2-P3-P4, aquela que
correspondente a amostra de poli(vinil) acetato parcialmente hidrolisado.
Ateno: Leve em conta as massas moleculares dos polmeros P1-P4 fornecidas.
P1 P2 P3 P4
Exame Prtico
>> Olimpada Brasileira de Qumica - 2013
137
NO UTILIZE ESTE POLMERO NA PARTE SEGUINTE DA EXPERINCIA.

6. Para determinar os parmetros da equao de Mark-Kuhn-Houwink e calcu-
lar a massa molecular do polmero desconhecido X, escolha e assinale com
um crculo as duas solues de poli(vinil) lcool com massas moleculares
diferentes, mais apropriadas. Considere que o erro associado determinao
da viscosidade intrnseca no depende da massa molecular da amostra.
P1 P2 P3 P4
7. Utilizando o material de vidro apropriado para preparar solues, prepare
solues diludas das 3 amostras de polmeros de poli(vinil lcool) consid-
eradas anteriormente, isto , a amostra desconhecida X e os 2 polmeros es-
colhidos na questo 6 e calcule as viscosidades reduzidas correspondentes a
cada um deles. No clculo das concentraes das solues diludas, considere
a densidade da soluo de polmero igual da gua. Determine a viscosidade
intrnseca para cada uma das solues analisadas.
Entregue a folha de papel milimetrado identifcada onde traou o grfco junta-
mente com o seu caderno de respostas. Nota: Se desejar traar os grfcos refer-
entes s diferentes amostras no mesmo papel milimetrado, assegure-se que est
usando smbolos sufcientemente diferentes entre si, para conjunto de dados.
NO necessrio preencher totalmente a tabela de respostas.
Amostra: ___
Concentrao,
g/mL:
Soluo
estoque, mL
gua, mL
Tempo de
escoamento, s:
45
th
IChO
Programa Nacional Olimpadas de Qumica <<
138
Tempo de escoamento
considerado, s
Viscosidade reduzida,
mL/g
Viscosidade Intrnseca [], mL/g
Amostra: ___
Concentrao,
g/mL:
Soluo
estoque, mL
gua, mL
Tempo de
escoamento, s:
Tempo de escoamento
considerado, s
Viscosidade reduzida,
mL/g
Viscosidade Intrnseca [], mL/g
Amostra: ___
Concentrao,
g/mL:
Soluo
estoque, mL
gua, mL
Exame Prtico
>> Olimpada Brasileira de Qumica - 2013
139
Tempo de
escoamento, s:
Tempo de escoamento
considerado, s
Viscosidade reduzida,
mL/g
Viscosidade Intrnseca [], mL/g
Sumrio dos resultados experimentais (registre apenas os valores medidos).
Amostra P__ P__ X
Concentrao (c), g/mL: 0,01 0,01 0,01
viscosidade reduzida (red), mL/g
c (1 diluio), g/mL:
red, mL/g
c (2 diluio), g/mL:
red, mL/g
c (3 diluio), g/mL:
red, mL/g
c (4 diluio), g/mL:
red, mL/g
c (5 diluio), g/mL:
red, mL/g


8. Escreva a equao que voc vai usar para determinar as constantes K e a
Determine os valores de K e para a soluo aquosa de poli(vinil) lcool.
45
th
IChO
Programa Nacional Olimpadas de Qumica <<
140
Exame Prtico
K = _________ mL/g = __________
9. Usando os valores de K e obtidos e a viscosidade intrnseca da soluo da
amostra X, calcule a massa molecular do polmero X. Se no conseguir deter-
minar K e , use K = 0,1 mL/g e = 0,5.
Clculos:
M(X)=______________
Substituio de material
ou reagentes
Assinatura do assistente
de laboratrio
Penalidade
Viscosmetro quebrado
>> Olimpada Brasileira de Qumica - 2013
141
18
a
OIAQ
EXAME EXPERIMENTAL ORGNICA
I. CROMATOGRAFIA
Voc se encontra no Laboratrio de Qumica na La cidade de La Paz cuja presso
atmosfrica se encontra a 495 de mmHg. Os pontos de ebulio e a volatilidade
dos compostos orgnicos geralmente so afetados por estas condies.
Voc tem na bancada do laboratrio trs sustncias qumicas: CH
3
CH
2
OH,
CH
3
COOCH
2
CH
3
e H
2
O. Tambm h uma amostra de um alcaloide no tubo No1,
este alcaloide est dissolvido em CH
2
Cl
2
. O alcaloide, alm de ser solvel em
CH
2
Cl
2
, solvel em CH
3
CH
2
OH, H
2
O e em CH
3
COOCH
2
CH
3
.
Para realizar o experimento voc ter disponvel: placas cromatogrfcas de
slica gel, papel fltro e o respectivo material de vidro.
Responder as seguintes perguntas.
a) Indicar se o CH
2
Cl
2
uma molcula polar ou no polar
2 pts.
LA PAZ - BOLVIA
Exame Experimental
Quarta 16 de octubre de 2013
Programa Nacional Olimpadas de Qumica <<
142
Experimental
b) Na cromatografa em camada fna (TLC) a Slica Gel a fase estacionria, na
cromatografa de papel, qual a fase estacionria?
2 pts.
c) Colocar em ordem decrescente de polaridade os seguintes compostos
CH
3
CH
2
OH, H
2
O, CH
3
COOCH
2
CH
3
e CH
2
Cl
2
. (escrever, primeiramente, o mais
polar)
4 pts.
II. MODIFICAO DO Rf DE UM ALCALOIDE, CUJA FRMULA
MOLECULAR C
8
H
10
N
4
O
2
Foi realizada uma anlise preliminar por TLC do alcaloide que est no Tubo No
1, mostrando um Rf = 0,56 usando como fase mvel o CH
3
CH
2
OH e um Rf = 0,18
usando CH
3
COOCH
2
CH
3
como fase mvel, nos dois casos utilizou-se Slica Gel
como fase estacionria.
O alcaloide existente no Tubo No 1, cuja frmula molecular C
8
H
10
N
4
O
2
, deve ter
um Rf entre 0,27 0,47.
Determine qual eluente(s) ou mistura de solventes, voc utilizaria para ter este
Rf, sua resposta deve estar expressa em volume.
Para realizar este experimento, voc tem trs substncias qumicas: CH
3
CH
2
OH,
H
2
O e CH
3
COOCH
2
CH
3
para usar como fase mvel. Tens quatro placas croma-
togrfcas de SiO
2
para obter o Rf entre 0,27 e 0,47. A amostra do alcaloide ab-
sorve com lmpada de UV 254 nm.
Procedimento
Aplicao da amostra
Trace, com um lpis, uma linha fna acima da borda inferior da placa (1 cm
aproximadamente).
>> Olimpada Brasileira de Qumica - 2013
143
18
a
OIAQ
Com a pipeta Pasteur, micropipeta ou capilar, tome uma pequena quantidade
da amostra problema (Tubo No1) e aplique uma pequena gota sobre a camada
de Slica Gel.
Para realizar o cromatograma
Introduza a placa na cmara de eluio (copo de 50 mL), ao qual, previamente,
colocado o eluente adequado (Sugesto volume total de 4 mL). Uma vez que a
placa esteja dentro da cmara, no se deve move-la. Quando a frente do eluente
estiver quase na borda superior da camada de Slica Gel, abra o frasco, retire a
placa e marque a frente do eluente com o lpis. Coloque a placa sobre a folha de
papel e deixe-a secar ao ar (2 min).
Para visualizar o Alcaloide
Dirija-se ao supervisor que tem a lmpada UV. Coloque a placa cromatogrfca
sob a lmpada de luz UV marcando o contorno da mancha com um lpis e de-
termine seu Rf.
Forma de entrega do resultado
Entregue ao supervisor uma s placa cromatogrfca eluda e marcada com lpis
o lugar onde aparece a mancha (colocar a placa eluda dentro da bolsa).
Escreva, qual eluente ou mistura de solventes utilizou para ter este Rf, e tambm
o volume utilizado de cada solvente no caso de ser uma mistura.

Penalidades
Por cada placa cromatogrfca extra que solicite ser apenado com 1 pto.
Por cada micropipeta extra que solicite ser apenado com 1 pto.
Reagentes que pode solicitar sem penalidade
Acetato de etila, etanol e gua
Material
Programa Nacional Olimpadas de Qumica <<
144
1 bquer de 50 mL
1 Cmara de Cromatografa com tampa
1 Proveta de 10 mL
2 Pipetas Pasteur
4 Placas cromatogrfcas de Slica Gel
1 Rgua de 20 cm
1 Lpis preto
1 Pina
1 Micropipeta (capilar)
1 tubo capilar
1 papel fltro
Algodo
Reagentes
CH
3
CH
2
OH CH
2
Cl
2
H
2
O
CH
3
COOCH
2
CH
3
CH
3
COOCH
2
CH
3
EXAME EXPERIMENTAL INORGNICA
MATERIAIS
10 Tubos de ensaio
2 tubos de ensaio marcados A e B
1 Suporte para tubo de ensaio

REAGENTES
1.- cido clordrico 11.- Ferrocianeto de potssio
2.- cido sulfrico 12.- Permanganato de potssio
3.- Hidrxido de sdio 13.- Oxalato de potssio
4.- Amonaco 14.- Glucose
5.- Cloreto de brio 15.- Fenolftalena
Experimental
>> Olimpada Brasileira de Qumica - 2013
145
6.- Carbonato de sdio 16.- Azul de metileno
7.- Dicromato de potssio 17.- gua oxigenada
8.- Sulfato de cobre 18.- Batata
9.- Cromato de potssio 19.- Limo
10.- Cloreto frrico 20.- Amostra marcada Sal
EQUAES
Os dados de presso de vapor frente temperatura pode se representar por
meio da Equao de Antoine:

UNIVERSIDAD MAYOR DE SAN ANDRES
FACULTAD DE CIENCIAS PURAS Y NATURALES
FACULTAD DE INGENIERIA

________________________________________________________________________________
1

EXAME EXPERIMENTAL INORGNICA

MATERIAIS
10 Tubos de ensaio
2 tubos de ensaio marcados A e B
1 Suporte para tubo de ensaio



REAGENTES
1.- cido clordrico
2.- cido sulfrico
3.- Hidrxido de sdio
4.- Amonaco
5.- Cloreto de brio
6.- Carbonato de sdio
7.- Dicromato de potssio
8.- Sulfato de cobre
9.- Cromato de potssio
10.- Cloreto frrico

11.- Ferrocianeto de potssio
12.- Permanganato de potssio
13.- Oxalato de potssio
14.- Glucose
15.- Fenolftalena
16.- Azul de metileno
17.- gua oxigenada
18.- Batata
19.- Limo
20.- Amostra marcada Sal

EQUAES
Os dados de presso de vapor frente temperatura pode se representar por meio da
Equao de Antoine:
C T
B
A P

ln
Onde P est em mmHg, T em K e A, B y C so constantes. Para a gua as constantes so
A=18,3036, B=3816,44 e C= - 46,13.

INSTRUES
Para realizar esta parte experimental tenha em conta as seguintes instrues:
As reaes so qualitativas
Todos os reagentes lquidos so solues aquosas diludas
Siga cuidadosamente as instrues, quando se menciona o termo exatamente
deve se utilizar a quantidade assinalada.
Observe todos as mudanas ocorridas e anote-as para que possa responder
adequadamente os questionamientos
Onde P est em mmHg, T em K e A, B y C so constantes. Para a gua as con-
stantes so A=18,3036, B=3816,44 e C= - 46,13.
INSTRUES
Para realizar esta parte experimental tenha em conta as seguintes instrues:
As reaes so qualitativas
Todos os reagentes lquidos so solues aquosas diludas
Siga cuidadosamente as instrues, quando se menciona o termo exata-
mente deve se utilizar a quantidade assinalada.
Observe todos as mudanas ocorridas e anote-as para que possa responder
adequadamente os questionamientos
A fenolftalena, o azul de metileno e a gua oxigenada devem ser solicitadas
ao instrutor.
NOTA - Todas as reaes, (semirreaes, reaes inicas lquidas, reaes
inicas totais, reaes molequlares) devem estar devidamente balanceadas
18
a
OIAQ
Programa Nacional Olimpadas de Qumica <<
146
PRIMERA PARTE
Ensaio A (1 ponto)
a) Em um tubo de ensaio coloque aproximadamente 25 gotas de cido sulfrico
e adicione exatamente 2 gotas de hidrxido de sdio. Agite a amostra e adi-
cione 1 gota de fenolftalena
b) soluo anterior adicione 10 gotas de cloreto de brio.
1. (0,3 pontos) De acordo com a reao do item a) assinale com um X
a resposta correta

i) O tipo de reao :
Precipitao ( )
Neutralizao ( )
Redox ( )
Formao de um composto de coordenao ( )
ii) Em relao ao intercmbio de energia ela ::
Exotrmica ( )
Endotrmica ( )
Sem intercmbio de energia ( )
2. (0,2 pontos) Na reao do item (a) indique qual o reagente limitante.
Experimental
>> Olimpada Brasileira de Qumica - 2013
147
3. (0,5 pontos) Escreva a equao qumica da reao que ocorre no inciso (b),
na forma inica (sem ions espectadores)
Reao
Ensaio B (1 ponto)
a) Em um tubo de ensaio coloque 20 gotas de carbonato de sdio, adicione 1 gota
de fenolftalena e, cuidadosamente, adicione 20 gotas de cido clordrico.
1. (0,5 pontos) Escreva a equao qumica molecular que representa a reao:
2. (0,5 pontos) Escreva as equaes qumicas inicas sucessivas que envolvem
o on carbonato ao agregar cido.
Ensaio C (2 pontos)
Em um tubo de ensaio coloque 20 gotas de soluo de glucose, adicione 5 go-
tas de soluo de hidrxido de sdio e fnalmente 1 gota de azul de metileno.
Posteriormente submeta a um ligeiro aquecimento, at observar uma mudana
aprecivel.
1. (1ponto) De acordo com o observado, assinale com um X a resposta cor-
reta.
18
a
OIAQ
Programa Nacional Olimpadas de Qumica <<
148
O tipo de reao :
Precipitao ( )
Neutralizao ( )
Redox ( )
Formao de um composto de coordenao ( )
2. (1ponto) Quando se agita a soluo anterior, ela se torna azul, que substn-
cia intervm nessa alterao
gua ( )
Oxgenio ( )
Dixido de carbono ( )
Nitrogenio ( )
Ensaio D (1 ponto)
a) Em um tubo de ensaio coloque 20 gotas de soluo de dicromato de potssio,
adicione gota a gota, soluo de hidrxido de sdio at observar uma mu-
dana e adicione mais 2 gotas.
b) Posteriormente, adicione gota a gota, soluo de cido clordrico at que
perceba uma nova alterao.
1) (0,5 pontos por reao) Escreva as equaes qumicas, na forma inica
Reao
item a)
Reao
item b)
Ensaio E
(2 pontos)
a) Introduza 20 gotas de sulfato de cobre em um tubo de ensaio e adicione ex-
atamente 5 gotas de amonaco
b) Posteriormente adicione 15 gotas de amonaco
Experimental
>> Olimpada Brasileira de Qumica - 2013
149
1) (1 ponto cada) Escreva as equaes qumicas correspondentes e as mudanas
de cor observadas em cada caso
Reao
Inciso A
Reao
Inciso B
Ensaio F (2 pontos)
a) Colocar em um tubo de ensaio 20 gotas de cromato de potssio e adicione 5
gotas de gua oxigenada. Logo em seguida adicione 10 gotas de cido sulfrico
1. (2 pontos) Escreva a equao qumica da reao
Ensaio G (1 ponto)
Em dois tubos de ensaio coloque:
a) No primeiro tubo aproximadamente 20 gotas de cloreto frrico e adicione 5
gotas de ferrocianeto de potssio (hexacianoferrato (II) de potssio).
b) No segundo tubo, 20 gotas de sulfato de cobre e adicone 5 gotas de Ferroci-
aneto de potssio (hexacianoferrato (II) de potssio).

1. (0,5 pontos por reao) Escreva as equaes qumicas a as cores dos rea-
gentess e produtos, assim como os nomes das substncias formadas
Tubo 1
Tubo 2
18
a
OIAQ
Programa Nacional Olimpadas de Qumica <<
150
SEGUNDA PARTE
Ensaio H (4 pontos)
Um profesor entregou a um estudante dois tubos de ensaio marcados A e B, os
quais continham solues incolores e solicitou ao estudante que determinasse
o contedo dos tubos.
O estudante recebe as seguintes informaes adicionais:
Um dos tubos contm uma soluo de cido forte e o outro contm uma
soluo de base forte
O pH das solues so 4 e 12
Ento o estudante decide realizar o seguinte ensaio:
a) Aos dois tubos de ensaio marcados A e B contendo as
solues, juntou 20 gotas de oxalato de potssio.
b) Posteriormente juntou exatamente 2 gotas de per-
manganato de potssio.
c) Finalmente colocou por 5 min em um banho de gua
quente.
Repita a experiencia realizada pelo estudiante.
1. (2 ponto) Complete a seguinte tabela:
Qual o meio da soluo antes da reao
Tubo A
Tubo B
2. (2 pontos) Escreva a equao qumica da reao molecular ocorrida no tubo B:
Experimental
>> Olimpada Brasileira de Qumica - 2013
151
Ensaio I (2 pontos)
Um operrio, por sugesto de um analista qumico, realizou a seguinte teste
com um sal: cortou uma batata em duas metades. Sobre uma metade da batata
colocou o sal slido e posteriormente juntou 2 gotas de limo.
Repita a Prova que realizou o operrio e responda as seguintes perguntas:
1. Que elemento do sal pode detectar-se mediante este ensaio?
TERCERA PARTE - Prova demostrativa
Ensaio J (4 pontos)
Com ajuda do instrutor, mea a
temperatura de ebulio da gua
na cidade de La Paz.
1. (2 pontos) Determine a presso atmosfrica no laboratorio
18
a
OIAQ
Programa Nacional Olimpadas de Qumica <<
152
Experimental
2. (2 pontos) Observe o fenmeno que realiza o instrutor






Tapou o erlenmeyer contendo gua em ebullio

Inverteu o erlenmeyer

Ps um gelo sobre o erlenmeyer
Marque con um X a resposta correta. A gua volta a ferver porque
a) A temperatura aumenta ( )
b) A temperatura diminui ( )
c) A presso interna aumenta ( )
d) A presso interna diminui ( )
>> Olimpada Brasileira de Qumica - 2013
153
18
a
OIAQ
Exame Terico
Constantes Fsicas, Frmulas e Equaes
Constante de Avogadro, N = 6,0221 x 10
23
mol
1
Constante de Boltzmann, kB = 1,3807 x 10
23
JK
1

Constante de Universal dos gases, R = 8,3145 JK
1
mol
1
= 0,08205 atmLK
1
mol
1
= 62,4 mmHg L K
-1
mol
-1

Velocidade da luz, c = 2,9979 x 10
8
ms
1
p = 3,141592
Constante de Planck, h = 6,6261 x 10
34
Js 1 Angstrm () = 10
10
m
Carga do eltron, e = 1,602x10
-19
C P = 1 atm = 1 bar = 10
5
Pa
Condies de presso padro e temperatura, CNTP T = 298 K
Presso atmosfrica ao nvel do mar 760 mmHg
Presso atmosfrica em La Paz 495 mmHg
1 nanmetro (nm) = 10
9
m
ppm = mg kg
-1
= mg L
-1


Dados
Constantes de produto de solubilidade de alguns sais a 25 C
Nome do composto Frmula qumica Kps
Carbonato de clcio CaCO
3
4,79 x 10
-9
Carbonato de magnsio Mg CO
3
1,0 x 10
-5
Carbonato de ltio Li 2CO
3
6,4 x 10
-3
Carbonato de brio Ba CO
3
5,12 x 10
-9
Carbonato de chumbo Pb CO
3
7,94 x 10
-14
Salina de Uyuni Potos - Bolivia
Programa Nacional Olimpadas de Qumica <<
154
Exame Terico
Pontuao Problema 1.
1.1 1.2 1.3 1.4 1.5 Total Ponderado
9 9 9 9 5 41 20
QUIMICA ANALTICA
Problema 1
Salina de Uyuni, localizada no estado de Potos, na Bolivia, o maior depsito
de sal do mundo e, ao mesmo tempo, a maior reserva de litio. Segundo espe-
cialistas, das dez mil milhes de toneladas de sal contidas nesta salina, cinco
milhes equivalem a esta matria prima.
O ltio importante no apenas para a Bolvia, mas, tambm para o resto do
mundo e foi denominado o ouro branco do futuro.
Devido ao seu potencial eletroqumico, o ltio utilizado para armazenar en-
ergia. Tal sua importncia que 60 por cento dos telefones celulares e 90 por
cento dos computadores portteis que se encontram no mercado mundial tm
uma bateria de ons de ltio. Em um futuro prximo, esta matria prima tambm
ser necessria, em maiores propores, para movimentar a indstria automo-
bilstica do futuro com a produo de mais carros eltricos.
Segundo especialistas, para a extrao de ltio so necessrios vrios processos.
O primeiro a concentrao da soluo salina, como o projeto dos cones. A
seguir, por meio de um processo qumico se obtm carbonato de ltio.
A obteno de carbonato de ltio baseia-se, fundamentalmente, na baixa solu-
bilidade que possui este sal, os valores da constante de produto de solubilidade
de alguns carbonatos so encontrados na tabela mostrada na folha de rosto do
problema.
Com o objetivo de conhecer as propriedades qumicas do carbonato de ltio, foi
solicitado a quatro grupos de estudantes realizar diferentes experimentos, com
a fnalidade de determinar a solubilidade e o Kps do carbonato de ltio.
1.1.- O Grupo A de estudantes preparou uma soluo de carbonato de ltio colo-
cando 9,31 g de carbonato de ltio em um balo volumtrico de 250,0 mL e com-
>> Olimpada Brasileira de Qumica - 2013
155
pletou o volume com gua destilada. Ajustaram a temperatura da soluo em
25C, fltraram o excesso de carbonato de ltio, secaram ao ar livre e pesaram,
anotando uma massa de 6,11 g.
1.2.- O Grupo B pegou uma poro de 100,0 mL da soluo saturada de carbonato
de ltio e reagiu com nitrato de chumbo para precipitar o carbonato. A massa do
precipitado junto com o papel fltro, aps lavar e secar ao ar livre foi de 2,64 g.
O papel fltro pesava 0,67 g.
1.3.- O Grupo C tomou outra poro de 100 mL da soluo saturada de carbon-
ato de litio, adicionou um excesso de cido sulfrico e o aqueceu. Recolheu o
dixido de carbono em uma bureta para gases e o volume de gs seco recolhido
foi 162,0 mL. A temperatura ambiente era de 25C e a presso baromtrica de 1
atmosfera.
1.4.- O Grupo D continuou aquecendo a soluo do grupo (C) at eliminar toda
a gua e o excesso de cido sulfrico. Pesaram o resduo de sulfato de ltio e
anotaram 0,82 g.
a) Calcule a solubilidade em mol/L e o Kps do carbonato de ltio a 25C, para
todos os grupos de trabalho.
b) Segundo The Handbook of Chemistry and Physics o Kps do carbonato de ltio
6,4 x10-3. Calcule o erro experimental obtido por cada grupo em comparao
com este valor. Qual ou quais dos grupos realizaram adequadamente seu ex-
perimento, considerando uma tolerncia mxima de 10%?
1.5.- Um quinto grupo de trabalho (Grupo E), misturou 500 mL de uma soluo
de LiCl 0,8 mol/L com 500 mL de uma soluo de carbonato de sdio 0,4 mol/L.
a) Qual a quantidade, em gramas, que obtiveram? Utilize o valor de Kps do The
Handbook of Chemistry and Physics.
18
a
OIAQ
Programa Nacional Olimpadas de Qumica <<
156
Problema 2
Dados termodinmicos

UNIVERSIDAD MAYOR DE SAN ANDRES
FACULTAD DE CIENCIAS PURAS Y NATURALES
FACULTAD DE INGENIERIA

________________________________________________________________________________
Problema 2




Dados termodinmicos
Composto M

/ kJ mol
-1

/J K
-1
mol
-1

/kJ mol
-1

/J K
-1
mol
-1

CH
4(g)

16,04 -74,81 186,26 -50,72 35,31
C
2
H
6(g)

30,07 -84,68 229,60 -32,82 52,60
CO
2(g)

44,00 -395,51 213,74 -394,36 37,11
H
2
O
(l)

18,015 -285,83 69,91 -237,13 75,291

Variao de entalpia e energia interna:

)
v
sada
velocidade de sada do gs; A rea do orifcio; N nmero de partculas dentro do
recipiente; V volume do recipiente; R constante dos gases; T temperatura dentro do
recipiente; M massa molar do gs

vsada
velocidade de sada do gs; A rea do orifcio; N nmero de partculas
dentro do recipiente; V volume do recipiente; R constante dos gases; T tem-
peratura dentro do recipiente; M massa molar do gs.
Exame Terico
>> Olimpada Brasileira de Qumica - 2013
157
Pontuao Problema 2
2.1 2.2 2.3 2.4 2.5 2.6 Total
5 5 5 5 3

FISICO-QUIMICA
Problema 2.- O gs natural um dos recursos naturais mais importantes da
Bolvia, o qual, geralmente, no comercializado em unidades volumtricas
(como metros cbicos ou ps cbicos) mas, mais adequadamente, em unidades
de energia (BTU). A energia calorfca do gs natural depende de sua composio
(metano, etano, propano, butano e outros hidrocarbonetos). Se o gs com-
posto, principalmente, por metano e etano, consideramos ser um gs seco, se o
gs formado por metano, etano, propano, butano e pentanos considera-se ser
um gs hmido ou gs liquefeito de petrleo, GLP.
2.1.- Um estudante olmpico deseja conhecer a composio do gs natural, para
tal realiza, inicialmente, uma anlise qualitativa, encontrando que este gs est
formado por metano, etano e propano. Posteriormente, realiza a combusto
completa de 100,0 g do gs natural com oxignio gasoso, resultando um con-
sumo de 394,473 g deste gs e formando-se 754 g de dixido de carbono.
a) Escreva as equaes qumicas de combusto
b) Qual a composio do gs natural, em % de peso?
2.2.- Outro estudante olmpico decidiu investigar a composio do gs natural
seco por outro mtodo: a efuso, a velocidade de efuso de um gs atravs de um
orifcio diretamente proporcional presso do gs e inversamente proporcional raiz
quadrada de sua temperatura e massa molar (a equao explcita se encontra na folha
de rosto do problema); alm disso, a relao de velocidades de efuso de dois gases est
dada pela Ley de Graham. Primeiramente, encha um recipiente de 20 L para gases,
com He, e observe que em 2,0 minutos, 28,4 mL do He se efundem atravs de um
pequeno orifcio. Posteriormente, nas mesmas condiciones de presso e tem-
peratura encha o recipiente com gs natural seco (metano e etano), e observe
que 25 mL deste se efundem atravs do mesmo orifcio no mesmo espao de
tempo.
18
a
OIAQ
Programa Nacional Olimpadas de Qumica <<
158
a) Qual a composio do gs, em % de volume, na sada do orifcio?
b) Qual a composio do gs natural, em % de volume, no interior do recipi-
ente?
2.3.- Na empresa YPFB da cidade de La Paz (P = 495 mmHg), se pede para aquecer,
presso constante, 100 Litros de gs natural, de uma composio em volume de
80% de metano e 20% de etano, desde 20 C at 100 C. Qual quantidade de calor
ser necessria para realizar esta tarefa? Os dados termodinmicos encontram-se
na folha de rosto do problema.
2.4.- Na mesma empresa YPFB se pede para aquecer 10,0 L de gua desde 20 at
85 C por combusto do gs natural. Qual volume de gs, em metros cbicos,
necessrio queimar para aquecer a gua? O gs natural utilizado tem uma com-
posio de 80 % e 20% , respectivamente, em volume de metano e etano?
2.5.- Quando o gs natural contm propano e butano se diz que um gs hmido
e denominado gs liquefeito de petrleo, GLP. Sabendo que o calor de combus-
to padro do propano 2220 kJ mol
-1
, qual a entalpia de formao padro do
propano, em kJ mol
-1
?
2.6.- Pesquisadores de materiais catalticos do IGN-UMSA La Paz, garantem ha-
ver encontrado um novo catalizador, baseado em um metal nobre e ter obtido
atravs da rota dos atranos (um mtodo de sntese), que torna possvel a con-
verso direta do metano em etano em condies de presso e temperatura am-
bientes:

UNIVERSIDAD MAYOR DE SAN ANDRES
FACULTAD DE CIENCIAS PURAS Y NATURALES
FACULTAD DE INGENIERIA

________________________________________________________________________________

2.4.- Na mesma empresa YPFB se pede para aquecer 10,0 L de gua desde 20 at 85 C
por combusto do gs natural. Qual volume de gs, em metros cbicos, necessrio
queimar para aquecer a gua? O gs natural utilizado tem uma composio de 80 % e 20% ,
respectivamente, em volume de metano e etano?

2.5.- Quando o gs natural contm propano e butano se diz que um gs hmido e
denominado gs liquefeito de petrleo, GLP. Sabendo que o calor de combusto padro do
propano 2220 kJ mol
-1
, qual a entalpia de formao padro do propano, em kJ mol
-1
?
2.6 Pesquisadores de materiais catalticos do IGN-UMSA La Paz, garantem haver
encontrado um novo catalizador, baseado em um metal nobre e ter obtido atravs da rota
dos atranos (um mtodo de sntese), que torna possvel a converso direta do metano em
etano em condies de presso e temperatura ambientes:
2 CH
4(g)
C
2
H
6(g)
+ H
2(g)

a) Se isto verdade, qual a constante de equilbrio Kp desta reao, a 25C?
b) Se definirmos Grau de Converso molar como a porcentagem de mols de CH
4

convertidos em C
2
H
6
, qual seria o Grau de Converso deste processo. O
experimento foi realizado na UMSA de La Paz.






a) Se isto verdade, qual a constante de equilbrio Kp desta reao, a 25C?
b) Se defnirmos Grau de Converso molar como a porcentagem de mols de
CH4 convertidos em C
2
H
6
, qual seria o Grau de Converso deste processo.
O experimento foi realizado na UMSA de La Paz.
Exame Terico
>> Olimpada Brasileira de Qumica - 2013
159
Problema 3

UNIVERSIDAD MAYOR DE SAN ANDRES
FACULTAD DE CIENCIAS PURAS Y NATURALES
FACULTAD DE INGENIERIA

________________________________________________________________________________
Problema 3

Dados

K
1
= 2,5x10
4

K
2
= 1/(K
ps
xK
1
) = 2x10
11

Z
Co
= 27, e Z
o
= 8
Raios inicos: r
Co
2+
= 0,72 e r
O
2
= 1,4
Relao de raios para um ambiente octadrico r
+
/r

de 0,41 at 0,73
V
esfera
= 4r
3
/3
Eletronegatividade de Pauling:
Co
= 1,82, e
O
= 3,4, a equao para o clculo de % de
carter inico : [1e^(0,25||
2
)]x100
1 Faraday = 96500 C/mol
I (A) = q(C)/t (s)
M
Co
= 58,9 g/mol
M
O
= 16 g/mol
M
H
= 1 g/mol
M
Li
= 6,94 g/mol










18
a
OIAQ
Programa Nacional Olimpadas de Qumica <<
160
Pontuao Problema 3.
1(a) 1(b) 1(c) 1(d) 1(e) 2(a) 2(b) 2(c) 2(d) 2(e) 3(a) 3(b) 3(c) 3(d) Total
1 1 2 1 1 2 1 1 2 1 1 2 2 2 20
QUMICA INORGNICA
Problema 3.- Uma cela de on ltio comercial est constituda de um material
catdico de LiCoO2, e de um material andico de carbono grafte (Figura N1),
em muitos pases do mundo esto pesquisando o comportamento destas pilhas,
porque o nmero de vezes que se pode carregar e descarregar muito maior
que qualquer pilha atualmente existente no mercado. Isto faz com que sejam
potencialmente interessantes para sua aplicao em automveis eltricos, ou
hbridos, o qual permitiria uma grande reduo de automveis a gasolina ou
diesel, evitando a contaminao ambiental.

FiguraN1.Caractersticasdeumapilhadeonltio(Issuesandchallengesfacingrechargeable
lithiumbatteries,J.-M.Tarascon*&M.Armand,NATURE|VOL414|15NOVEMBER2001)
O material catdico de LiCoO
2
obtido a partir da reao no estado slido entre
o carbonato de ltio (Li
2
CO
3
) e o xido de cobalto (II) (CoO). O Li
2
CO
3
obtido por
processos de vaporizao e recristalizao a partir de salmouras (solues sali-
nas que contm sais de K, Mg, Li, e Na, principalmente sulfatos e cloretos), at-
ualmente, a Bolvia est implementando uma planta piloto para este propsito,
a qual permitir construir una planta industrial e produzir 30.000 toneladas de
carbonato de ltio em 2015.
Exame Terico
>> Olimpada Brasileira de Qumica - 2013
161
Por outro lado, o CoO obtido por calcinao do hidrxido de cobalto (Co(OH)
2
), o
qual produzido pelo mtodo de precipitao descrito pela seguinte equao geral:
Co
(ac)
2+
+ 2 OH
(ac)
-
Co(OH)
2 (S)
Kps (1)
Este hidrxido com uma estrutura tipo Brusita (isto : Co(OH)
2
), possvel
ser obtido no laboratrio por adio de hidrxido de sdio (C
OH-
= 1,0 mol/L), a
0,5 litro de um sal de cobalto (C
Co+2
= 0,5 mol/L) com uma relao molar fnal de
OH-/Co
+2
= 2, e seu posterior envelhecimento (repouso da soluo com o pre-
cipitado) por 18 horas, tudo em atmosfera de nitrognio.
1. As reaes identifcadas neste processo so:
Co
+2
(ac)
+ OH
-(ac)
Co(OH)
+
(ac)
K1 (2)
Co(OH)
+
(ac)
+ OH
-(ac)
Co(OH)
2(s)
K2 (3)
a) Considerando um modelo covalente, a hibridao do Co
+2
no [Co(H
2
O)
6
]
+2

(i) sp
3
d
2
, (ii) d
2
sp
3
, ou (iii) dsp
3
d. Em qual orbital est o stimo eltron do
Co
+2
: (i) 3d, (ii) 4s, (iii) 4d, ou (iv) nenhum dos anteriores.
b) A cor do complexo aquoso [Co(H
2
O)
6
]
+2
rosa vermelhado, e produzido
por transies eletrnicas nos orbitais do Cobalto. Considerando a teoria do
campo cristalino estas transies so de: (i) p a d, (ii) s a p, (iii) d a d, ou (iv)
nenhuma das anteriores..
c) Em um experimento de laboratrio, se estas reaes ocorrem sob as con-
dies indicadas, qual a massa de brucita solvel que resta na soluo, de-
pois do envelhecimento: (i) 3,58x10
-4
g, (ii) 3,18x10
-5
g, (iii) 3,42x10

g , ou (iv)
nenhuma.
d) Por que necessria a atmosfera inerte no processo de sntese? O que acon-
teceria se fosse feito na presena de ar? Justifque a reao que ocorre.
e) Assumindo que o sistema est em equilbrio aps este tempo (18 h), a massa
de hidrxido de cobalto (II) obtida (i) 23,232 g, (ii) 23,256 g, ou (iii) nen-
huma.
2. A reao de obteno de CoO, ocorre por aquecimento do hidrxido de co-
balto (II) em atmosfera inerte a 500oC por 2 horas. O CoO obtido apresenta
uma estrutura tipo NaCl (fgura N 2):
18
a
OIAQ
Programa Nacional Olimpadas de Qumica <<
162
FiguraN2.EstruturacbicadoCoO,tiporedesalina(NaCl)
a) Considerando um modelo covalente, qual seria a hibridao nos tomos de
oxignio: (i) sp
3
d
2
, (ii) no h hibridao, (iii) 2 hbridos sp, e p
x
e p
z
puros.
b) Considerando o modelo inico, identifque o ndice de coordenao para o
Co, e o O, e calcule a aresta da cela unitria.
c) A relao de raios para o entorno do Co
+2
no CoO adequada para sua esta-
bilidade?
d) Se assumirmos que os ons so esfricos, qual a frao do volume da cela
que est ocupada?
e) Calcule, segundo o modelo de Pauling, a percentagem do carter inico das
ligaes CoO no CoO. Com base nele, justifque qual dos dois modelos con-
siderados (covalente ou inico) mais prximo da realidade?
3. A reao do CoO com carbonato de ltio, em presena de ar, a 900C, por 17
horas, produz o LiCoO
2
, com estrutura laminar (Figura N 3).

FiguraN3.EstruturalaminardoLiCoO2.
Exame Terico
>> Olimpada Brasileira de Qumica - 2013
163
a) Que tipo de interao existe entre as lminas (CoO
2
)
-
e Li
+
.
b) Nas baterias de on ltio (Figura N 4), se no processo de carga ocorre, no elet-
rodo de carbono, a semirreao representada pela seguinte equao qumi-
ca:
6 C + Li
+
+ 1e- LiC
6
Eo = 0,6 V (1)
Carga
Qual ser a equao qumica no outro eletrodo?
FiguraN4.Procesodecargaemumaceladeionlitio(LiCoO2/C(grafte)
c) Como varia o estado de oxidao do cobalto na reao de carga?
d) Se na cela temos 5 g de LiCoO
2
e sabemos que somente 50% de ltio pode sair
da estrutura para manter sua capacidade de carga e descarga, qual o tempo
necessrio de carga, quando utilizamos uma intensidade de corrente con-
stante de 0,15A?
18
a
OIAQ
Programa Nacional Olimpadas de Qumica <<
164
Problema 4
Liquenalectoriaochroleuca
Nylon630%
Exame Terico
>> Olimpada Brasileira de Qumica - 2013
165
Pontuao do Problema 4
1a 1b 2a 2b 2c 2d 2e Total
3 7 2 2 2 1 1 18
QUMICA ORGNICA - Problema 4
1 - Os lquenes so espcies vegetais complexas que se formam de uma asso-
ciao entre algas e fungos. No entanto, a morfologia, a qumica e a bioqumica
so diferentes quando as algas e os fungos esto separados. Os lquenes esto
tradicionalmente classifcados em trs grupos de acordo com a forma de cresci-
mento: crustosos, frutosos e folhosos.
Os lquenes geram metabolitos secundrios (molculas qumicas) caractersti-
cos, alguns semelhantes aos das plantas superiores. At o presente, cerca de 750
metabolitos secundrios foram isolados de lquenes, entre os quais se incluem
depsidonas, depsidos, dibenzofuranos, antraquinonas, xantonas, terpenides,
carotenides, difenilteres e cidos alifticos.
Na Bolvia crescem vrias espcies de lquenes, em condies extremas, como
por exemplo radiao UV elevada, estresse osmtico, temperatura (10
o
C) e
condies de seca. Uma destas espcies que cresce nestas condies a alecto-
ria ochroleuca que cresce no distrito de La Paz a 3800 metros acima do nvel do
mar. Esta espcie foi colhida e foi submetida a uma extrao slido-lquido com
diclorometano, tendo-se isolado o (+)-cido snico (A), que um slido amarelo
e sobre o qual foram publicados, nos ltimos 10 anos, mais de 714 artigos cien-
tfcos. Uma particularidade desta molcula (A) a presena de muitos grupos
funcionais. Sendo, deste modo, um excelente material de partida para obter no-
vos derivados.

UNIVERSIDAD MAYOR DE SAN ANDRES
FACULTAD DE CIENCIAS PURAS Y NATURALES
FACULTAD DE INGENIERIA

________________________________________________________________________________
1a 1b 2a 2b 2c 2d 2e Total
3 7 2 2 2 1 1 18

QUMICA ORGNICA - Problema 4
1 - Os lquenes so espcies vegetais complexas que se formam de uma associao entre
algas e fungos. No entanto, a morfologia, a qumica e a bioqumica so diferentes quando
as algas e os fungos esto separados. Os lquenes esto tradicionalmente classificados em
trs grupos de acordo com a forma de crescimento: crustosos, frutosos e folhosos.

Os lquenes geram metabolitos secundrios (molculas qumicas) caractersticos, alguns
semelhantes aos das plantas superiores. At o presente, cerca de 750 metabolitos
secundrios foram isolados de lquenes, entre os quais se incluem depsidonas, depsidos,
dibenzofuranos, antraquinonas, xantonas, terpenides, carotenides, difenilteres e cidos
alifticos.
Na Bolvia crescem vrias espcies de lquenes, em condies extremas, como por exemplo
radiao UV elevada, estresse osmtico, temperatura (10
o
C) e condies de seca. Uma
destas espcies que cresce nestas condies a alectoria ochroleuca que cresce no distrito
de La Paz a 3800 metros acima do nvel do mar. Esta espcie foi colhida e foi submetida a
uma extrao slido-lquido com diclorometano, tendo-se isolado o (+)-cido snico (A),
que um slido amarelo e sobre o qual foram publicados, nos ltimos 10 anos, mais de 714
artigos cientficos. Uma particularidade desta molcula (A) a presena de muitos grupos
funcionais. Sendo, deste modo, um excelente material de partida para obter novos
derivados.

(A)
a) Indicar a estereoqumica (R, S, Z, E) presente no cido snico (A)
Outros derivados deste cido foram obtidos segundo as reaes seguintes:

18
a
OIAQ
Programa Nacional Olimpadas de Qumica <<
166
a) Indicar a estereoqumica (R, S, Z, E) presente no cido snico (A)
Outros derivados deste cido foram obtidos segundo as reaes seguintes:

UNIVERSIDAD MAYOR DE SAN ANDRES
FACULTAD DE CIENCIAS PURAS Y NATURALES
FACULTAD DE INGENIERIA

________________________________________________________________________________


a) Escrever a estrutura dos produtos das reaes de transformao do cido snico
indicadas nos esquemas.
2.- A famlia dos nylon ou poliamidas formada por diferentes tipos: nylon 6,6; nylon 6;
nylon 6,10; nylon 6,12; nylon 11; nylon 12 e nylon 6-6,6 so os copolmeros mais comuns.
A diferena no nmero de grupos metileno define as propriedades dos diferentes tipos de
nylon.

Exame Terico
>> Olimpada Brasileira de Qumica - 2013
167
a) Escrever a estrutura dos produtos das reaes de transformao do cido
snico indicadas nos esquemas.
2.- A famlia dos nylon ou poliamidas formada por diferentes tipos: nylon
6,6; nylon 6; nylon 6,10; nylon 6,12; nylon 11; nylon 12 e nylon 6-6,6 so os
copolmeros mais comuns. A diferena no nmero de grupos metileno defne
as propriedades dos diferentes tipos de nylon.
a) Escreva a equao da reao de sntese do nylon 6 e indique o(s) monmero(s)
que so usados na reao.

UNIVERSIDAD MAYOR DE SAN ANDRES
FACULTAD DE CIENCIAS PURAS Y NATURALES
FACULTAD DE INGENIERIA

________________________________________________________________________________
a) Escreva a equao da reao de sntese do nylon 6 e indique o(s) monmero(s) que
so usados na reao.
nylon 6
b) Escreva a equao da reao de sntese do nylon 6,10 e indique o(s) monmero(s)
que so usados na reao.
nylon 6,10

Polissacardeos
Existem trs polissacardeos principais constitudos por D-glucose, que so: o amido, a
celulose e o glicognio. O amido a principal reserva de alimento nos vegetais, os
componentes do amido so a amilose e a amilopectina. O glicognio tem uma estrutura
semelhante da amilopectina, embora esta apresente cadeias mais ramificadas e funcione
como reserva de hidratos de carbono nos animais. A celulose um polmero muito
insolvel, duro e fibroso que ideal para as paredes das plantas.
O
OH O
OH
OH
CH
2
OH
O
O
OH
OH
CH
2
OH
O
O
OH
OH
CH
2
OH
O
OH
OH
CH
2
OH
O
O
OH
OH
OH
CH
2
OH
O
OH O
OH
OH
CH
2
OH
O
O
OH
OH
CH
2
OH
O
O
OH
OH
CH
2
OH
O
OH
OH
CH
2
OH
O
O
OH
OH
OH
CH
2
OH
O
OH O
OH
OH
CH
2
OH
O
O
OH
OH
CH
2
OH
O
O
OH
OH
CH
2
OH
O
OH
OH
CH
2
OH
O
O
OH
OH
OH
CH
2
OH
A
B
C
n
n
n


b) Escreva a equao da reao de sntese do nylon 6,10 e indique o(s)
monmero(s) que so usados na reao.

UNIVERSIDAD MAYOR DE SAN ANDRES
FACULTAD DE CIENCIAS PURAS Y NATURALES
FACULTAD DE INGENIERIA

________________________________________________________________________________
a) Escreva a equao da reao de sntese do nylon 6 e indique o(s) monmero(s) que
so usados na reao.
nylon 6
b) Escreva a equao da reao de sntese do nylon 6,10 e indique o(s) monmero(s)
que so usados na reao.
nylon 6,10

Polissacardeos
Existem trs polissacardeos principais constitudos por D-glucose, que so: o amido, a
celulose e o glicognio. O amido a principal reserva de alimento nos vegetais, os
componentes do amido so a amilose e a amilopectina. O glicognio tem uma estrutura
semelhante da amilopectina, embora esta apresente cadeias mais ramificadas e funcione
como reserva de hidratos de carbono nos animais. A celulose um polmero muito
insolvel, duro e fibroso que ideal para as paredes das plantas.
O
OH O
OH
OH
CH
2
OH
O
O
OH
OH
CH
2
OH
O
O
OH
OH
CH
2
OH
O
OH
OH
CH
2
OH
O
O
OH
OH
OH
CH
2
OH
O
OH O
OH
OH
CH
2
OH
O
O
OH
OH
CH
2
OH
O
O
OH
OH
CH
2
OH
O
OH
OH
CH
2
OH
O
O
OH
OH
OH
CH
2
OH
O
OH O
OH
OH
CH
2
OH
O
O
OH
OH
CH
2
OH
O
O
OH
OH
CH
2
OH
O
OH
OH
CH
2
OH
O
O
OH
OH
OH
CH
2
OH
A
B
C
n
n
n


Polissacardeos
Existem trs polissacardeos principais constitudos por D-glucose, que so: o
amido, a celulose e o glicognio. O amido a principal reserva de alimento nos
vegetais, os componentes do amido so a amilose e a amilopectina. O glicog-
nio tem uma estrutura semelhante da amilopectina, embora esta apresente
cadeias mais ramifcadas e funcione como reserva de hidratos de carbono nos
animais. A celulose um polmero muito insolvel, duro e fbroso que ideal
para as paredes das plantas.

18
a
OIAQ
Programa Nacional Olimpadas de Qumica <<
168

UNIVERSIDAD MAYOR DE SAN ANDRES
FACULTAD DE CIENCIAS PURAS Y NATURALES
FACULTAD DE INGENIERIA

________________________________________________________________________________
a) Escreva a equao da reao de sntese do nylon 6 e indique o(s) monmero(s) que
so usados na reao.
nylon 6
b) Escreva a equao da reao de sntese do nylon 6,10 e indique o(s) monmero(s)
que so usados na reao.
nylon 6,10

Polissacardeos
Existem trs polissacardeos principais constitudos por D-glucose, que so: o amido, a
celulose e o glicognio. O amido a principal reserva de alimento nos vegetais, os
componentes do amido so a amilose e a amilopectina. O glicognio tem uma estrutura
semelhante da amilopectina, embora esta apresente cadeias mais ramificadas e funcione
como reserva de hidratos de carbono nos animais. A celulose um polmero muito
insolvel, duro e fibroso que ideal para as paredes das plantas.
O
OH O
OH
OH
CH
2
OH
O
O
OH
OH
CH
2
OH
O
O
OH
OH
CH
2
OH
O
OH
OH
CH
2
OH
O
O
OH
OH
OH
CH
2
OH
O
OH O
OH
OH
CH
2
OH
O
O
OH
OH
CH
2
OH
O
O
OH
OH
CH
2
OH
O
OH
OH
CH
2
OH
O
O
OH
OH
OH
CH
2
OH
O
OH O
OH
OH
CH
2
OH
O
O
OH
OH
CH
2
OH
O
O
OH
OH
CH
2
OH
O
OH
OH
CH
2
OH
O
O
OH
OH
OH
CH
2
OH
A
B
C
n
n
n


c) Das substncias apresentadas nas estruturas A, B, C, D, E e F indicar qual
corresponde ao amido e qual corresponde celulose (indique a letra corres-
pondente a cada polmero).
cidos graxos.


UNIVERSIDAD MAYOR DE SAN ANDRES
FACULTAD DE CIENCIAS PURAS Y NATURALES
FACULTAD DE INGENIERIA

________________________________________________________________________________
O
OH
O
OH
OH
CH
2
OH
O
OH
OH
CH
2
OH
O
O
O
OH
OH
CH
2
OH
O
OH
OH
CH
2
OH
O
O
OH
OH
CH
2
OH
OH
O
OH
O
OH
OH
CH
2
OH
O
OH
OH
CH
2
OH
O
O
O
OH
OH
CH
2
OH
O
OH
OH
CH
2
OH
O
O
OH
OH
CH
2
OH
OH
O
OH
O
OH
OH
CH
2
OH
O
OH
OH
CH
2
OH
O
O
O
OH
OH
CH
2
OH
O
OH
OH
CH
2
OH
O
O
OH
OH
CH
2
OH
OH
D
E
F
n
n
n

c) Das substncias apresentadas nas estruturas A, B, C, D, E e F indicar qual
corresponde ao amido e qual corresponde celulose (indique a letra correspondente
a cada polmero).
cidos graxos.
COOH
COOH
COOH
COOCH
3
COOH
HOOC
H
3
COOC
H
HOOC
G
I
J
K
L
M
N

d) Das molculas G, H, I, J, K, L, M, N, qual o cido graxo que tem menor ponto de
fuso (coloque a letra respetiva no quadrado)

e) Das molculas G, H, I, J, K, L, M, N, qual o cido graxo que tem maior ponto de
fuso (coloque a letra respetiva no quadrado)
Exame Terico
>> Olimpada Brasileira de Qumica - 2013
169
d) Das molculas G, H, I, J, K, L, M, N, qual o cido graxo que tem menor ponto
de fuso (coloque a letra respetiva no quadrado)
e) Das molculas G, H, I, J, K, L, M, N, qual o cido graxo que tem maior ponto
de fuso (coloque a letra respetiva no quadrado)
Problema 5
Pontuao do Problema 5 (20 pts).
QUMICA ANALTICA
Problema 5.- PROBLEMA DE ELETROQUMICA
A reao entre o tiossulfato de sdio e iodo uma das reaes qumicas mais
conhecidas da qumica analtica. As solues de iodo podem ser tituladas com
solues de tiossulfato ou vice versa. A utilizao do iodo na determinao
quantitativa de outras espcies qumicas tambm amplamente conhecida e
denomina-se iodometria; por outro lado, as solues de tiossulfato de sdio
tambm tm muitas aplicaes em anlise qumica.
As titulaes das solues de tiossulfato com solues de iodo no so realiza-
das somente pela forma convencional, mas tambm por titulao coulomtrica,
aproveitando as propriedades eletroqumicas dessas substncias.
O carter eletroqumico destas espcies pode ser tambm aproveitado para o
estudo da eletrlise e de pilhas galvnicas.
5.1. (7 pts) - Misturam-se 30,0 mL de uma soluo de tiossulfato de sdio 0,10
mol/L com 50,0 mL de uma soluo de iodo 0,10 mol/L. Quando a reao
atingir o equilbrio:
a) (1 pts) Escreva a equao qumica, devidamente ajustada, que traduz a reao
b) (1 pts) Escreva as equaes das semi-reaes que ocorrem
c) (2 pts) Calcule a concentrao de todas as espcies em soluo.
d) (1 pts) Calcule a constante de equilbrio com as concentraes obtidas
e) ( 2 pts) Calcule a constante de equilbrio com os valores de potenciais padro
das reaes indicadas.
5.2. (6 pts) - Num copo se coloca uma soluo de tiossulfato de sdio com tetra-
18
a
OIAQ
Programa Nacional Olimpadas de Qumica <<
170
tionato de sdio. Noutro copo se coloca uma soluo de iodo e de iodeto de
potssio. Os dois copos so unidos com uma ponte salina, e em cada copo
introduzido um eletrodo de platina, por fm, medido o potencial do sistema.
a) ( 1 pts) Represente o esquema da pilha do sistema anteriormente referido.
b) ( 1 pts) Indique as reaes que se produzem no anodo e no catodo.
c) (2 pts) Com uma seta no esquema indique: o sentido em que circulam os el-
trons.
d) ( 2 pts) Se todas as concentraes so 0,10 mol/L e o pH igual a 1,0 , qual o
potencial da clula?
5.3. (7 pts) - Noutra experincia, se coloca num copo 50,0 mL de gua, se adiciona
2,5 g de iodeto de potssio, 2g de hidrogenocarbonato de sdio, para manter
o pH do meio, e 1,0 mL de soluo de amido como indicador. Introduzem-se
dois eletrodos de platina de 1 cm2 de rea. Agita-se a soluo para dissolver
os sais e faz-se passar corrente at que a soluo fque azul. Interrompe-se a
eletrlise e adiciona-se 2,0 mL de soluo de tiossulfato de sdio de concen-
trao desconhecida. A corrente volta a ser ligada a 150 mA e, ao fm de 140
s, a soluo volta a ter a cor azul tal como antes se tinha observado.
a) (2 pts) Escreva as equaes das reaes que ocorrem no anodo e no catodo
durante a eletrlise.
b) (2 pts) Calcule a concentrao da soluo de tiossulfato de sdio adicionada.
Considere que os volumes so aditivos e despreze a variao de volume re-
sultante da adio dos slidos.
d) (3 pts) Se o tempo usado for apenas 60 s, qual o valor do potencial da soluo?
Exame Terico
>> Olimpada Brasileira de Qumica - 2013
171
Problema 6
Dados
Composio da atmosfera em frao molar: X
O2
=0,21 y X
N2
=0,79
A solubilidade dos gases, Lei de Henry
S = k
H

*
P onde S a solubilidade, k
H
a constante da Lei de Henry e P a presso
do gs sobre a soluo.
Constantes da Lei de Henry a diferentes temperaturas para o oxigenio (O
2
) em
gua.
T / oC 0 10 15 20 25 50 100
k
H
x10
4
/mol L
-1
atm
-1
27,1 18,2 15,7 14,1 12,8 9,37 7,5
Equao Baromtrica

UNIVERSIDAD MAYOR DE SAN ANDRES
FACULTAD DE CIENCIAS PURAS Y NATURALES
FACULTAD DE INGENIERIA

________________________________________________________________________________

Problema 6

Dados

Composio da atmosfera em frao molar:




A solubilidade dos gases, Lei de Henry

S = k
H

*
P onde S a solubilidade, k
H
a constante da Lei de Henry e P a presso do gs
sobre a soluo.

Constantes da Lei de Henry a diferentes temperaturas para o oxigenio (O
2
) em gua.

T /
o
C 0 10 15 20 25 50 100
k
H
x10
4
/mol L
-1
atm
-1
27,1 18,2 15,7 14,1 12,8 9,37 7,5

Equao Baromtrica


g = acelerao da gravidade, h = altura, T = temperatura da atmosfera
Energia de ativao
g = acelerao da gravidade, h = altura, T = temperatura da atmosfera
Energia de ativao

UNIVERSIDAD MAYOR DE SAN ANDRES
FACULTAD DE CIENCIAS PURAS Y NATURALES
FACULTAD DE INGENIERIA

________________________________________________________________________________

]

Pontuao do Problema 6.
6.1 6.2 6.3 6.4 6.5 6.6 Total Ponderado
5 5 5 5 5 5 30 20

QUMICA-FSICA
Problema 6.- Bolvia um pas mineiro, que explora estanho, bismuto, prata, ouro e
outros metais. No norte da cidade de La Paz, o ouro encontra-se como aluvio, sendo o
mtodo de explorao efetuado por gravidade. No altiplano boliviano o ouro encontra-se
principalmente na forma de um p muito fino e no se pode aplicar o mtodo de explorao
por gravidade. Ento, para a sua explorao o ouro dissolvido em solues de NaCN ou
KCN, designando-se este processo por cianetizao. Nesta reao importante a
presena de oxignio molecular j que a dissoluo pode ser descrita pela seguinte equao
qumica:

2 Au + 4 NaCN + O
2
+ 2 H
2
O 2 Na[Au(CN)
2
] + 2 NaOH + H
2
O
2


O pH da soluo deve ser alcalino para evitar a formao de HCN(g), que muito txico.
As solues de Na[Au(CN)
2
] obtidas, so em seguida concentradas por adsoro em
colunas de carvo ativado e desadsoro com uma soluo de NaCN em meio alcalino e a
quente. Finalmente, o ouro pode ser obtido a partir destas solues, por eletrlise ou
precipitao de ouro metlico com p de zinco, processo denominado de cementao.

A cianetizao do ouro de carcter eletroqumico, onde o ouro oxidado a Au
+
, com
formao posterior do on complexo [Au(CN)
2
]

, o agente oxidante o oxignio molecular


que se encontra na soluo, resultante da dissoluo do O
2
existente na atmosfera. A
concentrao do oxignio na soluo depende da presso atmosfrica e da temperatura pelo
que no altiplano boliviano estes aspectos devem ser considerados. Por exemplo, a presso
atmosfrica deve ser corrigida devido altura (4100 m acima do nvel do mar), com a
Equao Baromtrica, considerando que a atmosfera isotrmica a 20 C e a concentrao
de oxignio na soluo obtida pela Lei de Henry.

1.1.- Sabendo que a Empresa Inti Raymi (empresa que explora ouro) se encontra a 4100 m
acima do nvel do mar, calcule a concentrao de oxignio (em mol L
1
) numa soluo a 15
C que intervir na dissoluo do ouro. Considere que a solubilidade do oxignio na
soluo semelhante quela na gua pura.
Pontuao do Problema 6.
6.1 6.2 6.3 6.4 6.5 6.6 Total Ponderado
5 5 5 5 5 5 30 20

18
a
OIAQ
Programa Nacional Olimpadas de Qumica <<
172
QUMICA-FSICA
Problema 6.- Bolvia um pas mineiro, que explora estanho, bismuto, prata,
ouro e outros metais. No norte da cidade de La Paz, o ouro encontra-se como
aluvio, sendo o mtodo de explorao efetuado por gravidade. No altiplano
boliviano o ouro encontra-se principalmente na forma de um p muito fno e
no se pode aplicar o mtodo de explorao por gravidade. Ento, para a sua
explorao o ouro dissolvido em solues de NaCN ou KCN, designando-se este
processo por cianetizao. Nesta reao importante a presena de oxignio
molecular j que a dissoluo pode ser descrita pela seguinte equao qumica:
2 Au + 4 NaCN + O
2
+ 2 H
2
O 2 Na[Au(CN)
2
] + 2 NaOH + H
2
O
2
O pH da soluo deve ser alcalino para evitar a formao de HCN(g), que muito
txico. As solues de Na[Au(CN)
2
] obtidas, so em seguida concentradas por
adsoro em colunas de carvo ativado e desadsoro com uma soluo de NaCN
em meio alcalino e a quente. Finalmente, o ouro pode ser obtido a partir destas
solues, por eletrlise ou precipitao de ouro metlico com p de zinco,
processo denominado de cementao.
A cianetizao do ouro de carcter eletroqumico, onde o ouro oxidado
a Au+, com formao posterior do on complexo [Au(CN)
2
]

, o agente oxidan-
te o oxignio molecular que se encontra na soluo, resultante da dissoluo
do O
2
existente na atmosfera. A concentrao do oxignio na soluo depende
da presso atmosfrica e da temperatura pelo que no altiplano boliviano estes
aspectos devem ser considerados. Por exemplo, a presso atmosfrica deve ser
corrigida devido altura (4100 m acima do nvel do mar), com a Equao Baro-
mtrica, considerando que a atmosfera isotrmica a 20 C e a concentrao de
oxignio na soluo obtida pela Lei de Henry.
1.1.- Sabendo que a Empresa Inti Raymi (empresa que explora ouro) se encontra
a 4100 m acima do nvel do mar, calcule a concentrao de oxignio (em mol
L
1
) numa soluo a 15 C que intervir na dissoluo do ouro. Considere que a
solubilidade do oxignio na soluo semelhante quela na gua pura.
1.2.- Obtm-se a velocidade de cianetizao do ouro, com as concentraes de
NaCN e oxignio constantes, atravs da quantidade de ouro dissolvido por uni-
dade de tempo e rea. Numa experincia para determinar a velocidade de disso-
luo do ouro, se dissolveu uma lmina de ouro de rea conhecida (A = 0,60 cm2)
Exame Terico
>> Olimpada Brasileira de Qumica - 2013
173
em 500 mL de uma soluo com concentraes de NaCN e oxignio constantes,
com um pH de 11,5. A concentrao de ouro dissolvido na soluo em tempos
diferentes foi medida usando um espectrofotmetro de Absoro Atmica. Os
valores obtidos esto apresentados no Quadro 1.1.

Quadro 1.1.- Cianetizao do ouro, rea da lmina de ouro = 0,60 cm
2
,
pH = 11,5, e concentraes de NaCN e O
2
(disol.) constantes
Tempo / min 0 10 20 30 45 60
[Au
+
] / ppm 0 1,55 3,40 5,40 8,13 10,79
ppm = partes por milho
Calcule a velocidade de cianetizao do ouro em mol cm
2
s
1
.
1.3.- Para determinar a constante de velocidade de cianetizao e a ordem da
reao em relao concentrao de CN

, determinou-se a velocidade de cia-


netizao do ouro para diferentes concentraes de NaCN, mantendo-se cons-
tante a concentrao do oxignio na soluo ao pH de 11,5 ; os valores obtidos
esto apresentados no Quadro 1.2.
Quadro 1.2.- Velocidade de cianetizao do ouro para concentraes diferen-
tes de CN

.
[CN

] / mol L
-1
0,001 0,0025 0,00375 0,005
v / mol cm
2
s
1
3,56 x 10
9
10,11 x 10
9
13,80 x 10
9
18,02 x 10
9
Calcule a constante de velocidade e a ordem da reao em relao concentra-
o de CN

.
1.4.- Para determinar a energia de ativao determinou-se a velocidade de
cianetizao do ouro a temperaturas diferentes, mantendo-se constantes as
concentraes de cianeto e de oxignio, e o pH; os valores obtidos esto apre-
sentados no Quadro seguinte:
18
a
OIAQ
Programa Nacional Olimpadas de Qumica <<
174
T / C 20 25 35 40 55
V x 109 / mol cm
-2
s
-1
11,67 12,28 13,76 14,28 16,48
Calcule a energia de ativao em kj mol
-1
1.5.- A dissoluo do ouro em solues de cianeto de carter eletroqumico,
escreva as equaes das semi-reaes que ocorrem na superfcie do ouro.
1.6.- O carvo ativado uma substncia tal que um grama de carvo pode che-
gar a ter uma rea superfcial de 1000 m
2
. Determina-se a superfcie do carvo
ativado por adsoro do nitrognio molecular, N
2
, considerando que sobre a
superfcie do carvo se forma unicamente uma monocamada de molculas de
nitrognio (o N2 adsorvido sobre o carvo formando uma camada da espes-
sura de uma molcula). Sabendo que cada molcula de nitrognio ocupa uma
superfcie de 16
2
e, conhecendo a quantidade de nitrognio adsorvido, pode
determinar-se a superfcie do carvo.
a) Numa experincia para determinar a superfcie de um carvo fabricado na
UMSA a partir de cascas de nozes e ativado com vapor de gua, procedeu-
-se adsoro de nitrognio gasoso e observou-se que 1,10 g do carvo
ativado pode adsorver 3,52 L de nitrognio gasoso a 495 mmHg e 20 C. Qual
a superfcie do carvo, em m
2
?
B) Se for utilizado o mesmo carvo ativado para adsorver Na[Au(CN)
2
] e, saben-
do que este ocupa uma superfcie aproximada de 50
2
, que massa de ouro
(mg) por grama de carvo pode ser adsorvida?
Exame Terico
>> Olimpada Brasileira de Qumica - 2013
175
Destaques Olmpicos
A escolha dos estudantes que formaram as delegaes brasileiras nas olim-
padas internacionais de 2013 teve incio na Fase IV da olimpada. Nesta estavam
seguir 43 estudantes, aqueles medalhados na etapa anterior (Fase III).
A Fase IV constituiu de exibio de vdeo com demonstrao de alguns expe-
rimentos qumicos sobre os quis os participante deveriam responder perguntas
e apresentar um relatrio. Como resultado, tivemos quinze estudantes classif-
cados conforme lista: Cear: Francisco Markan Nobre de Souza Filho, Joaquim
Ivo Vasques Dantas Landim, Lia de Oliveira Domingues, Lvia Rodrigues de
Arajo, Mateus Caracas Veras, Mauroclio Rocha Pontes Filho, Nathrcia Castro
Mota, Nicholas de Souza Costa Lima e Pedro talo Oliveira Gomes; Rio de Janeiro:
Nathan de Souza Mateus, Pedro Henrique Fonseca Duque e Thiago Silva Viana;
So Paulo: Rafael Tedeschi Eugnio Pontes Barone e Luis Fernando Machado
Poletti Valle; Bahia: Thiago Matheus Santos Rios.
A esses quinze estudantes juntou-se a aluna Vitria Medeiros, selecionada
anteriormente por critrio hours concours e seguiram para Teresina onde se
cumpriu a fase seguinte.
DelegaobrasileiraemMoscou:Prof.Arimatia,Vitria,Mauroclio,LviaeNicholas.
Destaques
Programa Nacional Olimpadas de Qumica <<
176
Destaques
A seleo foi fnalizada na Fase VI, quando foi aplicado exame preparado por
professores da UFPI com base nos cursos por eles ministrados na Fase V, classi-
fcamos cinco estudantes. Nicholas, Lvia e Mauroclio, juntamente com Vitria,
participaram da 45
th
International Chemistry Olympiad, em Moscou. Natrcia e
Pedro talo substituram, respectivamente, Vitria e Lvia e formaram a equipe
que representou o Brasil na 18
a
Olimpada Ibero-americana de Qumica, na Bolvia.
Vitria,medalhadebronzena45
th
IChO.
Nathrcia,medalhadepratana18OIAQ
Lvia,medalhadebronzena45
th
IChO.
Pedro,medalhadepratana18OIAQ
>> Olimpada Brasileira de Qumica - 2013
177
Nicholas,medalhadebronzena45
th
IChO
emedalhadeourona18
a.
OIAQ.
Mauroclio,medalhadebronzena45
th
IChO
emedalhadebronzena18
a.
OIAQ
DelegaobrasileiranaBolvia:Pedrotalo,Mauroclio,NathrciaeNicholas.
Destaques
Programa Nacional Olimpadas de Qumica <<
178
Depoimentos
Olimpadas de Qumica, parte da minha histria
Comecei a participar de olimpadas cientfcas, em 2006, na rea de mate-
mtica. Em 2007, um amigo me convidou para frequentar aulas noturnas de
olimpada de qumica na minha escola. No primeiro dia de aula prtica, logo me
apaixonei por qumica. Lembro-me bem do experimento que me deixou alu-
cinado por qumica: o experimento da chama fria. Estudei bastante e comecei
a competir em nvel estadual participando da Maratona Cearense de Qumica
e da Olimpada Cearense de Qumica, sendo logo classifcado para a olimpada
nacional (OBQ).
Em 2009, fz pela primeira vez a OBQ, conquistando a medalha de prata que
me garantiu uma vaga na seletiva para a IChO 2010. Ento, passei na primeira
fase da seletiva e fui para o curso de aprofundamento e excelncia em qumica
em Campinas, minha primeira viagem olmpica, mas no consegui uma das qua-
tro vagas para a IChO (Olimpada Internacional de Qumica): acabei me conten-
tando com o oitavo lugar na seletiva fnal. Tive que aprender a superar minha
frustrao pois teria mais uma oportunidade. Em 2010, novamente conquistei
uma medalha de prata na OBQ, sendo mais uma vez classifcado para a seletiva
da IChO. Passei da primeira fase da seletiva e fui pela segunda vez para o curso
de aprofundamento e excelncia na UNICAMP. Como tinha experincia do ano
anterior, consegui conquistar uma melhor colocao (segundo) e garanti minha
vaga para a IChO 2011, que seria realizada na cidade de Ancara, Turquia. Essa
viagem foi um marco na minha vida, pois conheci pessoas de muitos pases di-
ferentes, troquei muitas ideias e desfz muitos preconceitos que tinha de vrios
pases. Conquistei uma medalha de prata nessa olimpada, o que me garantiu
uma vaga para a Olimpada Ibero Americana de Qumica do mesmo ano, que
seria realizada em setembro de 2011, em Teresina. A OIAQ foi a olimpada mais
especial da qual participei, fquei muito amigo das pessoas que conheci l, tenho
excelentes lembranas em solo piauiense. Conquistei medalha de Ouro e contri-
bu para a conquista do melhor resultado brasileiro na OIAQ.
Graas ao meu currculo olmpico, consegui ser aceito em uma universidade
americana e hoje estudo na University of Southern California em Los Angeles
com bolsa do governo brasileiro, fruto do meu currculo construdo principal-
mente pelo meu sucesso nas olimpadas de qumica. No me arrependo de ter
perdido vrias noites e fnais de semana estudando para conquistar meus obje-
tivos. Para a gerao mais nova deixo uma mensagem para quando se sentirem
desestimulados a participarem da olimpadas: Estudem, vocs no imaginam a
>> Olimpada Brasileira de Qumica - 2013
179
Depoimentos
quantidade de portas que essas competies podem abrir pra vocs, as olimpa-
das transcendem o universo escolar chegando a ajudar no seu futuro universi-
trio e profssional. Portanto, vale muito a pena participar das olimpadas de
qumica. Fico muito feliz pelas olimpadas de qumica fazerem parte da minha
histria.
A educao necessria para enfrentar o mundo. Choremos pelos que perde-
ram a vida na boate ao respirar o ar venenoso, e pelos que no vo receber nas
escolas.
Davi Rodrigues Chaves
PMedalha de ouro na XVI OIAQ, Teresina.
Medalha de prata na 43rd IChO, Ancara.
Medalhas de prata OBQ 2009 e OBQ 2010.
Graduando na University of Southern California.
Olmpiadas e o modelo de educao brasileiro
Meu envolvimento com cincias comeou cedo, graas ao incentivo da mi-
nha me biloga. Quando criana, meu passatempo favorito era fazer poes,
misturando tudo o que podia encontrar em casa, e rezando para que a mistura
iniciasse alguma reao, fazendo meus olhos brilharem, e infestando a casa com
odores insuportveis. A bronca valia a pena, e eu continuava me divertindo e
tentando entender a cincia por trs das minhas brincadeiras.
Eu sempre fui o aluno pentelho que fazia pergunta aps pergunta, e nunca
se cansava de explorar como o mundo funcionava. Parecia mais que natural que
qumica seria uma de minhas matrias favoritas, e que minha paixo por cincias
seria duradoura. Na minha primeira aula de qumica na escola, minha expecta-
tiva era grande, mas aps as primeiras aulas, a minha paixo pareceu se apagar.
O que encontrei na sala de aula foram incontveis defnies, frmulas e pro-
priedades para decorar, sem nenhum enigma interessante para investigar. Nada
parecia desafador, e qumica se tornou rapidamente uma das minhas matrias
mais odiadas no ensino fundamental.
Ensino mdio comea, e meu dio por qumica continua. Qualquer co-
mentrio sobre pH ou ligaes qumicas me fazia tremer de agonia. No mesmo
ano, no entanto, eu comecei a me envolver em olimpadas, e decidi tentar tam-
bm a OBQ. Ao fazer a prova naquele ano, eu esperava encontrar questes sim-
ples e similares s das minhas provas do colgio. O que encontrei, no entanto,
Programa Nacional Olimpadas de Qumica <<
180
foram problemas desafadores, surpreendentemente interessantes, e claramente
acima do meu nvel de conhecimento da matria.
Ao sair da prova, eu senti que eu fnalmente havia reencontrado a parte da
qumica que eu sempre amei, e que o modelo de educao brasileiro, focado em
provas de vestibular e decoreba, havia escondido de mim. As questes da prova
no apenas me desafaram, mas tambm me deixaram curioso e entusiasmado
para estudar mais e ter um melhor desempenho no ano seguinte.
Como minha escola ainda no oferecia aulas para olimpadas de qumica, co-
mecei a estudar por mim mesmo, e a explorar bibliotecas em busca de livros
para me preparar. Atkins se tornou meu melhor amigo, e eu recomecei a pente-
lhar professores de qumica com cada vez mais perguntas sobre a matria que
eu havia reaprendido a amar. Meu esforo valeu a pena, e no meu segundo ano
do ensino mdio, eu recebi minha primeira medalha da OBQ prata em 2008. A
medalha tambm me classifcou para o programa de seleo da IChO, e minhas
expectativas aumentaram ainda mais.
Aps um ano de seletiva, centenas de pginas estudadas, e um curso incrvel
na UNICAMP oferecido pela OBQ, eu recebi a notcia de que eu no havia sido
selecionado, fcando a duas posies da minha vaga para a IChO. Apesar de devas-
tado com a noticia, eu me senti ainda mais desafado a continuar estudando para
a minha ltima chance na OBQ, em 2009.
Minha paixo por qumica havia crescido tanto que comecei a dar aulas de
olimpada e criar simulados para colegas da escola e a visitar o laboratrio da
escola vrias vezes por semana. No fm do meu ensino mdio, eu recebi uma das
noticias mais gratifcantes da minha vida: eu no apenas havia recebido uma me-
dalha de ouro na OBQ, mas tambm havia alcanado o primeiro lugar geral do
pas na olimpada.
Este e outros sucessos em olimpadas me proporcionaram maturidade e um
currculo que me garantiram uma vaga em algumas das universidades mais pres-
tigiosas do mundo, entre elas a Harvard University, onde hoje estudo. J no meu
terceiro ano na universidade, qumica ainda uma de minhas maiores paixes.
Apesar de agora estudar cincias da computao, eu ainda encontro tempo para
fazer matrias de qumica orgnica por diverso, e para visitar laboratrios e
supervisionar experimentos.
Com total certeza, a OBQ mudou minha vida drasticamente, e eu devo muito
ela e seus idealizadores. A OBQ trouxe de volta meu entusiasmo por qumica e
cincia, e me apresentou a um universo que eu no haveria conhecido se tivesse
me contentado com o que aprenderia na escola. Tenho sorte de ter estudado por
sete anos em uma escola excepcional com professores maravilhosos, o Colgio
Depoimentos
>> Olimpada Brasileira de Qumica - 2013
181
Militar de Braslia, mas o modelo de educao brasileiro ainda tem muito a mu-
dar. Nessa luta por trocar decoreba por desafos, as olimpadas cientfcas so
nossos maiores aliados.
Nos ltimos anos, colgios como o meu comearam a dar grande importn-
cia a tais competies e oferecer aulas de preparao para elas. Um maior envol-
vimento em tais competies gerou um crescimento exponencial no nmero de
medalhas recebidas pela minha escola, no de formandos que estudam cincias, e
no de ex-alunos estudando em universidades de ponta no exterior. A correlao
entre olimpadas e paixo por cincias evidente, e o envolvimento de escolas
em tais competies importantssimo.
O Brasil perde potenciais cientistas talentosos todos os dias devido ao atual
modelo de ensino de cincia nas escolas, e tal desperdcio imperdovel. En-
quanto uma reforma absoluta na educao no implementada, devemos agra-
decer a competies como a OBQ, que mudam a vida de centenas de alunos por
ano, e esperar que um dia olimpadas cientfcas e olimpadas esportivas sero
tratadas pelo nosso governo com a mesma importncia.
Lucas Daniel Gonzaga de Freitas
Harvard University, 2011 2015
Aprovado nas universidades de Harvard, MIT, Yale, e Dartmouth
OBQ: prata em 2008, ouro em 2009
OQDF: ouro e primeiro lugar geral em 2008 e 2009
OBF: ouro em 2007 e 2009, prata em 2008
OBA: ouro em 2008 e 2009
OBMEP: ouro em 2007, 2008, e 2009
Depoimentos
Programa Nacional Olimpadas de Qumica <<
182
Consideraes
Perspectivas de ampliao das olimpadas e qumica
Um novo ciclo se fecha no Programa Nacional Olimpadas de Qumica, agora
com nmeros mais expressivos que o ano anterior, 1780 estudantes com pre-
miaes, motivo de orgulho e de muita satisfao para todos os coordenadores
e colaboradores que voluntariamente somaram esforos para atingir esse xito.
Adquirindo musculatura ao longo dos anos, o Programa Nacional Olimpa-
das de Qumica prepara-se para ultrapassar um desafo maior, organizar uma
olimpada voltada exclusivamente para estudantes das escolas pblicas. Apoia-
do pelo MEC partiremos, em 2014, para um projeto de dimenses mais largas na
expectativa de superar a marca de 3,5 milhes de estudantes participantes. A
equipe constituda para planejar as aes e defnir um calendrio operacional
discutiu e avaliou a abrangncia dessa olimpada e se prepara para apresentar
ao MEC sua proposta de execuo do evento. Ser o coroamento do trabalho de
nossa equipe na ocasio em que iremos comemorar 20 anos de atividades inin-
terruptas, desde 1994, ano em que semeamos uma proposta, no to ambiciosa,
para olimpadas de qumica.
Temos a ressaltar os estudantes das escolas pblicas que participaram nas
quatro modalidades de olimpadas realizadas em 2013, eles foram mais presen-
tes nos resultados, um maior nmero deles recebeu premiao, destaque para
o estudante cenecista Aleson Sstenes Valentim que alcanou o lugar de maior
destaque na VI Olimpada Brasileira de Qumica Jnior, medalha de ouro. Segu-
ramente, a escola cenecista no uma tpica escola pblica, mas, em face de seu
carter flantrpico e da natureza das comunidades nas quais esto inseridas,
pode se considerar sua equivalncia com a escola pblica. Tambm, se desta-
caram muitos dos estudantes que participam do projeto Aes Construtivas do
Conhecimento Qumico nas Escolas Pblicas apoiado pela CAPES, dos cursos
ministrados nos interiores brasileiros surgiram promissores talentos. -me gra-
to v-los sobressados dentre os 189 mil estudantes que participaram das olim-
padas de qumica. Congratulaes aos licenciandos em qumica, atuantes nesse
projeto, cujas tcnicas educacionais, ainda em cristalizao, mostraram-se ef-
cientes nesse propsito.
O quadro de medalhas fcou melhor distribudo, deixaram de se concentrar
em estados tradicionalmente monopolizadores dessas premiaes para favore-
cer outras regies brasileiras. Isso demonstra o empenho que os coordenadores
estaduais tiveram na divulgao, na ampliao do raio de ao e na realizao
>> Olimpada Brasileira de Qumica - 2013
183
Consideraes
de solenidades nas quais foram reconhecidos os esforos das escolas e dos seus
professores para aperfeioar o ensino de qumica.
A participao brasileira nas olimpadas internacionais continuou bem des-
tacada na Olimpada Iberoamericana de Qumica, na edio atual acontecida
em La Paz Bolvia a equipe brasileira trouxe em sua bagagem quatro medalhas,
sendo uma de ouro, duas de prata e uma de bronze. Na Olimpada Internacional
de Qumica manteve-se a hegemonia dos pases asiticos, as mais destacadas
premiaes foram conquistadas por estudantes dessa regio. Os quatro jovens
que formaram a equipe brasileira foram agraciados com medalhas de bronze,
um resultado que expressa o bom nvel de conhecimentos nos exigentes exa-
mes a eles aplicados. Ainda temos um caminho a percorrer no sentido de suprir
as defcincias que os estudantes apresentam no desempenho em laboratrio
quando participam de olimpadas internacionais, so pouqussimas as escolas
de ensino mdio que mantm laboratrios abertos aos seus alunos, sem eles o
aprimoramento dos estudos fca comprometido.
Prof. Srgio Maia Melo
Coordenador do Programa
Nacional Olimpadas de Qumica
Programa Nacional Olimpadas de Qumica <<
184
Endereos
Endereos Coordenadorias
NCLEO COORDENADOR ENDEREO PROFISSIONAL
COORDENADORIA GERAL Prof. Srgio Maia Melo
melo@ufc.br
obq@ufc.br
Programa Nacional Olimpadas de
Qumica
Rua Antnio Lima, 191
60.115-270 Fortaleza - CE
COORDENADORIA REGIES SUL/SUDESTE Prof. lvaro Chrispino
alvaro.chrispino@gmail.com
CEFET - RJ
(21) 642.6644 (Telefax)
Vice-COORDENADORIA Prof. Jos Arimatia Dantas Lopes
arilopes@gmail.com
arilopes@ufpi.br
Universidade Federal do Piau
Centro de Cincias da Natureza
64.049-550 Teresina - PI
(86) 3215.5840 3215.5692 (Telefax)
ESTADO COORDENADOR ENDERECO PROFISSIONAL
ACRE Prof. Dlcio Dias Marques
delciomarques@globo.com
Prof. Rogrio A. Sartori
rogerio_sartori@yahoo.com.br
Universidade Federal do Acre - CCN
Depto. de Ciencias da Natureza BR 364 Km 4
63.915-900 Rio Branco - AC
(68) 3901.2591
ALAGOAS Prof. Joacy Vicente Ferreira
joacyferreira@ifal.edu.br
IFAL - Campus Macei
Instituto Federal de Alagoas
Rua Mizael Domingues, 75 Poo
57.020-600 Maceio - AL
Fone: (82) 2126-7000 / 7024 2126 .7050 (fax
AMAP
www.unifap.br/olimpiadas
Prof. Roberto Messias Bezerra
messias@unifap.br
Universidade Federal do Amap
Rod. Juscelino Kubitscheck, Km 02
68.902-280 Macap - AP
Fone: (96)3312-1700
AMAZONAS
www.oaq.ufam.edu.br
olimpadasdequimica@
hotmail.com
Profa. Tereza Cristina Souza de Olivei-
raterezacristina@ufam.edu.br
Prof . Paulo Rogrio da Costa Couceiro
couceiro@ufam.edu.br
Universidade Federal do Amazonas
Instituto de Cincias Exatas - Departamento de
Qumica - Bloco 10 - Departamento de Qumica/
ICE/UFAM Setor Norte do Campus Universitrio
Sen. Arthur Virglio Filho
Av . Gal . Rodrigo Otvio Jordo Ramos, 6.200
69.077-000 Manaus - AM Coroado
(92) 3305-2874 (telefax)
>> Olimpada Brasileira de Qumica - 2013
185
BAHIA
www.obaq.ufba.br
Prof . Lafaiete Almeida Cardoso
lafaiete@ufba.br
Universidade Federal da Bahia
Instituto de Qumica - Depto . Qui . Orgnica
Rua Baro de Geremoabo, s/n (Ondina) 40 .170-
115 Salvador - BA
(71) 3283 .6813 3237.4117 (Fax)
CEAR
http://www.necim.ufc.br
Prof. Leonilde Maria Cmara Jatahy
necim@bol.com.br
leojatahy@ig.com.br
Prof. Cludia Christina B. S. Carneiro
Universidade Federal do Cear
NECIM - Ncleo de Ensino de Cincias e Matemtica
Av. da Universidade, 2470
60020-180 Fortaleza - Cear (85) 3366.7796
DISTRITO FEDERAL
http://www.petiq.unb.br/
Profa. Elaine Rose Maia
emaia@unb.br elaine.rose.maia@
gmail.com petqui@unb.br
Universidade de Braslia - Instituto de Qumica
Campus Universitrio Darcy Ribeiro -
70910-970 ICC Sul - Asa Norte Cx.Postal: 04478
(61) (61) 3107-3895 / 3893 / 3806
ESPRITO SANTO
www.cce.ufes.br/dqui/
ocq-es
Prof. Carlos Vital Paixo de Melo
cvpaixao@npd.ufes.br
Universidade Federal do Esprito Santo
Departamento de Qumica - CCE
Av. Fernando Ferrari, 514 Goiabeiras
29.075-910 Vitria ES
(27) 3335.2486 3335.2826
GOIS
http://www.jatai.cefetgo.
br/licenciatura/OBQ.html
www.obqgoias.com.br
Renato Cndido da Silva
obqgoias@yahoo.com.br
Prof. Carlos Czar da Silva
cefetjatai@yahoo.com.br
Prof. Hernane de Toledo Barcelos
Universidade Federal de Gois, Instituto de Qumica
Campos II Samambaia Bloco I
74001-970 Goiania, GO
Telefone: 62 3521.1167
Centro Federal de Educao Tecnolgica de Gois
UNED JATA
Rua Riachuelo, 2090 Setor Samuel Graham
75.800-000 Jata - GO
(64) 3632.8600
Centro Federal de Educao Tecnolgica de Gois
Campus Goinia
Rua 75, n 46, Centro.
74055-110. Goinia - GO
(62) 3227-2700
MARANHO Prof. Jean Carlo Antunes Catapreta
jcac889@hotmail.com
Prof. Roberto Batista de Lima
rblimas@gmail.com
Universidade Federal do Maranho
Departamento de Qumica
Av. dos Portugueses, s/n Campus da Bacanga
65.080-805 So Luis - MA
Fone: (98) 3272.8227 / 9241
MATO GROSSO Prof. Luiz Both
bothluiz@ibest.com.br ou luiz_both@
hotmail.com
luiz.both@blv.ifmt.br
IFMT
Rua 28, Quadra 38, Casa 14, Jardim Universitrio
78.075-592 Cuiab - MT
Fone: 65 3653.9206 (IFMT) 3663.1374
Endereos
Programa Nacional Olimpadas de Qumica <<
186
Endereos
MATO GROSSO DO SUL Prof. Onofre Salgado Siqueira
olimpiada.quimica.ms@gmail.com
Universidade Federal do Mato Grosso do Sul
Instituto de Qumica
Rua Filinto Muller, 1555 (Cidade universitria)
79.070-900 Campo Grande - MS
(67) 345.3556 345.3552 (FAX)
MINAS GERAIS
http://www.qui.ufmg.br/
omq/
Profa. Ana Luiza de Quadros
omq.ufmg@gmail.com
Universidade Federal de Minas Gerais
Departamento de Qumica
Instituto de Cincias Exatas (ICEx)
Av. Pres. Antnio Carlos, 6627 Pampulha
31.270-901 Belo Horizonte - MG
(31) 3409.7558 Fax: (31) 3499.5700
PAR Prof. Mrcio de Souza Farias
toraqk@yahoo.com.br
Profa. Patrcia da Luz
pdaluz@yahoo.com
Instituto Federal do Par - IFPA
Departamento de Qumica
Avenida Almirante Barroso, 1155
Bairro do Marco
66093-020 Belm - PA
PARABA Prof. Vimario Simes Silva
vimario@deq.ufcg.edu.br
Francisco Ferreira Dantas Filho
Prof. Jos Estrela dos Santos
santosje@gmail.com
Universidade Federal de Campina Grande - Unidade
Acadmica de Engenharia Qumica
58.109-900 Campina Grande (83) 2101.1115
Universidade Estadual da Paraba UEPB -Campus
VIII - Centro de Cincias Tecnologia e Sade
Rua Coronel Pedro Targino SN, Centro - CEP:
5823300 - Araruna -PB - Telefone: (83) 3373-1040
- (83) 9919 -7772
Universidade Federal de Campina Grande
Rua Srgio Moreira de Figueiredo, S/N
Bairro: Casas Populares - 58.900-000 Cajazeiras - PB
(83) 3532-2040 3532.2063 3531.3940 (FAX)
PARAN
http://www.oprq.daqbi.
ct.utfpr.edu.br/
Prof. Jos Carlos Colombo
colombo@utfpr.edu.br
Universidade Tecnolgica Federal do Paran
Campus Curitiba - Sede Ecoville
Rua Dep. Heitor Alencar Furtado, 4900
81280-340 - Curitiba - PR - Brasil
Fone: (41) 3279.4575
PERNAMBUCO
www.espacociencia.pe.gov.
br/index.php/atividade/
quimica/
Prof. Antnio Carlos Pavo
pavao@ufpe.br
Vice-coordenadora:
Profa. Lindomar Silva
lindomar.avelino88@gmail.com
Espao Cincia - Memorial Arcoverde, Complexo de
Salgadinho Olinda -PE
Fone: 81-3183.5525 / 3183.5528
Universidade Federal de Pernambuco, CCEN, Depto.
de Qumica Fundamental, Cidade Universitria, 50
740-521 Recife - PE
Fone: 81-2126.7415 / 2126 .8442 (fax)
Endereos
>> Olimpada Brasileira de Qumica - 2013
187
PIAU
http://www.ufpi.br/
quimica/opq/
Prof. Jos Milton Elias de Matos
jmematos@gmail.com
Universidade Federal do Piau
Depto de Qumica - SG2 CCN - Campus da Ininga
64049-550 - Teresina - PI
Fone: (86) 3215.5620 telefax
RIO DE JANEIRO
http://sites.google.com/
site/olimpiadadequimicarj/
Prof. Paulo Chagas
paulo.chagas@ifrj.edu.br
Prof. Luis Carlos de Abreu Gomes
luisquimica@globo.com
IFRJ - Instituto Federal de Cincia e Tecnologia do
Rio de Janeiro - Rua Senador Furtado, 121 Praa da
Bandeira - 20.270-021 Rio de Janeiro - RJ
(21) 3978. 5918 3567.0283 (Fax)
CIEP 436 Neusa Brizola - 24.425 - 004 Neves - So
Gonalo - RJ - Fone: 21 9795 0176
Rua Dr. Jos Augusto Pereira dos Santos s/n
Fone: 21 8897-4492
RIO GRANDE DO NORTE
oqrn.quimica.ufrn.br
Prof. Fabiano do Esprito Santo Gomes
feibi_natal@yahoo.com.br
Profa. Maria de Ftima Vitria de
Moura
mfvmoura@quimica.ufrn.br
Universidade Federal do Rio Grande do Norte
Centro de Cincias Exatas e da Terra
Depto de Qumica
Av. Senador Salgado Filho, 3000 Lagoa Nova
Campus Universitrio
59.072-970 Natal - RN
Fone: (84) 3215.3828 R: 222 3215.9224 (Fax
RIO GRANDE DO SUL
http://gaia.liberato.com.br/
olimpiada/index.php
Twitter: http://twitter.com/
oqdors
Coordenao Colegiada:
Prof. Daniel Jacobus
Prof. Fvio Roberto Becker Dillio
Prof. Sabrina da Silva Bazzan
oqdors@gmail.com
quimica@liberato.com.br
Fundao Escola Tcnica Liberato Salzano V. da
Cunha
Rua Inconfdentes, 395 Primavera
93.340-140 - Novo Hamburgo - RS
(51) 3584.2027
RONDNIA Prof. Jandi Costa jandi@gmail.com
Prof. Maral Frana
Fundao Universidade Federal de Rondnia- UNIR
Laboratrio de Qumica Analtica de Solos
Departamento de Qumica BR 364 km 9
78.000-000 Porto Velho - RO
Fone (69) 2182.2193
RORAIMA Profa. Maria Lcia Taveira
taveiraml@ig.com.br
Universidade Federal de Roraima
Departamento de Qumica - Campus do Paricarana
69.301-270 Boa Vista - RR
Fone: (95) 621.3140 621.3137 623.1581 224.7302
(Fax)
SANTA CATARINA Prof. Gilson Rocha Reynaldo
gilson@unisul.br
Jos Maximiliano Muller Netto
max@crqsc.gov.br
Universidade do Sul de Santa Catarina - UNISUL
Av. Jos Accio Moreira, 787 - Caixa postal 370
88.704-900 Tubaro - SC Bairro Dehon Fone: (48)
621.3371 - Fax (48) 621 3000
Endereos
Programa Nacional Olimpadas de Qumica <<
188
SO PAULO
http://allchemy.iq.usp.br
Prof. Ivano G. R. Gutz
abqsp@iq.usp.br
gutz@iq.usp.br
ABQ Regional So Paulo
Instituto de Qumica da USP
Av. Prof. Lineu Prestes, 748 sala 1274
05.508-000 So Paulo - SP
(11) 3091.2159 (Mirian, 8 -12h) 3091.2150
SERGIPE
www.osequim.com.br
Prof. Juvenal Carolino da Silva Filho
jcarolino@hotmail.com
Profa. Eliana Midori Sussuchi
esmidori@gmail.com
Universidade Federal de Sergipe - Depto. de
Qumica
Campus Prof. Alberto Carvalho
Av. Vereador Olimpio Grande, s/n Centro
49.500.000 Itabaiana - SE
Fone: (79) 3431.8216 3432.8200
TOCANTINS
http://olimpiadatocanti-
nensedequimica.blogspot.
com.br/
Prof. Jos Expedito Cavalcante da Silva
mailto:eliete@unitins.br
jecs@mail.uft.edu.br
Universidade Federal do Tocantins
Coordenao de Qumica,
Rua Paraguai, s/n (esquina com Urixamas) - Setor
Cimba
77.838-824, Araguana-TO
Fone: 63 81112869, 63 21122201
PATROCINADORES
ABICLOR - Associao Brasileira da Indstria de lcalis e Cloro
Derivados
www.abiclor.com.br
Rua Chedid Jafet, 222 Bloco C 4 andar Vila Olmpia
04.551-065 So Paulo SP
(11) 2148.4780 FAX 2148.4788
ABIQUIM - Associao Brasileira da Indstria Qumica
www.abiquim.org.br
Rua Chedid Jafet, 222 Bloco C 4 andar Vila Olmpia
04.551-065 So Paulo SP
(11) 2148.4700 FAX 2148.4760
Banco do Nordeste do Brasil SA
www.bnb.gov.br
Av. Paranjana, 5700 Castelo
60.180-420 Fortaleza - CE
(85) 4005.3300
PROMOTORES
Universidade Federal do Cear
Pr-Reitoria de Extenso
Av. da Universidade, 2932 Campus do Benfca
60.020 Fortaleza - CE
Fone: (85) 3366.7300
Universidade Estadual do Cear
Pr-Reitoria de Extenso
Av. Dede Brasil, 1700 Paranjana
60.740-000 Fortaleza- CE
Fone: (85) 3299.2555
Universidade Federal do Piau
Pr-Reitoria de Extenso
Campus da Ininga
64.049-550 Teresina - PI
Fone: (86) 3215.5692 Fax: (86) 215.5570
Endereos
>> Olimpada Brasileira de Qumica - 2013
189
APOIO
CNPq - Conselho Nacional de Desenvolvimento Cientfco e
Tecnolgico
SHIS QI 1 Conjunto B - Bloco D, 2 andar
Edifcio Santos Dumont, Lago Sul
71605-190 Braslia - DF
Fone: (61) 3211-9408
CAPES - Coordenao de Aperfeioamento de Pessoal de
Nvel Superior
Ministrio da Educao - MEC
Setor Bancrio Norte, Quadra 02, Bloco L, Lote 6, 4. Andar
70.040-020 - Braslia/DF
Associao Brasileira de Engenharia Qumica
abeq@abeq.org.br
Rua Lbero Bardar, 152 - 11 andar - Centro
01008-903 So Paulo - SP
Fone: (11) 3107-8747 ou Telefax: (11) 3104-4649
Associao Norte-Nordeste de Qumica
www.annq.com
Caixa Postal 167
69.301-970 Boa Vista RR
Fone: (95) 3224.7128
Espao Cincia
www.espacociencia.pe.gov.br
Complexo de Salgadinho s/n Parque 2
53.111-970 Olinda PE
Fone: (81) 3301-6139
Fundao Estudar
www.estudar.org.br
Rua Tabapu, 145 Itaim Bibi
So Paulo - SP
REALIZADOR
Associao Brasileira de Qumica
www.abq.org.br
Av. Presidente Vargas, 633 sala 2208
20071-004 Rio de Janeiro RJ
Telefone: 21 2224-4480 Fax: 21 2224-6881
E-mail: abqrj@alternex.com.br
Endereos

Você também pode gostar